Vous êtes sur la page 1sur 107

JURISDICTION

without restraint of the person, and that it is not in the form required by law. The court denied the motion and proceeded to hear the case. Issue: Whether or not he court may, of itself and on its own motion, create not only a process but a procedure by which the process may be made effective. Held:

WEST COAST LIFE INSURANCE CO. VS. GEO HURD, JUDGE OF COURT OF FIRST INSTANCE March 30, 1914 G.R. No. L-8527 Facts: Petitioner is a foreign life-insurance corporation, duly organized under and by virtue of the laws of the State of California, doing business regularly and legally in the Philippine Islands pursuant to its laws. The assistant prosecuting attorney of the city of Manila filed an information in a criminal action in the Court of First Instance of that city against the plaintiff, said corporation, and also against John Northcott and Manuel C. Grey, charging said corporation and said individuals with the crime of libel. The defendant in his official capacity as judge of the court of First Instance signed and issued a process directed to the plaintiff and the other accused in said criminal action, summoning them to appear before him to answer the charges filed. However, the plaintiff, together with the other persons named as accused in said process filed with the clerk of the court a motion to quash said summons and the service thereof, on the ground that the court had no jurisdiction over the said company, there being no authority in the court for the issuance of the process, the order under which it was issued being void. Plaintiffs argue that under the laws of the Philippine Islands, to proceed against a corporation, as such, criminally, to bring it into court for the purpose of making it amenable to the criminal laws. The plaintiff, further attacking said process, alleges that the process is a mixture of civil and criminal process, that it is not properly signed, that it does not direct or require an arrest; that it is an order to appear and answer on a date certain

While having the inherent powers which usually go with courts of general jurisdiction, they have only such authority in criminal matters as is expressly conferred upon them by statute or which it is necessary to imply from such authority in order to carry out fully and adequately the express authority conferred. The Courts of First Instance do not have authority to create new procedure and new processes in criminal law. The exercise of such power verges too closely on legislation. Even though it be admitted that there are various penal laws in the Philippine Islands which corporation as such may violate, still the courts are not authorized to go to the extent of creating special procedure and special processes for the purpose of carrying out those penal statutes, when the legislature itself has neglected to do so. To bring a corporation into court criminally requires many additions to the present criminal procedure. While it may be said to be the duty of courts to see to it that criminals are punished, it is no less their duty to follow prescribed forms of procedure and to go out upon unauthorized ways or act in an unauthorized manner. There are many cases cited by counsel for the defendant which show that corporations have been proceeded against criminally by indictment and otherwise and have been punished as malefactors by the courts. Of this, of course, there can be no doubt; but it is clear that, in those cases, the statute, by express words or by necessary intendment, included corporations within the persons who could offend against the criminal laws;

and the legislature, at the same time established a procedure applicable to corporations. It is adjudged that the Court of First Instance of the city of Manila be and it is enjoined and prohibited from proceeding further in the criminal case so far as said proceedings relate to the said West Coast Life Insurance Company, a corporation, the plaintiff in the case. ABEJO VS. DE LA CRUZ G.R. No. L-63558 May 19,1987 Facts: Pocket Bell Philippines, Inc. is a tone and voice paging corporation, with petitioner spouses Jose Abejo and Aurora Abejo as its principal stockholders. Telectronic Systems, Inc. purchased their 133,000 minority shareholdings for P5 million and 63,000 shares registered in the name of Virginia Braga which were covered by five stock certificates endorsed in blank by her for P1,674,450.00. The spouses Agapito Braga and Virginia Braga were the majority stockholders, but with the said purchases, Telectronics would become the majority stockholder, holding 56% of the outstanding stock and voting power of Pocket Bell. With the said purchases, Telectronics requested the corporate secretary of the corporation, Norberto Braga, to register and transfer to its name, and those of its nominees the total 196,000 Pocket Bell shares in the corporation's transfer book, cancel the surrendered certificates of stock and issue the corresponding new certificates of stock. However, Braga refused to do so asserting and claiming pre-emptive rights over the 133,000 Abejo shares and that Virginia Braga never transferred her 63,000 shares to Telectronics but had lost the five stock certificates representing those shares. The parties filed independent actions with different jurisdictions.

The Bragas assert that the regular civil court has original and exclusive jurisdiction as against the Securities and Exchange Commission, while the Abejos claim the contrary. Issue: Who, between the CFI (RTC) and the SEC, has original and exclusive jurisdiction over the dispute? Held: The SEC has jurisdiction over the matter as supported by the applicable provisions of P.D. No. 902-A which reorganized the SEC with additional powers including a more active public participation in the affairs of private corporations and enterprises through which desirable activities may be pursued for the promotion of economic development. Nowhere does the law empower any CFI to interfere with the orders of the Commission and consequently any ruling by the trial court on the issue of ownership of the shares of stock is not binding on the Commission for want of jurisdiction. A petition for mandamus in the SEC to compel the corporate secretary to register the transfers and issue new certificates in favor of Telectronics and its nominees was properly resorted to under Rule XXI, Section 1 of the SEC's New Rules of Procedure, which provides for the filing of such petitions with the SEC. Section 3 of said Rules further authorizes the SEC to issue orders expediting the proceedings and to grant a preliminary injunction for the preservation of the rights of the parties pending such proceedings. The dispute falls within the general classification of cases within the SEC's original and exclusive jurisdiction to hear and decide, under Section 5 of the said law. Insofar as the Bragas and their corporate secretary's refusal on behalf of the corporation Pocket Bell to record the transfer of the 56% majority shares to Telectronics may be deemed a device or scheme amounting to fraud and misrepresentation employed by them to keep themselves in control of the corporation to the detriment of

Telectronics (as buyer and substantial investor in the corporate stock) and the Abejos (as substantial stockholders-sellers), the case falls under paragraph (a). The dispute is likewise an intra-corporate controversy between and among the majority and minority stockholders as to the transfer and disposition of the controlling shares of the corporation, failing under paragraph (b). An intra-corporate controversy is one which arises between a stockholder and the corporation. There is no distinction, qualification, nor any exemption whatsoever. The provision is broad and covers all kinds of controversies between stockholders and corporations. The issue of whether or not a corporation is bound to replace a stockholder's lost certificate of stock is a matter purely between a stockholder and the corporation. It is a typical intra-corporate dispute.

Act 4363. Private respondents opposed the motion. Respondent judge issued an order re-affirming the previous order of deferment for the reason that "the rule laid down under Republic Act. No. 4363, amending Article 360 of the Revised Penal Code, is not applicable to actions against nonresident defendants, and because questions involving harassment and inconvenience, as well as disruption of public service do not appear indubitable. Issues: I. Whether or not the jurisdiction of the CFI, a Philippine Court may be challenged by a foreign corporation by writ of certiorari or prohibition; and II. Whether or not Republic Act 4363 is applicable to action against a foreign corporation or non-resident defendant. Held: I. The dismissal of the present petition is asked on the ground that the petitioner foreign corporation failed to allege its capacity to sue in the courts of the Philippines. Respondents rely on section 69 of the Corporation law. They also invoke the ruling in Marshall-Wells Co. vs. Elser & Co., Inc. that no foreign corporation may be permitted to maintain any suit in the local courts unless it shall have the license required by the law, and the ruling in Atlantic Mutual Ins. Co., Inc. vs. Cebu Stevedoring Co., Inc. that "where ... the law denies to a foreign corporation the right to maintain suit unless it has previously complied with a certain requirement, then such compliance or the fact that the suing corporation is exempt therefrom, becomes a necessary averment in the complaint." These doctrines cannot be a propos in the case at bar, since the petitioner is not "maintaining any suit" but is merely defending one against itself; it did not file any complaint but

TIME, INC.ET.AL. VS. VILLEGAS G.R. No. L-28882 May 31, 1971 Facts: Time, Inc. is an American corporation with principal offices at Rocketfeller Center, New York City, N. Y., and is the publisher of "Time", a weekly news magazine. Antonio J. Villegas and Juan Ponce Enrile seek to recover from the herein petitioner damages upon an alleged libel arising from a publication of Time (Asia Edition) magazine, in its issue of 18 August 1967, of an essay, entitled "Corruption in Asia". On motion of the respondents-plaintiffs, the respondent judge, issued a writ of attachment on the real and personal estate of Time, Inc. When petitioner received the summons and a copy of the complaint at its offices in New York, it filed a motion to dismiss the complaint for lack of jurisdiction and improper venue, relying upon the provisions of Republic

only a corollary defensive petition to prohibit the lower court from further proceeding with a suit that it had no jurisdiction to entertain. Petitioner's failure to aver its legal capacity to institute the present petition is not fatal, for a foreign corporation may, by writ of prohibition, seek relief against the wrongful assumption of jurisdiction. And a foreign corporation seeking a writ of prohibition against further maintenance of a suit, on the ground of want of jurisdiction in which jurisdiction is not bound by the ruling of the court in which the suit was brought, on a motion to quash service of summons, that it has jurisdiction. II. That respondents-plaintiffs could not file a criminal case for libel against a non-resident defendant does not make Republic Act No. 4363 incongruous of absurd, for such inability to file a criminal case against a non-resident natural person equally exists in crimes other than libel. It is a fundamental rule of international jurisdiction that no state can by its laws, and no court which is only a creature of the state, can by its judgments or decrees, directly bind or affect property or persons beyond the limits of the state. Not only this, but if the accused is a corporation, no criminal action can lie against it, whether such corporation or resident or non-resident. At any rate, the case filed by respondents-plaintiffs is case for damages.

the DND Undersecretary for Civil Relations and Administration who was tasked by the respondent DND Secretary to conduct an extensive management audit of the records of petitioner. Petitioner claims that it is not a public nor a governmental entity but a private organization, and advances this claim to prove that the issuance of DND Department Circular No. 04 is an invalid exercise of respondent Secretarys control and supervision. Issue: Whether or not petitioner is a private corporation. Held: 1. Petitioner claims that the VFP does not possess the elements which would qualify it as a public office, particularly the possession/delegation of a portion of sovereign power of government to be exercised for the benefit of the public; In Laurel v. Desierto, we adopted the definition of Mechem of a public office, that it is "the right, authority and duty, created and conferred by law, by which, for a given period, either fixed by law or enduring at the pleasure of the creating power, an individual is invested with some portion of the sovereign functions of the government, to be exercised by him for the benefit of the public." In the same case, we went on to adopt Mechems view that the delegation to the individual of some of the sovereign functions of government is "[t]he most important characteristic" in determining whether a position is a public office or not. Such portion of the sovereignty of the country, either legislative, executive or judicial, must attach to the office for the time being, to be exercised for the public benefit. Unless the powers conferred are of this nature, the individual is not a public officer. The most important characteristic which distinguishes an office from an employment or contract is that the creation and conferring of an office involves a delegation to the individual of some of the sovereign functions of government, to be exercised by him for the benefit of the public; that some portion of the sovereignty of the country, either legislative, executive or judicial, attaches, for the time being, to be exercised for the public benefit. Unless the powers conferred are of this nature, the individual is not a public officer. The issue, therefore, is whether the VFAs officers have been delegated some portion of the sovereignty of the country, to be exercised for the public benefit. In several cases, we have dealt with the

VETERAN FOUNDATION OF THE PHILIPPINES VS. REYES GR No. 155027 February 28, 2006 Facts: Petitioner in this case is the Veterans Federation of the Philippines (VFP), a corporate body organized under Republic Act No. 2640, dated 18 June 1960, as amended, and duly registered with the Securities and Exchange Commission. Respondent Angelo T. Reyes was the Secretary of National Defense (DND Secretary) who issued the assailed Department Circular No. 04, dated 10 June 2002. Respondent Edgardo E. Batenga was

issue of whether certain specific activities can be classified as sovereign functions. These cases, which deal with activities not immediately apparent to be sovereign functions, upheld the public sovereign nature of operations needed either to promote social justiceor to stimulate patriotic sentiments and love of country. The growing complexities of modern society, however, have rendered this traditional classification of the functions of government [into constituent and ministrant functions] quite unrealistic, not to say obsolete. The areas which used to be left to private enterprise and initiative and which the government was called upon to enter optionally, and only "because it was better equipped to administer for the public welfare than is any private individual or group of individuals," continue to lose their welldefined boundaries and to be absorbed within activities that the government must undertake in its sovereign capacity if it is to meet the increasing social challenges of the times. Here[,] as almost everywhere else[,] the tendency is undoubtedly towards a greater socialization of economic forces. Here, of course, this development was envisioned, indeed adopted as a national policy, by the Constitution itself in its declaration of principle concerning the promotion of social justice. In the case at bar, the functions of petitioner corporation enshrined in Section 4 of Rep. Act No. 2640 should most certainly fall within the category of sovereign functions. The protection of the interests of war veterans is not only meant to promote social justice, but is also intended to reward patriotism. All of the functions in Section 4 concern the well-being of war veterans, our countrymen who risked their lives and lost their limbs in fighting for and defending our nation. It would be injustice of catastrophic proportions to say that it is beyond sovereigntys power to reward the people who defended her. 2. Petitioner claims that VFP funds are not public funds. The fact that no budgetary appropriations have been released to the VFP does not prove that it is a private corporation. The DBM indeed did not see it fit to propose budgetary appropriations to the VFP, having itself believed that the VFP is a private corporation. If the DBM, however, is mistaken as to its conclusion regarding the nature of VFPs incorporation, its previous assertions will not prevent future budgetary appropriations to the VFP. The erroneous application of the law by public officers does not bar a subsequent correct application of the law. It is important to note here that

the membership dues collected from the individual members of VFPs affiliate organizations do not become public funds while they are still funds of the affiliate organizations. A close reading of Section 1 of Rep. Act No. 2640 reveals that what has been created as a body corporate is not the individual membership of the affiliate organizations, but merely the aggregation of the heads of the affiliate organizations. Thus, only the money remitted by the affiliate organizations to the VFP partake in the public nature of the VFP funds. In Republic v. COCOFED, we held that the Coconut Levy Funds are public funds because, inter alia, (1) they were meant to be for the benefit of the coconut industry, one of the major industries supporting the national economy, and its farmers; and (2) the very laws governing coconut levies recognize their public character. The same is true with regard to the VFP funds. No less public is the use for the VFP funds, as such use is limited to the purposes of the VFP which we have ruled to be sovereign functions. Likewise, the law governing VFP funds (Rep. Act No. 2640) recognizes the public character of the funds as shown in the enumerated provisions above. 3. Petitioner argues that it is a civilian federation where membership is voluntary. Neither is the civilian nature of VFP relevant in this case. The Constitution does not contain any prohibition, express or implied, against the grant of control and/or supervision to the Secretary of National Defense over a civilian organization. The Office of the Secretary of National Defense is itself a civilian office, its occupant being an alter ego of the civilian Commander-in-Chief. This set-up is the manifestation of the constitutional principle that civilian authority is, at all times, supreme over the military. There being no such constitutional prohibition, the creation of a civilian public organization by Rep. Act No. 2640 is not rendered invalid by its being placed under the control and supervision of the Secretary of National Defense. 4. Petitioner claims that the Administrative Code of 1987 does not provide that the VFP is an attached agency, and nor does it provide that it is an entity under the control and supervision of the DND in the context of the provisions of said code.

CORPORATE PERSONALITY AND EXISTENCE The Administrative Code, by giving definitions of the various entities covered by it, acknowledges that its enumeration is not exclusive. The Administrative Code could not be said to have repealed nor enormously modified Rep. Act No. 2640 by implication, as such repeal or enormous modification by implication is not favored in statutory construction. 5. Petitioner offers as evidence the DBM opinion that the VFP is a nongovernment organization in its certification that the VFP "has not been a direct recipient of any funds released by the DBM." Respondents claim that the supposed declaration of the DBM that petitioner is a non-government organization is not persuasive, since DBM is not a quasi-judicial agency. They aver that what we have said of the Bureau of Local Government Finance (BLGF) in Philippine Long Distance Telephone Company (PLDT) v. City of Davao can be applied to DBM: In any case, it is contended, the ruling of the Bureau of Local Government Finance (BLGF) that petitioners exemption from local taxes has been restored is a contemporaneous construction of Section 23 [of R.A. No. 7925 and, as such, is entitled to great weight. The ruling of the BLGF has been considered in this case. But unlike the Court of Tax Appeals, which is a special court created for the purpose of reviewing tax cases, the BLGF was created merely to provide consultative services and technical assistance to local governments and the general public on local taxation and other related matters. Thus, the rule that the "Court will not set aside conclusions rendered by the CTA, which is, by the very nature of its function, dedicated exclusively to the study and consideration of tax problems and has necessarily developed an expertise on the subject, unless there has been an abuse or improvident exercise of authority" cannot apply in the case of the BLGF. On this score, though, we disagree with respondents and hold that the DBMs appraisal is considered persuasive. Respondents misread the PLDT case in asserting that only quasi-judicial agencies determination can be considered persuasive. What the PLDT case points out is that, for an administrative agencys opinion to be persuasive, the administrative agency involved (whether it has quasi-judicial powers or not) must be an expert in the field they are giving their opinion on. ALHAMBRA CIGAR vs. SEC G.R. No. L-23606 July 29, 1968 24 SCRA 269 Facts: Alhambra Cigar was incorporated Jan 15, 1912. Under its articles, it had a corporate life of 50 years and on Jan 15, 1962, its term of existence expired. On that date it ceased transacting business and entered into a state of liquidation. On June 20, 1963, Republic Act 3531 was enacted empowering corporations to extend their life beyond the period fixed by its articles for a term not exceeding 50 years. Prior to the law, Corporations had a non-extendable life of 50 years. On July 15, 1963, the Board of Alhambra resolved to amend the life of the Corporation for another 50 years which was approved by the stockholders on August 1963. The amended articles were then filed with the SEC but the SEC returned the amended articles and said that the law has no retroactive effect. Issue: Whether or not the SEC correctly denied the amended articles of Alhambra Held: Supreme Court affirms the decision of the SEC. Continuance of a dissolved corporation for 3 years has only for its purpose the closure of its affairs and no other. The Corporation is enjoined from doing business for which it was established. Liquidation is necessary because the Corps life has ended. For this reason alone, the Corps life may no longer be extended. An extension, which is in fact an amendment, must be made during the life of the Corp and before the expiration of the term of existence as fixed by

the Articles. Moreover, the filing of the certificate of extension cannot retroact to the date of the passing of the resolution extending the life.

the trial court and appellate court rendered a judgement in favor of defendants and dismissed the complaint. Issue:

LINTONJUA, JR. vs ETERNIT CORPORATION GR. 144805 June 8, 2006 Facts: The Eternit Corporation (EC) manufactures roofing materials and pipe products. Ninety (90%) percent of the shares of stocks of EC were owned by Eteroutremer SA Corporation (ESAC), a corporation registered under the laws of Belgium. Glanville was the General Manager and President of EC, while Delsaux was the Regional Director for Asia of ESAC. In 1986, because of the political situation in the Philippines the management of ESAC wanted to stop its operations and to dispose the land in Mandaluyong City. They engaged the services of realtor/broker Lauro G. Marquez. Marquez thereafter offered the land to Eduardo B. Lintojua, Jr. for PhP 27,000,000.00. Lintonjua counter offered PhP 20,000,000.00 cash. Marquez apprised Glanville and Delsaux of the offer. Delsaux sent a telex stating that, based on nthe Belgian/Swiss decision, the final offer was US$1,000,000.00 and PhP 2,500,000.00. The Lintonjua brothers deposited US$1,000,000.00 with the Security Bank & Trust Company, and drafted an Escrow Agreement to expedite the sale. Meanwhile, with the assumption of Corazon C. Aquino as President, the political situation improved. Marquez received a leet from Delsaux that the ESAC Regional Office decided not to proceed with the sale. When informed of this, the Lintonjuas, filed a complaint for specific performance and payment of damages on account of the aborted sale. Both

Whether or not the appellate court committed a grave error in holding that Marquez needed a written authority from respondent ETERNIT before the sale can be perfected.

Held: Respondents maintain that Glanville, Delsaux and Marquez had no authority from the stockholders of EC and its Board of Directors to offer the properties for sale to the petitioners. Petitioners assert that there was no need for a written authority from the Board of Directors of EC for Marquez to validly act as broker. As broker, Marquez was not an ordinary agent because his only job as a broker was to look for a buyer and to bring together the parties to the transaction. He was not authorized to sell the properties; hence, petitioners argue, Article 1874 of the New Civil Code does not apply. A corporation is a juridical person separate and distinct from its stockholders and is not affected by the personal rights, obligations and transactions of the latter. It may act only through its board of directors or, when authorized by its board resolution, through its officers or agents. The general principles of agency govern the relation between the corporation and its officers or agents, subject to the articles of incorporation, by-laws, or relevant provisions of law. Agency may be oral unless the law requires a specific form. However, to create or convey real rights over immovable property, a special power of attorney is necessary. Thus, when a sale of a piece of land or any portion thereof is through an agent, the authority of the latter shall be in writing, otherwise, the sale shall be void.

In this case, the petitioners failed to adduce in evidence any resolution of the Board of Directors of EC empowering Marquez, Glanville or Delsaux as its agents, to sell, let alone offer for sale, for and its behalf, the eight parcels of land owned by it. Moreover, the evidence of petitioners shows that Adams and Glanville acted on the authority of Delsaux, who, in turn, acted on authority of ESAC, through its Committee for Asia, and the Belgian/Swiss component of the management of ESAC. The offer of Delsaux emanated only form the Belgian/Swiss Decision, and not the entire management of Board of Directors of ESAC. While it is true that petitioners accepted the counter-offer of ESAC was not a party to the transaction between them; hence, EC was not bount by such acceptance.

in its answer as special and affirmative defense lack of cause of action by Sulo ng Bayan Innc. And the barring of such action by prescription and laches. Issue: Whether the corporation (non-stock) may institute an action in behalf of its individual members for the recovery of certain parcels of land allegedly owned by said members, among others.

Held: It is a doctrine well-established and obtains both at law and in equity that a corporation is a distinct legal entity to be considered as separate and apart from the stockholders or members who compose it, and is not affected by the personal rights, obligations and transactions of its stockholders or members. The property of the corporation is its property and not that of the stockholders, as owners, although they have equities in it. Properties registered in the name of the corporation are owned by it as an entity separate and distinct from its members. Conversely, a corporation, even in the case of a one-man corporation. The mere fact that one is president of a corporation does not render the property which he owns or possesses the property of the corporation, since the president, as individual, and the corporation are separate similatities. Similarly, stockholders in a corporation engaged in buying and dealing in real estate whose certificates of stock entitled the holder thereof to an allotment in the distribution of the land of the corporation upon surrender of the stock certificated were considered not to have such legal or equitable title or interest in the land, as would support a suit for title, especially against parties other than the corporation. It must be noted, however, that the juridical personality of the corporation, as separate and distinct from the persons composing it, is but a legal fiction introduced for the purpose of convenience.

SULO NG BAYAN vs. ARANETA GR L-31061 August 17, 1976 Facts: On April 26, 19966, Sulo ng Bayan, Inc. filed an accion de revindicacion with the Court of First Instance of Bulacan, against Gregorio Araneta Inc. (GAI), Paradise Farms Inc., Nationa Waterworks & Sewage Authority (NAWASA), Hacienda Caretas Inc., and the Register of Deeds of Bulacan to recover the ownership and possession of a large tract of land in San Jose del Monte, Bulacan containing an area of 27,982,250 sq.ms., more or less, registered under the Torrens System in the name of GAI, et. Als predecessors-in-interest (who are members of the corporation). On September 2, 1966, GAI filed a motion to dismiss the amended complaint on the grounds that (1) the complaint states no cause of action; and (2) the cause of action, if any, is barred by prescription and laches. Paradise Farms Inc. and Hacienda Caretas Inc. filed motions to dismiss on the same grounds. NAWASA did not file any motion to dismiss. However, it pleaded

This separate personality of the corporation may be disregarded, or the veil of corporate fiction pierced, in cases where it is used as a cloak or cover for fraud or illegality, or to work an injustice, or where necessary to achieve equity. It has not been claimed that the members have assigned or transferred whatever rights they may have on the land in question to the corporation. Absent any showing of interest, therefore, a corporation, has no personality to bring the action for and in behalf of its stockholders or members for the purpose of recovering property which belongs to said stockholders or members in their personal capacities.

would be able to readily enter or leave the property. In a Letter to Roxas, WHI President Dy offered to buy Lot No. 491-A-3-B-2 under stated terms and conditions for P7,213,000. Roxas indicated his acceptance of the offer on page 2 of the deed. Less than a month later Roxas, as President of RECCI, as vendor, and Dy, as President of WHI, as vendee, executed a contract to sell in which RECCI bound and obliged itself to sell to Dy Lot No. 491-A-3-B-2. A Deed of Absolute Sale in favor of WHI was issued, under which Lot No. 491-A-3-B-2 was sold for P5,000,000, receipt of which was acknowledged by Roxas. WHI complained to Roberto Roxas that the vehicles of RECCI were parked on a portion of the property over which WHI had been granted a right of way. Roxas promised to look into the matter. Dy and Roxas discussed the need of the WHI to buy a 500-square-meter portion of Lot No. 491-A-3-B-1 covered by TCT No. 78085 as provided for in the deed of absolute sale. However, Roxas died soon thereafter. WHI wrote the RECCI, reiterating its verbal requests to purchase a portion of the said lot as provided for in the deed of absolute sale, and complained about the latter's failure to eject the squatters within the three-month period agreed upon in the said deed. The WHI demanded that the RECCI sell a portion of Lot No. 491-A-3-B-1 covered by TCT No. 78085 for its beneficial use within 72 hours from notice thereof, otherwise the appropriate action would be filed against it. RECCI rejected the demand of WHI. WHI reiterated its demand in a Letter dated May 29, 1992. There was no response from RECCI. WHI filed a complaint against the RECCI with the Regional Trial Court of Makati, for specific performance and damages. Issue: Whether the respondent is bound by the provisions in the deed of absolute sale granting to the Petitioner beneficial use and a right of way

WOODCHILD HOLDINGS, INC. VS. ROXAS ELECTRIC AND CONSTRUCTION COMPANY, INC. G.R. No. 140667 August 12, 2004 Facts: The respondent Roxas Electric and Construction Company, Inc. (RECCI), formerly the Roxas Electric and Construction Company, was the owner of two parcels of land, Lots A and B. A portion of a lot A (Lot No. 491-A-3-B-2) was a dirt road accessing to the Sumulong Highway, Antipolo, Rizal. The respondent's Board of Directors approved a resolution authorizing the corporation, through its president, Roberto B. Roxas, to sell Lot No. 491-A-3-B-2 at a price and under such terms and conditions which he deemed most reasonable and advantageous to the corporation; and to execute, sign and deliver the pertinent sales documents and receive the proceeds of the sale for and on behalf of the company. Petitioner wanted to buy Lot No. 491-A-3-B-2 to construct its warehouse building, and a portion of the adjoining lot, Lot No. 491-A-3-B-1, so that its 45-foot container van

over a portion of Lot No. 491-A-3-B-1 accessing to the Sumulong Highway and granting the option to the petitioner to buy a portion thereof, and, if so, whether such agreement is enforceable against the respondent

rights over immovable property must appear in a public document. The petitioner cannot feign ignorance of the need for Roxas to have been specifically authorized in writing by the Board of Directors to be able to validly grant a right of way and agree to sell a portion of Lot No. 491-A-3B-1. The rule is that if the act of the agent is one which requires authority in writing, those dealing with him are charged with notice of that fact. Powers of attorney are generally construed strictly and courts will not infer or presume broad powers from deeds which do not sufficiently include property or subject under which the agent is to deal. The general rule is that the power of attorney must be pursued within legal strictures, and the agent can neither go beyond it; nor beside it. The act done must be legally identical with that authorized to be done. In sum, then, the consent of the respondent to the assailed provisions in the deed of absolute sale was not obtained; hence, the assailed provisions are not binding on it. WENSHA SPA CENTER, INC. and/or XU ZHI JIE vs. LORETA T. YUNG Facts:

Held: A corporation is a juridical person separate and distinct from its stockholders or members. Accordingly, the property of the corporation is not the property of its stockholders or members and may not be sold by the stockholders or members without express authorization from the corporation's board of directors. Section 23 of BP 68, otherwise known as the Corporation Code of the Philippines.\ Generally, the acts of the corporate officers within the scope of their authority are binding on the corporation. However, under Article 1910 of the New Civil Code, acts done by such officers beyond the scope of their authority cannot bind the corporation unless it has ratified such acts expressly or tacitly, or is estopped from denying them. Evidently, Roxas was not specifically authorized under the said resolution to grant a right of way in favor of the petitioner on a portion of Lot No. 491-A-3-B-1 or to agree to sell to the petitioner a portion thereof. The authority of Roxas, under the resolution, to sell Lot No. 491-A-3-B2did not include the authority to sell a portion of the adjacent lot, Lot No. 491-A-3-B-1, or to create or convey real rights thereon. Neither may such authority be implied from the authority granted to Roxas to sell Lot No. 491-A-3-B-2 to the petitioner "on such terms and conditions which he deems most reasonable and advantageous." Under paragraph 12, Article 1878 of the New Civil Code, a special power of attorney is required to convey real rights over immovable property. Article 1358 of the New Civil Code requires that contracts which have for their object the creation of real

Loreta stated that she used to be employed by Manmen Services Co., Ltd. (Manmen) where Xu was a client. Xu was apparently impressed by Loretas performance. After he established Wensha, he convinced Loreta to transfer and work at Wensha. . Enticed, Loreta resigned from Manmen and transferred to Wensha. Loreta was later on asked to resign from Wensha because according to the Feng Shui master, her aura did not match that of Xu. Loreta refused but was informed that she could no longer continue working at Wensha. That same afternoon, Loreta went to the NLRC and filed a case for illegal dismissal against Xu and Wensha. The Labor Arbiter dismissed Loretas complaint for lack of merit. He found it more probable that Loreta was dismissed from her employment

due to Wenshas loss of trust and confidence in her. This ruling was affirmed by the NLRC. The CA reversed the decision and orered Wensha Spa Center, Inc. and Xu Zhi Jie to, jointly and severally, pay Loreta T. Yung her full backwages, other privileges, and benefits. Facts: Issue: Whether Xu Zhi Jie should be held jointly and severally liable with Wensha Spa Center. Held: Elementary is the rule that a corporation is invested by law with a personality separate and distinct from those of the persons composing it and from that of any other legal entity to which it may be related. "Mere ownership by a single stockholder or by another corporation of all or nearly all of the capital stock of a corporation is not of itself sufficient ground for disregarding the separate corporate personality." In labor cases, corporate directors and officers may be held solidarily liable with the corporation for the termination of employment only if done with malice or in bad faith. Bad faith does not connote bad judgment or negligence; it imports a dishonest purpose or some moral obliquity and conscious doing of wrong. In the subject decision, the CA concluded that petitioner Xu and Wensha are jointly and severally liable to Loreta. However there is no evidence proving bad faith or malice on the part of Xu. There is, therefore, no justification for such a ruling. To sustain such a finding, there should be an evidence on record that an officer or director acted maliciously or in bad faith in terminating the services of an employee. Moreover, the finding or indication that the dismissal was effected with malice or bad faith should be stated in the decision itself.

TAYAG VS. BENGUET CONSOLIDATED INC. GR NO. L-23145 NOVEMBER 29, 1968

Idonah Slade Perkins, who died on March 27, 1960 in New York City, left among others, two stock certificates covering 33,002 shares of appellant, the certificates being in the possession of the County Trust Company of New York, which as noted, is the domiciliary administrator of the estate of the deceased. Then came this portion of the appellant's brief: "On August 12, 1960, Prospero Sanidad instituted ancillary administration proceedings in the Court of First Instance of Manila; Lazaro A. Marquez was appointed ancillary administrator, and on January 22, 1963, he was substituted by the appellee Renato D. Tayag. A dispute arose between the domiciary administrator in New York and the ancillary administrator in the Philippines as to which of them was entitled to the possession of the stock certificates in question. On January 27, 1964, the Court of First Instance of Manila ordered the domiciliary administrator, County Trust Company, to "produce and deposit" them with the ancillary administrator or with the Clerk of Court. The domiciliary administrator did not comply with the order, and on February 11, 1964, the ancillary administrator petitioned the court to "issue an order declaring the certificate or certificates of stocks covering the 33,002 shares issued in the name of Idonah Slade Perkins by Benguet Consolidated, Inc., be declared [or] considered as lost." The court granted the petion. Issue: Whether or not the respondent can refuse to follow the lawful order of the court on the grounds that the stock certificates cannot be declared or considered as lost and that hat there was a failure to observe certain requirements of its by-laws before new stock certificates could be issued Held:

A corporation is an artificial being created by operation of law....It owes its life to the state, its birth being purely dependent on its will. As Berle so aptly stated: Classically, a corporation was conceived as an artificial person, owing its existence through creation by a sovereign power."As a matter of fact, the statutory language employed owes much to Chief Justice Marshall, who in the Dartmouth College decision defined a corporation precisely as "an artificial being, invisible, intangible, and existing only in contemplation of law." The well-known authority Fletcher could summarize the matter thus: "A corporation is not in fact and in reality a person, but the law treats it as though it were a person by process of fiction, or by regarding it as an artificial person distinct and separate from its individual stockholders.... It owes its existence to law. It is an artificial person created by law for certain specific purposes, the extent of whose existence, powers and liberties is fixed by its charter." Dean Pound's terse summary, a juristic person, resulting from an association of human beings granted legal personality by the state, puts the matter neatly.

NON-ENTITLEMENT TO MORAL DAMAGES

ABS-CBN VS. CA AND VIVA FILMS G.R. No. 128690 January 21, 1999 Facts: In 1990, ABS-CBN and Viva executed a Film Exhibition Agreement whereby ABS-CBN was given the right of first refusal to the next twenty-four (24) Viva films for TV telecast under such terms as may be agreed upon by the parties hereto, provided, however, that such right shall be exercised by ABS-CBN from the actual offer in writing. Consequently, Viva, through defendant Del Rosario, offered ABS-CBN, through its vice-president Charo Santos-Concio, a list of three(3) film packages (36 titles) from which ABS-CBN may exercise its right of first refusal under the afore-said agreement. ABS CBN rejected said list. On February 27, 1992, Del Rosario approached Ms. Concio, with a list consisting of 52 original movie titles, as well as 104 re-runs from which ABS-CBN may choose another 52 titles, or a total of 156 titles, proposing to sell to ABS-CBN airing rights over this package of 52 originals and 52 re-runs for P60,000,000.00. The package was rejected by ABS-CBN. On April 06, 1992, Del Rosario and Mr. Graciano Gozon of RBS discussed the terms and conditions of Vivas offer to sell the 104 films. On April 07, 1992, defendant Del Rosario received through his secretary, a handwritten note from Ms. Concio which reads: Heres the draft of the contract. I hope you find everything in order, to which was attached a draft exhibition agreement, a counter-proposal covering 53 films for a consideration of P35 million. The said counter-proposal was however rejected by Vivas Board of Directors. On April 29, 1992, Viva granted RBS the exclusive right to air 104 Viva-produced and/or acquired films including the fourteen (14) films

As a matter of fact, a corporation once it comes into being, following American law still of persuasive authority in our jurisdiction, comes more often within the ken of the judiciary than the other two coordinate branches. It institutes the appropriate court action to enforce its right. Correlatively, it is not immune from judicial control in those instances, where a duty under the law as ascertained in an appropriate legal proceeding is cast upon it. To assert that it can choose which court order to follow and which to disregard is to confer upon it not autonomy which may be conceded but license which cannot be tolerated. It is to argue that it may, when so minded, overrule the state, the source of its very existence; it is to contend that what any of its governmental organs may lawfully require could be ignored at will. So extravagant a claim cannot possibly merit approval.

subject of the present case. ABS-CBN then filed a a complaint for specific performance. RTC rendered a decision in favor of RBS and VIVA and against ABS-CBN, ruling that there was no meeting of minds on the price and terms of the offer. Furthermore, the right of first refusal under the 1990 Film Exhibition Agreement had previously been exercised per Ms. Concios letter to Del Rosario ticking off ten titles acceptable to them, which would have made the 1992 agreement an entirely new contract. The Court of Appeals affirmed the decision of the RTC. Hence, this petition.

ISSUES: 1. WHETHER THE CONTRACT BETWEEN LOPEZ AND DEL ROSARIO WAS PERFECTED 2. WHETHER THE RESPONDENT RBS IS ENTITLED TO MORAL DAMAGES HELD: 1. Contracts that are consensual in nature are perfected upon mere meeting of the minds, once there is concurrence between the offer and the acceptance upon the subject matter, consideration, and terms of payment a contract is produced. The offer must be certain. To convert the offer into a contract, the acceptance must be absolute and must not qualify the terms of the offer; it must be plain, unequivocal, unconditional, and without variance of any sort from the proposal. A qualified acceptance, or one that involves a new proposal, constitutes a counter-offer and is a rejection of the original offer. Consequently, when something is desired which is not exactly what is proposed in the offer, such acceptance is not sufficient to generate consent because any modification or variation from the terms of the offer annuls the offer. Under Corporation Code, unless otherwise provided by said Code, corporate powers, such as the power; to enter into contracts; are

exercised by the Board of Directors. However, the Board may delegate such powers to either an executive committee or officials or contracted managers. The delegation, except for the executive committee, must be for specific purposes, Delegation to officers makes the latter agents of the corporation; accordingly, the general rules of agency as to the bindings effects of their acts would apply. For such officers to be deemed fully clothed by the corporation to exercise a power of the Board, the latter must specially authorize them to do so. That Del Rosario did not have the authority to accept ABS-CBN's counter-offer was best evidenced by his submission of the draft contract to VIVA's Board of Directors for the latter's approval. In any event, there was between Del Rosario and Lopez III no meeting of minds. 2. Moral damages are in the category of an award designed to compensate the claimant for actual injury suffered. And not to impose a penalty on the wrongdoer. The award is not meant to enrich the complainant at the expense of the defendant, but to enable the injured party to obtain means, diversion, or amusements that will serve to obviate then moral suffering he has undergone. It is aimed at the restoration, within the limits of the possible, of the spiritual status quo ante, and should be proportionate to the suffering inflicted. Trial courts must then guard against the award of exorbitant damages; they should exercise balanced restrained and measured objectivity to avoid suspicion that it was due to passion, prejudice, or corruption on the part of the trial court. The award of moral damages cannot be granted in favor of a corporation because, being an artificial person and having existence only in legal contemplation, it has no feelings, no emotions, no senses, It cannot, therefore, experience physical suffering and mental anguish, which call be experienced only by one having a nervous system. The statement in People

v. Manero and Mambulao Lumber Co. v. PNB that a corporation may recover moral damages if it "has a good reputation that is debased, resulting in social humiliation" is an obiter dictum. On this score alone the award for damages must be set aside, since RBS is a corporation.

ASSET PRIVATIZATION TRUST VS. COURT OF APPEALS Facts: The Philippine Government undertook to support the financing of Marinduque Mining and Industrial Corporation (MMIC). the Philippine Government obtained a firm commitment from the DBP and/or other government financing institutions. DBP approved guarantees in favor of MMIC and subsequent requests for guarantees were based on the unutilized portion of the Government commitment. Thereafter, the Government extended accommodations to MMIC in various amounts. MMIC, PNB and DBP executed a Mortgage Trust Agreement whereby MMIC, as mortgagor, agreed to constitute a mortgage in favor or PNB and DBP as mortgagees, over all MMIC's assets. MMIC was having a difficult time meeting its financial obligations. MMIC had an outstanding loan with DBP in the amount of P13.7 billion and with PNB in the amount of P8.7 billion. Thus, a financial restructuring plan (FRP was drafted by the Sycip Gorres Velayo accounting firm. The various loans and advances made by DBP and PNB to MMIC had become overdue. DBP and PNB as mortgagees of MMIC assets, decided to exercise their right to extrajudicially foreclose the mortgages in accordance with the Mortgage Trust Agreement.

Jesus S. Cabarrus, Sr., together with the other stockholders of MMIC, filed a derivative suit against DBP and PNB before the RTC of Makati, Branch 62, for Annulment of Foreclosures, Specific Performance and Damages. APT, as successor of the DBP and the PNB's interest in MMIC, mutually agreed to submit the case to arbitration. The issues to be submitted for the Committee's resolution shall be (a) Whether plaintiffs have the capacity or the personality to institute this derivative suit in behalf of the MMIC or its directors, (b) Whether or not the actions leading to, and including,. the PNB-DBP foreclosure of the MMIC assets were proper, valid and in good. The Committee finds, there is no foreclosure at all as it was not legally and validly done. APT was ordered to pay Marinduque Mining and Industrial Corporation the sum of P13,000.000.00, as and for moral and exemplary damages. On appeal, the Court of Appeals denied due course and dismissed the petition for certiorari. Issues: Whether MMIC was entitled to moral damages. Held: The MMIC cannot be entitled to a big amount of moral damages when its credit reputation was not exactly something to be considered sound and wholesome. Under Article 2217 of the Civil Code, moral damages include besmirched reputation which a corporation may possibly suffer. A corporation whose overdue and unpaid debts to the Government alone reached a tremendous amount of P22 Billion Pesos cannot certainly have a solid business reputation to brag about. It must be pointed out that when the supposed wrongful act of foreclosure was done, MMIC's credit reputation was no longer a desirable

one. The company then was already suffering from serious financial crisis which definitely projects an image not compatible with good and wholesome reputation. So it could not be said that there was a "reputation" besmirched by the act of foreclosure.

DOCTRINE OF PIERCING THE VEIL OF CORPORATE ENTITY HI-CEMENT CORPORATION VS. INSULAR BANK OF ASIA AND AMERICA G.R. No. 132403 September 28, 2007 Facts: Petitioners Enrique Tan and Lilia Tan were the controlling stockholders of E.T. Henry & Co., Inc., a company engaged in the business of processing and distributing bunker fuel. Among E.T. Henry's customers were petitioner Hi-Cement Corporation, Riverside Mills Corporation and Kanebo Cosmetics Philippines, Inc. For their purchases, these corporations issued postdated checks to E.T. Henry. Respondent Insular Bank of Asia and America (later PCIB and now Equitable PCI-Bank) granted E.T. Henry a credit facility known as Purchase of Short Term Receivables. Through this arrangement, E.T. Henry was able to encash, with pre-deducted interest, the postdated checks of its clients (re-discounting of checks). For every transaction, respondent required E.T. Henry to execute a promissory note and a deed of assignment bearing the conformity of the client to the rediscounting. E.T. Henry was able to re-discount its clients' checks (with deeds of assignment) with respondent. However, when 20 checks of Hi-Cement (which were crossed and which bore the restriction deposit to payees account only), Riverside, and Kanebo were dishonoured, respondent filed

a complaint for sum of money in the then Court of First Instance of Rizal against E.T. Henry, the spouses Tan, Hi-Cement (including its general manager and its treasurer as signatories of the postdated crossed checks), Riverside and Kanebo. Respondent sought for the payment of actual damages and for the collection from E.T. Henry and the spouses Tan other loan obligations (amounting to P1,661,266.51 and P4,900,805, respectively) as deficiencies resulting from the foreclosure of the real estate mortgage on E.T. Henry's property in Sucat, Paraaque. Hi-Cement filed its answer alleging, among others, that: (1) its general manager and treasurer were not authorized to issue the postdated crossed checks in E.T. Henry's favor; (2) the deed of assignment purportedly executed by Hi-Cement assigning them to respondent only bore the conformity of its treasurer and (3) respondent was not a holder in due course as it should not have discounted them for being crossed checks. In their answer (with counterclaim against respondent and cross-claims against Hi-Cement, Riverside and Kanebo), E.T. Henry and the spouses Tan claimed that: (1) the drawers of the postdated checks failed to honor them due to the adverse economic conditions prevailing at the time respondent presented them for payment; (2) the extra-judicial sale of the mortgaged Sucat property was void due to gross inadequacy of the bid price and (3) their loans were subjected to a usurious interest rate of 21% p.a. For their part, Riverside and Kanebo sought the dismissal of the case against them, arguing that they were not privy to the re-discounting arrangement between respondent and E.T. Henry. The trial court rendered judgment in favor of respondent and against E.T. Henry, spouses Tan, Hi-Cement, Riverside and Kanebo. Only petitioners appealed the decision to the CA which affirmed it in toto. Issues: (1) Whether or not petitioners may be held liable for the dishonoured postdated cross checks.

(2) Whether or not the doctrine of piercing the veil of corporate entity is applicable herein. Held: (1) Both the general manager and treasurer of Hi-Cement were authorized to issue the subjects checks. Hi-Cement was already estopped from denying such authority since it never objected to the signatories' issuance of all previous checks to E.T. Henry which the latter, in turn, was able to re-discount with respondent. However, notwithstanding such fact, respondent could not be considered a holder in due course. Absent any of the elements set forth in Section 52, the holder is not a holder in due course. In the case at bar, the last two requirements were not met. The respondent's claim that it acted in good faith when it accepted and discounted Hi-Cements postdated crossed checks from E.T. Henry (as payee therein) is unconvincing. Good faith becomes inconsequential amidst proof of respondent's grossly negligent conduct in dealing with the subject checks. Respondent was all too aware that subject checks were crossed and bore restrictions that they were for deposit to payee's account only; hence, they could not be further negotiated to it. The records likewise reveal that respondent completely disregarded a telling sign of irregularity in the rediscounting of the checks when the general manager did not acquiesce to it as only the treasurer's signature appeared on the deed of assignment. As a banking institution, it behooved respondent to act with extraordinary diligence inevery transaction. Its business is impressed with public interest, thus, it was not expected to be careless and negligent, specially so where the checks it dealt with were crossed. Here, there was no doubt that it was E.T. Henry that re-discounted Hi-Cement's checks and received their value from respondent. Since E.T. Henry had no justification to refuse payment, it should pay respondent. HiCement could not also be made solidarily liable with Riverside and Kanebo for the face value of their checks. Hi-Cement had nothing to do with the

checks of these two corporations. However, although the language of the trial court decision's dispositive portion seemed confusing, a reading of the decision in its entirety reveals that the fallo was for each corporation to be liable solidarily with E.T. Henry and/or the spouses Tan for the respective values of their checks. At any rate, the issue has become moot in view of our ruling that Hi-Cement is not liable for the checks. (2) The general rule is that the corporation will be looked upon as a legal entity until sufficient reasons to the contrary appear. It is only when the fiction or notion of legal entity is used to defeat public convenience, justify wrong, perpetuate fraud or defend crime that the law will shred the corporate legal veil and regard it as a mere association of persons. This is referred to as the doctrine of piercing the veil of corporate entity. However, said doctrine may not be applied herein. First, the trial court failed to provide a clear ground why the doctrine was used. It merely stated that it agreed with respondents arguments but did not explain why the doctrine was relevant to petitioner E.T. Henry's and the spouses Tans case. Similarly, the CA left a gaping hole by failing to provide the basis for its ruling that E.T. Henry and the spouses Tan defrauded respondent. It did not also state what act constituted the fraud. Fraud is an allegation of fact that demands clear and convincing evidence. It is never presumed. Second, the mere ownership by a single stockholder or by another corporation of all or nearly all of the capital stock of a corporation is not of itself sufficient ground for disregarding the separate corporate personality. For this ground to stand in this case, there must be proof that the spouses Tan: (1) had control or complete domination of E.T. Henrys finances and that the latter had no separate existence with respect to the act complained of; (2) used such control to commit fraud or wrong and (3) the control was

the proximate cause of the loss or injury complained of by respondent. The records of this case do not show that these elements were present. Third, mere inadequacy of the price obtained at the sheriffs sale, unless shocking to the conscience, was not sufficient to set aside the sale if there was no showing that, in the event of a regular sale, a better price could be obtained. Furthermore, in the absence of any irregularity in the foreclosure proceeding or proof that it was carried out without strict observance of the procedure, an assumption as to its regularity shall stay.

stockholders of the PADCO are also the same controlling stockholders of the Graphic and, therefore, the levy upon the said machinery which was found in the premises occupied by the Graphic should be upheld. In addition, petitioner contends that respondent judge gravely exceeded, if not, acted without jurisdiction, in nullifying the sheriffs sale not only because Section 17, Rule 39 of the Rules of Court was not complied with, but more importantly because PADCO could not have litigated its claim in the same case, but in an independent civil proceeding. Issue: Held: Whether or not there is a need to pierce the corporate veil.

TAN BOON BEE VS. JARENCIO 163 SCRA 205 Facts: Petitioner, doing business under the name and style of Anchor Supply Co., sold on credit to herein private respondent Graphic Publishing, Inc. paper products. For failure of Graphic to pay any installment, as agreed on the contract of sale, petitioner filed with the CFI of Manila a collection suit for a sum of money to which the latter rendered judgment ordering Graphic to pay the petitioner. On motion of petitioner, a writ of execution was issued and the executing sheriff levied upon one unit printing machine identified as "Original Heidelberg Cylinder Press" Type H 222, NR 78048, found in the premises of Graphic. However, private respondent, Philippine American Drug Company (PADCO), later informed the sheriff that the printing machine is its property and not that of Graphic. Nevertheless, the sheriff proceeded with the scheduled auction sale, and sold the property to the petitioner. PADCO filed an "Affidavit of Third Party Claim" with the Office of the City Sheriff and thereafter a Motion to Nullify Sale on Execution (With Injunction) with the CFI. Despite petitioners opposition, the CFI ruled in favor of PADCO. Petitioner contends that the controlling

The doctrine that a corporation is a legal entity distinct and separate from the members and stockholders who compose it is recognized and respected in all cases which are within reason and the law. However, this separate and distinct personality is merely a fiction created by law for convenience and to promote justice. Accordingly, this separate personality of the corporation may be disregarded, or the veil of corporate fiction pierced, in cases where it is used as a cloak or cover for fraud or illegality, or to work an injustice, or where necessary to achieve equity or when necessary for the protection of creditors. Corporations are composed of natural persons and the legal fiction of a separate corporate personality is not a shield for the commission of injustice and inequity. Likewise, this is true when the corporation is merely an adjunct, business conduit or alter ego of another corporation. In such case, the fiction of separate and distinct corporation entities should be disregarded. In the instant case, petitioners evidence established that PADCO was never engaged in the printing business; that the board of directors and the officers of GRAPHIC and PADCO were the same; and that PADCO holds 50% share of stock of GRAPHIC. Petitioner likewise stressed that PADCOs own evidence shows that the printing machine in question had

been in the premises of GRAPHIC since May, 1965, long before PADCO even acquired its alleged title on July 11, 1966 from Capitol Publishing. That the said machine was allegedly leased by PADCO to GRAPHIC on January 24, 1966, even before PADCO purchased it from Capital Publishing on July 11, 1966, only serves to show that PADCOs claim of ownership over the printing machine is not only farce and sham but also unbelievable.

The UNION in its Motion for Reconsideration, prays that the veil of corporate fiction be pierced and that all the individual private respondents and not only the President, should be held jointly and severally liable with RANSOM. Issue: Whether the officers together with Ransom should be held liable. Held: When the notion of legal entity is used as a means to perpetrate fraud or an illegal act or as a vehicle for the evasion of an existing obligation, the circumvention of statutes, and or confuse legitimate issues the veil which protects the corporation will be lifted. The alleged bankruptcy of RANSOM furnishes no justification for non-payment of back wages to the employees concerned taking into consideration Article 110 of the Labor Code, which gives preference to the claim of employees for back wages. Aggravating RANSOM's clear evasion of payment of its financial obligations is the organization of a run-away corporation, ROSARIO at the time the unfair labor practice case was pending before the CIR by the same persons who were the officers and stockholders of RANSOM, engaged in the same line of business as RANSOM, producing the same line of products, occupying the same compound, using the same machineries, buildings, laboratory, bodega and sales and accounts departments used by RANSOM, and which is still in existence. Both corporations were closed corporations owned and managed by members of the same family. Its organization proved to be a convenient instrument to avoid payment of backwages and the reinstatement of the 22 workers. This is another instance where the fiction of separate and distinct corporate entities should be disregarded.

A.C. RANSOM LABOR UNION-CCLU vs. NATIONAL LABOR RELATIONS COMMISSION, First Division A.C. RANSOM (PHIIS.) CORPORATION RUBEN HERNANDEZ, MAXIMO C. HERNANDEZ, SR., PORFIRIO R. VALENCIA, LAURA H. CORNEJO, FRANCISCO HERNANDEZ, CELESTINO C. HERNANDEZ and MA. ROSARIO HERNANDEZ.

Facts: Motions were filed by petitioner for the collection of back wages of the 22 employees of the corporation which RANSOM opposed stressing due to its financial condition. The officers of Ransom later on organized another corporation named Rasario Industrial Corporation which is substantially the same as Ransom but the officers declared that ROSARIO is a distinct and separate corporation, which was organized long before these instant cases were decided by the CIR adversely against RANSOM. The Labor arbiter held Ransom and its officers liable for the amount being demanded, but the NLRC modified the decision limiting the liability to the president of Ransom, Ruben Hernandez together with the other presidents of the same corporation who was elected subsequently up to the termination of the life of the corporation.

PNB vs. ANDRADA ELECTRIC AND ENGINEERING COMPANY GR No. 142936

Facts: On 26 August 1975, the Philippine national Bank (PNB) acquired the assets of the Pampanga Sugar Mills (PASUMIL)that were earlier foreclosed by the Development Bank of the Philippines (DBP) under LOI311. The PNB organized the National Sugar Development Corporation (NASUDECO) in September 1975, to take ownership and possession of the assets and ultimately to nationalize and consolidate its interest in other PNB controlled sugar mills. Prior to 29 October 1971, PASUMIL engaged the services of the Andrada Electric & Engineering Company (AEEC) for electrical rewinding and repair, most of which were partially paid by PASUMIL, leaving several unpaid accounts with AEEC. On 29 October 1971, AEEC and PASUMIL entered into a contract for AEEC to perform the (a) Construction of a power house building;3 reinforced concrete foundation for 3 units 350 KW diesel engine generating sets, 3 reinforced concrete foundation for the5,000 KW and 1,250 KW turbo generator sets, among others. Aside from the work contract, PASUMIL required AEEC to perform extra work, and provide electrical equipment and spare parts. Out of the total obligation of P777,263.80, PASUMIL had paid only P250,000.00, leaving an unpaid balance, as of 27 June1973, amounting toP527,263.80. Out of said unpaid balance of P527,263.80, PASUMIL made a partial payment to AEEC of P14,000.00, in broken amounts, covering the period from 5 January 1974 up to 23 May1974, leaving an unpaid balance of P513,263.80. PASUMIL and PNB, and now NASUDECO, allegedly failed and refused to pay AEEC their just, valid and demandable obligation (The President of the NASUDECO is also the Vice-President of the PNB. AEEC besought said official to pay the outstanding obligation of PASUMIL, inasmuch as PNB and NASUDECO now owned and possessed the assets of PASUMIL, and

these defendants all benefited from the works, and the electrical, as well as the engineering and repairs, performed by AEEC). Because of the failure and refusal of PNB, PASUMIL and/or NASUDECO to pay their obligations, AEEC allegedly suffered actual damages in the total amount of P513,263.80; and that in order to recover these sums, AEEC was compelled to engage the professional services of counsel, to whom AEEC agreed to pay a sum equivalent to 25% of the amount of the obligation due by way of attorney's fees.PNB and NASUDECO filed a joint motion to dismiss on the ground that the complaint failed to state sufficient allegations to establish a cause of action against PNB and NASUDECO, inasmuch as there is lack or want of privity of contract between the them and AEEC. Said motion was denied by the trial court in its 27November order, and ordered PNB and NASUDECO to file their answers within 15 days. After due proceedings, the Trial Court rendered judgment in favor of AEEC and against PNB, NASUDECO and PASUMIL; the latter being ordered to pay jointly and severally the former (1) the sum of P513,623.80 plus interest thereon at the rate of 14% per annum as claimed from 25 September 1980 until fully paid; (2) the sum of P102,724.76 as attorney's fees; and, (3) Costs. PNB and NASUDECO appealed. The Court of Appeals affirmed the decision of the trial court in its decision of 17 April 2000 (CA-GR CV 57610. PNB and NASUDECO filed the petition for review ISSUE: Whether PNB and NASUDECO for PASUMILs liability to AEEC. HELD: Basic is the rule that a corporation has a legal personality distinct and separate from the persons and entities owning it. The corporate veil may be lifted only if it has been used to shield fraud, defend crime, justify a wrong, defeat public convenience, insulate bad faith or perpetuate injustice. Thus, the mere fact that the Philippine National Bank (PNB)acquired ownership or management of some assets of the Pampanga Sugar may be held liable

Mill (PASUMIL), which had earlier been foreclosed and purchased at the resulting public auction by the Development Bank of the Philippines (DBP), will not make PNB liable for the PASUMIL's contractual debts to Andrada Electric & Engineering Company (AEEC). Piercing the veil of corporate fiction may be allowed only if the following elements concur: (1) control not mere stock control, but complete domination not only of finances, but of policy and business practice in respect to the transaction attacked, must have been such that the corporate entity as to this transaction had at the time no separate mind, will or existence of its own; (2) such control must have been used by the defendant to commit a fraud or a wrong to perpetuate the violation of a statutory or other positive legal duty, or a dishonest and an unjust act in contravention of plaintiff's legal right; and (3) the said control and breach of duty must have proximately caused the injury or unjust loss complained of. The absence of the foregoing elements in the present case precludes the piercing of the corporate veil. First, other than the fact that PNB and NASUDECO acquired the assets of PASUMIL, there is no showing that their control over it warrants the disregard of corporate personalities. Second, there is no evidence that their juridical personality was used to commit a fraud or to do a wrong; or that the separate corporate entity was farcically used as a mere alter ego, business conduit or instrumentality of another entityor person. Third, AEEC was not defrauded or injured when PNB and NASUDECO acquired the assets of PASUMIL. Hence, although the assets of NASUDECO can be easily traced to PASUMIL, the transfer of the 15latter's assets to PNB and NASUDECO was not fraudulently entered into in order to escape liability for its debt to AEEC. Neither was there any merger or consolidation with respect to PASUMIL and PNB. The procedure prescribed under Title IX of the Corporation Code 59 was not followed. In fact, PASUMIL's corporate existence had not been legally extinguished or terminated. Further, prior to PNB's acquisition of the foreclosed assets, PASUMIL had previously made partial payments to AEEC for the former's obligation in the amount of P777,263.80. As of 27 June1973, PASUMIL had paid P250,000 to AEEC

and, from 5 January1974 to 23 May 1974, another P14,000. Neither did PNB expressly or impliedly agree to assume the debt of PASUMIL to AEEC. LOI 11 explicitly provides that PNB shall study and submit recommendations on the claims of PASUMIL's creditors. Clearly, the corporate separateness between PASUMIL and PNB remains despite AEEC's insistence to the contrary. SITUS / NATIONALITY OF CORPORATION SEC En Banc Case No. 09-09-177 For: Review of CED Ruling Redmont Consolidated Mines Corporation vs. McArthur Mining Inc., Tesoro Mining and Development, Inc., Narra Nickel Mining and Development Inc., Sara Marie Mining Inc., Patricia Louise Mining and Development Corporation, Madridejos Mining Corporation, Bethlehem Nickel Corporation, San Juanico Nickel Corpoatration and MBMI Resources, Inc. Facts: Redmont is a domestic corporation engaged in the mining business with a SEC Registration No. Respondent corporations are likewise corporations incorporated in the Philippines to engage in mining with their respective SEC Registration Nos. MBMI is a Canadian mining company focused on the exploration and development of nickel mineral properties in the Philippines. Redmont filed a Complaint for Revocation before the CED, praying that the certificates for registration of respondent corporations be revoked on the ground that they violated the constitutional and statutory restriction on the foreign ownership of corporations engaged in the exploitation, development and utilization of natural resources. According to Redmont:: (1) MBMI

actually funded 99.78864%, 99.78571% and 99.80039% of the paid up capital of Madridejos, Patricia Louise, Sara Marie and an Juanico, respectively; (2) MBMI actually funded 69.35204%, 69.35204%, and 69.41678% of the paid up capital of McArthur, Tesoro and Bethlehem, respectively; in both cases, way above the maximum cap for foreign ownership of 40% as shown by the corporations respective Articles of Incorporation. Further, respondent corporations employed fraudulent schemes of cascading or company layering, hiding its actual direct ownership of stocks through a first layer corporation. Redmont arrived at such figures using the Grandfather Rule, which it argued must be observed in determining the nationality of a corporation instead of the Control Test. The Compliance and Enforcement Department (CED) of SEC issued a Letter-Resolution disposing of the Redmonts Complaint for revocation of certificates of registration against respondents, finding McArthur, Tesoro, Sara Marie, Madridejos, and Bethlehem to have not yet started their commercial operations, while Narra and Patricia Louise were directed to cease from small-scale mining activity unless they amend their Articles of Incorporation and pay the penalties to SEC pursuant to the violations recorded. It also held that respondent corporations were in fact Philippine nationals, as defined under Section 3 of the Foreign Investments Act of 1991 (R.A. 7042 as amended by R.A. 8179), except for MBMI which is registered in Canada. Issues: (1) As to McArthur, Tesoro and Narra: Whether or not appelleecorporations are Philippine nationals qualified to engage in mining. (2) As to Madridejos, Patricia Louise, Sara Marie, San Juanico and Bethlehem: Which between the Grandfather Rule and the Control Test should be used in the case at bar to determine the said corporations nationalities?

Held: (1) In deference to the CA (where Redmont currently has an appeal pending therein regarding the same matter, originating from the POA and MAB of the DENR which both declared the corporations to be Filipino nationals) the ruling of the CED that McArthur, Tesoro and Narra are Philippine nationals, are set aside and thus the complaint for revocation against said corporations is dismissed. (2) Under DOJ Opinion No. 020, Series of 2005, which adopted the 1967 SEC Rules, the requirement of the Constitution and other laws that the controlling interests in enterprises engaged in the exploitation of natural resources shall be owned by Filipino citizens is implemented. It is provided therein that there are two cases in determining the nationality of the Investee Corporation. The first case is the liberal rule, later coined by the SEC as the Control Test, and pertains to the shares belonging to corporations or partnerships at least 60% of the capital of which is owned by Filipino citizens shall be considered as of Philippine nationality. Under the liberal Control Test, there is no need to further trace the ownership of the 60% or more Filipino stockholdings of the Investing Corporation since a corporation which is at least 60% Filipino-owned is considered as Filipino. The second case is the Strict Rule or the Grandfather Rule Proper and pertains to the portion referred to in the SEC Rules as the percentage of Filipino ownership in the corporation or partnership which is less than 60%, only the number of shares corresponding to such percentage shall be counted as of Philippine nationality. Thus, the combined totals in the Investing Corporation and the Investee Corporation must be traced to determine the total percentage of Filipino ownership. Thus, based on the said SEC Rule and DOJ Opinion, applies only when the 60-40 Filipinoforeign equity ownership is in doubt. Such doubt exists in this case because the foreign investor, MBMI, provided practically all the funds of the remaining appellee-corporations.

The legal fiction under the Control Test that the 60% investment o the investing corporation is deemed Filipino actually favors foreigners because their indirect interest in the investee corporation through their ownership of 40% in the investing corporation is unduly ignored. On the other hand, if we apply the Grandfather Rule, the foreigners would be barred from participating in such nationalized area of investment, their actual and true participation having been established. Hence, the Grandfather Rule must be applied to accurately determine the actual participation, both direct and indirect, of foreigners in a corporation engaged in a nationalized activity or business. Lastly, it was the intent of the framers of the 1987 Constitution to adopt the Grandfather Rule as may be gleaned from in the discussions on Article XII. The case is thus remanded to the CED for further proceedings and the CRMD is directed to assist the CED. The CED Letter-Resolution is thus set aside.

24% foreign + 40% (direct foreign investment in Corp. C) = 64% total foreign investment

CERTIFICATES OF STOCK SAN MIGUEL CORPORATION VS. SANDIGANBAYAN G.R. Nos. 104637-38 September 14, 2000 Facts: San Miguel Corporation involved the sale by the 14 CIIF Companies, through the United Coconut Planters Bank (UCPB), of 33,133,266 SMC shares, to the SMC. Before the perfection of the sale, however, the said shares were sequestered. Thus, the SMC group suspended payment of the purchase price of the shares, while the UCPB group rescinded the sale. Later, the SMC and UCPB groups entered into a Compromise Agreement and Amicable Settlement, whereby they undertook to continue with the sale of the subject shares of stock. They likewise agreed to pay an "arbitration fee" of 5,500,000 SMC shares composed of 3,858,831 A shares and 1,641,169 B shares to the PCGG to be held in trust for the Comprehensive Agrarian Reform Program. The Republic and COCOFED opposed the said agreement contending that the involved coco-levy funds, whether in the form of earnings or dividends therefrom, or in the form of the value of liquidated corporate assets represented by all sequestered shares (like the value of assets sold/mortgaged to finance the P500M first installment), or in the form of cash, or, as in the case of subject "Settlement," in the form of "proceeds" of sale or of "payments" of certain alleged obligations are public funds. As public funds, the coco-levy funds, in any form or transformation, are beyond or "outside the commerce," and perforce not within the private disposition of private individuals.

*Note: CONTROL TEST: 60% (Filipino equity in Corp. A) x 60% (Corp. As equity in Corp. C) = 36% Filipino
__________________________________________

100

40% (Filipino equity in Corp. A) x 60% (Corp. As equity in Corp. C) = 24% Filipino
__________________________________________

100

Nevertheless, the parties moved for the approval of this agreement by the Sandiganbayan where the case was then pending. Later, UCPB and the SMC groups implemented their agreement extra-judicially, withdrawing, at the same time, their petition for the approval of their aforementioned compromise agreement. Thereafter, the Sandiganbayan issued an Order dated August 5, 1991, directing the SMC to deliver to the graft court the sequestered SMC shares that it bought from UCPB. This was followed by another Order dated March 18, 1992, for the delivery to the court of dividends pertaining to the subject SMC shares. It was these two delivery Orders that were submitted for the consideration of the SC. SMC contends that the questioned orders would deprive SMC of property already paid for. They unduly protect the claimants of sequestered companies, at the expense of SMC. The fact of sequestration, by itself, does not mean that the possessor of the sequestered assets must be dispossessed thereof at all costs. In the present case, there are weighty reasons why the treasury shares and any "dividends" thereon should remain with SMC. The purported issue of ownership does not justify the dispossession of SMC of these shares. According to private respondent COCOFED, the transformation of the SMC shares into treasury shares is but part and parcel of the compromise agreement which has not yet been approved. Thus, it is premature for the SMC Group to treat these shares as such and to refuse to turn over the same as well as the accrued dividends thereon to the PCGG, as ordered by the Sandiganbayan. Moreover, the transformation is extremely disadvantageous to the CIIF Companies. Further, the CIIF Companies, being the disputed owners of the SMC shares, are entitled to have the dividends on the SMC shares applied to its indebtedness to UCPB. On the other hand, until the question of which entity is entitled thereto is settled,

the SMC shares corresponding to the P500 million first installment and the dividends thereon should be turned over to the PCGG. Issue: Whether or not the certificates of stock of SMC shares and the dividends thereon should be delivered to the PCGG. Held: First. The cases at bar do not merely involve a compromise agreement dealing with private interest. The Compromise Agreement here involves sequestered shares of stock now worth more than nine (9) billions of pesos, per estimate given by COCOFED. Their ownership is still under litigation. It is not yet known whether the shares are part of the alleged illgotten wealth of former President Marcos and his "cronies." Any Compromise Agreement concerning these sequestered shares falls within the unquestionable jurisdiction of and has to be approved by the Sandiganbayan. The parties themselves recognized this jurisdiction. In the Compromise Agreement itself, the petitioners and the UCPB Group expressly acknowledged the need to obtain the approval by the Sandiganbayan of its terms and conditions. The PCGG Resolution of June 15, 1990 also imposed the approval of the Sandiganbayan as a condition sine qua non for the transfer of these sequestered shares of stock. Thus, the petitioners voluntarily submitted to the jurisdiction of the Sandiganbayan by asking for the approval of the said Compromise Agreement. Second. Given its undisputed jurisdiction, the Sandiganbayan ordered that the treasury shares should be delivered to PCGG and that their dividends should be paid pending determination of their real ownership which is the key to the question whether they are part of the alleged ill-gotten wealth of former President Marcos and his "cronies." In the exercise of its discretion, the Sandiganbayan can require a party-litigant to deliver a sequestered property to the PCGG. The order of the

Sandiganbayan regarding the subject treasury shares is merely preservative in nature - for the purpose of preventing the destruction, concealment or dissipation of, and otherwise conserving and preserving the same. the Sandiganbayan cautioned that "the PCGG, the UCPB and the SMC Group shall always act with due regard to the sequestered character of the shares of stock involved as well as the fruits thereof, more particularly to prevent the loss or dissipation of their value. The Sandiganbayan observed that the conversion of the SMC shares to treasury shares will result in a change in the status of the sequestered shares in that: (1) When the SMC converts these common shares to treasury stock, it is converting those outstanding shares into the corporation's property for which reasontreasury shares do not earn dividends; (2) The retained dividends which would have accrued to those shares if converted to treasury would go into the corporation and enhance the corporation as a whole. The enhancement to the specific sequestered shares, however, would be only to the extent aliquot in relation to all the other outstanding SMC shares; (3) By converting the 26.45 million shares of stock into treasury shares, the SMC has altered not only the voting power of those shares of stock since treasury shares do not vote, but the SMC will have actually enhanced the voting strength of the other outstanding shares of stock to the extent that these 26.45 million shares no longer vote. As to the payment to the PCGG of an arbitration fee in the form of 5,500,000 of SMC shares is denounced as illegal, shocking and unconscionable. COCOFED, et al. have assailed the legal right of PCGG to act as arbiter as well as the fairness of its acts as arbiter. COCOFED, et al. estimate that the value of the SMC shares given to PCGG as arbitration fee which allegedly is not deserved, can run toP1,966,635,000.00. This is a serious allegation and the Sandiganbayan cannot be charged with grave abuse of discretion when it ordered that SMC should

be temporarily dispossessed of the subject treasury shares and that SMC should pay their dividends while the Compromise Agreement involving them is still under question.

PAID-UP CAPITAL MSCI VS. NATIONAL WAGE AND PRODUCTIVITY COMMISSION G.R. No. 125198 Facts: Monomer Sugar Central Inc. (MSCI) was created by the merger of Asturias Sugar Central,Inc. (ASCI) and tries, Inc. (MTII). MSCI applied for exemption from the coverage of Wage Order No. RO VI-01 issued by the Board on the ground that it is a distressed employer. However, the Petitioners, local union of MSCI, contested the application saying that the documents submitted by the company does not reflect its true and valid financial status and that the paid-up capital would have been higher than P5 million and thus impairment would have been lower than 25% had the preorganization agreement between ASCI and MTII been complied with. Petitioners argue that the paid-up capital should be P64M rather than P5M thus making it impossible for the corporation to apply for a distressed employer because at P64M, the loss would only be at 5.25% which does not reach the required 25% loss. Issue: Held: Since the paid-up capital is the portion of the capital which has been subscribed and paid, the assets transferred to and the loans extended to a corporation should not be considered in computing the paid-up capital of Whether or not the paid up capital is P64M or P5M.

the corporation. Not all funds or assets received by the corporation can be considered as paid-up capital for this term has a technical signification in Corporation Law. Such must form part of the authorized capital of the corporation, subscribed and then actually paid-up. The same test should also be applied in determining if the paid-up capital of the Corporation has been impaired so as to qualify it for the exemption from the increase in the minimum wages. The paid-up capital stock of MSCI for the period covered by the application for exemption still stood at P5M. The impairment it has incurred reached 271.08% which is beyond the 25% requirement to be an applicant.

Issue: Whether or not the dead members should still be counted in the quorum.

Held: Based on the By-Laws of the corporation, the remaining members of the board of trustees of the GCHS may convene and fill up the vacancies in the board. Except as provided, the vote necessary to approve a particular corporate act as provided in this Code shall be deemed to refer only to stocks with voting rights: (1) Amendment of the articles of incorporation; (2) Adoption and amendment of by-laws; (3) Sale, lease, exchange, mortgage, pledge or other disposition of all or substantially all of the corporation property; (4) Incurring, creating or increasing bonded indebtedness; (5) Increase or decrease of capital stock; (6) Merger or consolidation of the corporation with another corporation or other corporations; (7) Investment of corporate funds in another corporation or business in accordance with this Code; and (8) Dissolution of the corporation. A quorum in a members meeting is to be reckoned as the actual number of members of the corporation. In stock corporations, the shareholders may generally transfer their shares on the death of a shareholder, the executor or administrator duly appointed by the Court is vested with the legal title to the stock and entitled to vote it. Until a settlement and division of the estate is effected, the stocks of the decedent are held by the administrator or executor. As to non-stock corporations, the same is personal and non-transferable unless the articles of incorporation or the bylaws of the corporation provide otherwise. Under Section 91 of the Corporation Code, termination extinguishes all the rights of a member of the corporation, unless otherwise provided in the articles of incorporation or the bylaws. Whether or not

QUORUM TAN VS. SYCIP G.R. No. 153468 August 17, 2006 Facts: Grace Christian High School (GCHS) is a non-stock non-profit educational corporation with 15 regular members, who also constitute the board of trustees. During the annual members meeting, there were only 11 living member-trustees as 4 had already died. 7 attended the meeting through their respective proxies. The meeting was convened and chaired by Atty. Sabino Padilla Jr. over the objection of Atty. Antonio C. Pacis who argued that there was no quorum. In the meeting, Petitioners Ernesto Tanchi, Edwin Ngo, Virgin Khoo, and Judith Tan were voted to replace the 4 deceased member-trustees. According to the SEC, the meeting was void due to lack of quorum based on the AIC, also applying Section 24 and Section 89 of the Corporation Code. When it reached the CA, the latter dismissed the case due to technicalities.

"dead members" are entitled to exercise their voting rights (through their executor or administrator), depends on those articles of incorporation or bylaws. Under the By-Laws of GCHS, membership in the corporation shall be terminated by the death of the member. Hence, with 11 remaining members, the quorum is 6. Under Section 29, vacancies in the office of the director or trustee is to be filled by a majority vote of the remaining directors or trustees if still constituting a quorum. Otherwise, said vacancies must be filled by the stockholders in a regular or special meeting called for that purpose. A director or trustee so elected to fill a vacancy shall be elected only for the unexpired term of his predecessor in office. The filling of vacancies in the board by the remaining directors or trustees constituting a quorum is merely permissive, not mandatory; either by the remaining directors constituting a quorum, or by the stockholders or members in a regular or special meeting called for the purpose. The ByLaws of GCHS prescribed the specific mode of filling up existing vacancies in its board of directors; that is, by a majority vote of the remaining members of the board. The said remaining member-trustees must sit as a board (as a body in a lawful meeting) in order to validly elect the new ones.

its corporate articles it was to exist for 50 years from incorporation. When it expired, it ceased transacting business and entered into a state of liquidation. Thereafter, a new corporation, Alhambra Industries, Inc., was formed to carry on the business of Alhambra. Alhambras stockholders, by resolution, named Angel S. Gamboa trustee to take charge of its liquidation. On June 20, 1963, within Alhambras three-year statutory period for liquidation, Republic Act 3531 was enacted into law. It amended Section 18 of the Corporation Law by empowering domestic private corporations to extend their corporate life beyond the period fixed by the articles of incorporation for a term not to exceed fifty years in any one instance. Previous to Republic Act 3531, the maximum non-extendible term of such corporations was fifty years. At a special meeting, Alhambras board of directors resolved to amend paragraph Fourth of its articles of incorporation to extend its corporate life for an additional fifty years, or a total of 100 years from its incorporation. Alhambras stockholders, representing more than two-thirds of its subscribed capital stock, voted to approve the foregoing resolution. Alhambras articles of incorporation, as amended, was then submitted to the SEC. The latter however, returned said amended articles of incorporation to Alhambras counsel with the ruling that Republic Act 3531 which took effect only on June 20, 1963, cannot be availed of by the said corporation, for the reason that its term of existence had already expired when the said law took effect in short, said law has no retroactive effect. Issue: May a corporation extend its life by amendment of its articles of incorporation effected during the three-year statutory period for liquidation when its original term of existence had already expired? Held: When Alhambra made its attempt to extend its corporate existence, its original term of fifty years had already expired and it was in the midst of

TERM OF EXISTENCE ALHAMBRA CIGAR VS. SEC G.R. No. L-23606 July 29, 1968 Facts: Petitioner Alhambra Cigar and Cigarette Manufacturing Company, Inc. was duly incorporated under Philippine laws on January 15, 1912. By

the three-year grace period statutorily fixed in Section 77 of the Corporation Law. Plain from the language of the provision is its meaning: continuance of a dissolved corporation as a body corporate for three years has for its purpose the final closure of its affairs, and no other; the corporation is specifically enjoined from continuing the business for which it was established. The liquidation of the corporations affairs set forth in said section became because its life had ended. For this reason alone, the corporate existence and juridical personality of that corporation to do business may no longer be extended. Since the privilege of extension is purely statutory, all of the statutory conditions precedent must be complied with in order that the extension may be effectuated during the life of the corporation, and before the expiration of the term of existence as original fixed by its charter or the general law, since, as a rule, the corporation is ipso facto dissolved as soon as that time expires. The filing and recording of a certificate of extension after that time cannot relate back to the date of the passage of the stockholders resolution. The contrary is true, however, and the doctrine of relation will apply, where the delay is due to the neglect of the officer with whom the certificate is required to be filed, or to a wrongful refusal to receive it. A distinction is to be made between the extension of a charter and the grant of a new one. To renew a charter is to revive a charter which has expired, or, in other words, to give a new existence to one which has been forfeited, or which has lost its vitality by lapse of time. To extend a charter is to increase the time for the existence of one which would otherwise reach its limit at an earlier period. Nowhere in Section 18 is the word renew. The law limits itself to extension of corporate existence.

BY LAWS FLEISCHER VS. BOTICA NOLASCO Facts: The plaintiff filed an amended complaint against the Botica Nolasco, Inc., alleging that he became the owner of five shares of stock of said corporation, by purchase from their original owner, one Manuel Gonzalez; that the said shares were fully paid; and that the defendant refused to register said shares in his name in the books of the corporation in spite of repeated demands to that effect made by him upon said corporation, which refusal caused him damages amounting to P500. Plaintiff prayed for a judgment ordering the Botica Nolasco, Inc. to register in his name in the books of the corporation the five shares of stock recorded in said books in the name of Manuel Gonzalez, and to indemnify him in the sum of P500 as damages, and to pay the costs. The defendant again filed a demurrer on the ground that the amended complaint did not state facts sufficient to constitute a cause of action, and that said amended complaint was ambiguous, unintelligible, uncertain, which demurrer was overruled by the court. The defendant answered the amended complaint denying generally and specifically each and every one of the material allegations thereof, and, as a special defense, alleged that the defendant, pursuant to article 12 of its by-laws, had preferential right to buy from the plaintiff said shares at the par value of P100 a share, plus P90 as dividends corresponding to the year 1922, and that said offer was refused by the plaintiff. The defendant prayed for a judgment absolving it from all liability under the complaint and directing the plaintiff to deliver to the defendant the five shares of stock in question, and to pay damages in the sum of P500, and the costs. Upon the issue presented by the pleadings above stated, the cause was brought on for trial, at the conclusion of which, and on August 21,

1924, the Honorable N. Capistrano, judge, held that, in his opinion, article 12 of the by-laws of the corporation which gives it preferential right to buy its shares from retiring stockholders, is in conflict with Act No. 1459 (Corporation Law), especially with section 35 thereof; and rendered a judgment ordering the defendant corporation, through its board of directors, to register in the books of said corporation the said five shares of stock in the name of the plaintiff, Henry Fleischer, as the shareholder or owner thereof, instead of the original owner, Manuel Gonzalez, with costs against the defendant. Issue: Whether or not article 12 of the by-laws of the corporation is in conflict with the provisions of the Corporation Law (Act No. 1459) Held: As a general rule, the by-laws of a corporation are valid if they are reasonable and calculated to carry into effect the objects of the corporation, and are not contradictory to the general policy of the laws of the land. (Supreme Commandery of the Knights of the Golden Rule vs. Ainsworth, 71 Ala., 436; 46 Am. Rep., 332.) On the other hand, it is equally well settled that by-laws of a corporation must be reasonable and for a corporate purpose, and always within the charter limits. They must always be strictly subordinate to the constitution and the general laws of the land. They must not infringe the policy of the state, nor be hostile to public welfare. (46 Am. Rep., 332.) They must not disturb vested rights or impair the obligation of a contract, take away or abridge the substantial rights of stockholder or member, affect rights of property or create obligations unknown to the law. (People's Home Savings Bank vs. Superior Court, 104 Cal., 649; 43 Am. St. Rep., 147; Ireland vs. Globe Milling Co., 79 Am. St. Rep., 769.)

The validity of the by-law of a corporation is purely a question of law. (South Florida Railroad Co. vs. Rhodes, 25 Fla., 40.) The power to enact by-laws restraining the sale and transfer of stock must be found in the governing statute or the charter. Restrictions upon the traffic in stock must have their source in legislative enactment, as the corporation itself cannot create such impediments. By-law are intended merely for the protection of the corporation, and prescribe regulation and not restriction; they are always subject to the charter of the corporation. The corporation, in the absence of such a power, cannot ordinarily inquire into or pass upon the legality of the transaction by which its stock passes from one person to another, nor can it question the consideration upon which a sale is based. A by-law cannot take away or abridge the substantial rights of stockholder. Under a statute authorizing by- laws for the transfer of stock, a corporation can do no more than prescribe a general mode of transfer on the corporate books and cannot justify an unreasonable restriction upon the right of sale. (4 Thompson on Corporations, sec. 4137, p. 674. The right of unrestrained transfer of shares inheres in the very nature of a corporation, and courts will carefully scrutinize any attempt to impose restrictions or limitations upon the right of stockholders to sell and assign their stock. The right to impose any restraint in this respect must be conferred upon the corporation either by the governing statute or by the articles of the corporation. It cannot be done by a by-law without statutory or charter authority.

PREFFERED AND COMMON STOCKS PHILIPPINE COCONUT PRODUCERS FEDERATION, INC. (COCOFED),et. al. vs. RP G.R. Nos. 177857-58 September 17, 2009 Facts: COCOFED seeks the Courts approval of the conversion of 753,848,312 Class A and Class B common shares of San Miguel Corporation (SMC) registered in the names of Coconut Industry Investment Fund and the so-called 14 Holding Companies into 753,848,312 SMC Series 1 Preferred Shares. COCOFED proposes to constitute a trust fund to be known as the Coconut Industry Trust Fund (CITF) for the Benefit of the Coconut Farmers, with respondent Republic, acting through the Philippine Coconut Authority (PCA), as trustee. As proposed, the constitution of the CITF shall be subject to terms and conditions which, for the most part, reiterate the features of SMCs conversion offer, albeit specific reference is made to the shares of the 14 CIIF companies. Republic of the Philippines filed its Comment questioning COCOFEDs personality to seek the Courts approval of the desired conversion. Respondent Republic also disputes COCOFEDs right to impose and prescribe terms and conditions on the proposed conversion, maintaining that the CIIF SMC common shares are sequestered assets and are in 29ustodial egis under Presidential Commission on Good Governments (PCGGs) administration. It postulates that, owing to the sequestrated status of the said common shares, only PCGG has the authority to approve the proposed conversion and seek the necessary Court approval. Jovito R. Salonga and four others sought leave to intervene asserting that the government bears the burden of showing that the conversion is indubitably advantageous to the public interest or will result in clear and material benefit. Failure of the government to carry the burden means that the current status of the sequestered stocks should be maintained pending final disposition of G.R. Nos. 177857-58. They further postulate that even assuming that the proposal to convert the SMC shares is beneficial to the government, it cannot pursue the exchange offer because it is without power to exercise acts of strict dominion over the sequestered shares. Lastly, they argue that the proposed conversion x x x is not only not advantageous to the public interest but is in fact positively disadvantageous. Respondent Republic filed a Supplemental Comment in which it cited the Partial Summary Judgment rendered by the Sandiganbayan on May 27, 2004 in Civil Case No. 33-F, declaring the Republic as owner, in trust for the coconut farmers, of the subject CIIF SMC shares (27%). The same comment also referred to Resolution No. 365-2009 passed on August 28, 2009 by the United Coconut Planters Bank (UCPB) Board of Directors expressing the sense that the proposed conversion of the CIIF SMC common shares to SMC Series I preferred shares is financially beneficial. By way of relief, respondent Republic prayed that the PCGG be allowed to proceed and effect the conversion. Issue: Whether or not PCGG, not COCOFED, was the authorized party to seek the imprimatur on the conversion of the Series 1 preferred shares

Held:

As records show, PCGG sequestered the 753,848,312 SMC common shares registered in the name of CIIF companies on April 7, 1986. From that time on, these sequestered shares became subject to the management, supervision, and control of PCGG, pursuant to Executive Order No. (EO) 1, Series of 1986. Eventually, the coconut levy funds that were used to acquire the sequestered CIIF SMC common shares in question were peremptorily determined to be prima facie public funds. PCGGs authority to vote the sequestered shares acquired from the coconut levy is based on the principle that voting is an act of dominion that should be exercised by the share owner. One of the recognized rights of an owner is the right to vote at meetings of the corporation. The right to vote is classified as the right to control. Voting rights may be for the purpose of, among others, electing or removing directors, amending a charter, or making or amending by laws. Because the subject UCPB shares were acquired with government funds, the government becomes their prima facie beneficial and true owner. Ownership includes the right to enjoy, dispose of, exclude and recover a thing without limitations other than those established by law or by the owner. X x x And the right to vote shares is a mere incident of ownership. In the present case, the government has been shown to be the prima facie owner of the funds used to purchase the shares. Hence, it should be allowed the rights and privileges flowing from such fact.

Oscar and Rodrigo C. Reyes are two of the four children of the spouses Pedro and Anastacia Reyes. Pedro, Anastacia, Oscar, and Rodrigo; each owned shares of stock of Zenith Insurance Corporation. Zenith is a domestic corporation established by their family. When Pedro and Anastacia died, Pedro's estate was judicially partitioned among his heirs but not with Anastacia's estate, which included her shareholdings in Zenith. Zenith and Rodrigo filed a complaint with the SEC against Oscar, stating that it is "a derivative suit initiated and filed by the complainant Rodrigo C. Reyes to obtain an accounting of the funds and assets of Zenith which are now or formerly in the control, custody, and/or possession of Oscar and to determine the shares of stock of deceased spouses Pedro and Anastacia Reyes that were arbitrarily and fraudulently appropriated by Oscar for himself and which were not collated and taken into account in the partition, distribution, and/or settlement of the estate of the deceased spouses, for which he should be ordered to account for all the income from the time he took these shares of stock, and should now deliver to his brothers and sisters their just and respective shares. Oscar denied the charge and asserted, as a defense asserted that he purchased the subject shares with his own funds from the unissued stocks of Zenith. He claimed that the complaint is a mere nuisance or harassment suit and should, according to the Interim Rules of Procedure for Intra-Corporate Controversies, be dismissed; and that it is not a bona fide derivative suit as it partakes of the nature of a petition for the settlement of estate of the deceased Anastacia that is outside the jurisdiction of a special commercial court. The trial court ruled that it is not a derivative suit and should properly be threshed out in a petition for settlement of estate. This was affirmed by the Court of Appeals. Issues: Whether or not there exists herein an intra-corporate controversy.

TRANSFER OF STOCKS REYES VS. RTC OF MAKATI 561 SCRA 593 (2008) Facts:

Held: Initially, the main consideration in determining whether a dispute constitutes an intra-corporate controversy was limited to a consideration of the intra-corporate relationship existing between or among the parties. The types of relationships embraced under Section 5(b), were as follows: (a) between the corporation, partnership, or association and the public; (b) between the corporation, partnership, or association and its stockholders, partners, members, or officers; (c) between the corporation, partnership, or association and the State as far as its franchise, permit or license to operate is concerned; and (d) among the stockholders, partners, or associates themselves. The existence of any of the above intra-corporate relations was sufficient to confer jurisdiction to the SEC, regardless of the subject matter of the dispute. This came to be known as the relationship test. However, in the 1984 case of DMRC Enterprises v. Esta del Sol Mountain Reserve, Inc., the Court introduced the nature of the controversy test. The Court declared in this case that it is not the mere existence of an intra-corporate relationship that gives rise to an intra-corporate controversy; to rely on the relationship test alone will divest the regular courts of their jurisdiction for the sole reason that the dispute involves a corporation, its directors, officers, or stockholders. Under the nature of the controversy test, the incidents of that relationship must also be considered for the purpose of ascertaining whether the controversy itself is intra-corporate. The controversy must not only be rooted in the existence of an intra-corporate relationship, but must as well pertain to the enforcement of the parties' correlative rights and obligations under the Corporation Code and the internal and intra-corporate regulatory rules of the corporation. If the relationship and its incidents are merely incidental to the controversy or if

there will still be conflict even if the relationship does not exist, then no intra-corporate controversy exists. The Court then combined the two tests and declared that jurisdiction should be determined by considering not only the status or relationship of the parties, but also the nature of the question under controversy. Thus under the relationship test, the transfer of title by means of succession, though effective and valid between the parties involved (i.e. between the decedents estate and her heirs) does not bind the corporation and third parties. The transfer must be registered in the books of the corporation to make the transferee-heir a stockholder entitled recognition as such both by the corporation and by third parties. Therefore, each of the decedents heirs holds only an undivided interest in the shares. This interest is still inchoate and subject to the outcome of a settlement proceeding; the right of the heirs to specific, distributive shares of inheritance will not will not be determined until all the debts of the estate of the decedent are paid. Insofar as the subject shares of stock are concernedthe heir of the deceased stockholder cannot be considered a stockholder of the corporation. Applying the nature of the controversy test, an accounting of funds and assets of the corporation to determine the extent and value of the decedents shareholdings will be undertaken by the probate court and not by a special commercial court is consistent with the probate courts limited jurisdiction. Hence, the case is not considered intra-corporate controversy even if the dispute is among stockholders if the issue is determination and distribution of successional rights to the shareholdings of a deceased shareholder.

TCL SALES CORP. VS. COURT OF APPEALS G.R. No. 129777 January 5, 2001 Facts: Ting Ping Lay, not one of the original subscribers of the shares of stock of TCL Sales Corporation, acquired his shares by purchasing those of some of the original subscribers. In order to protect his shareholdings with TCL, Lay requested Anna Teng, TCL Corporate Secretary to enter the transfer of shares of stock for proper recording of his acquisitions in the Stock & Transfer Book of TCL. He too demanded issuance of new certificates of stock in his favor. TCL, however, even after repeated demands, refused. Lay filed a case with the SEC for mandamus against TCL and Teng. This was in turn granted by the SEC denying a later MR as well. The CA dismissed TCLs petition as well for being filed out of time. Issues: (1 )Whether or not SEC has jurisdiction over the petition for mandamus filed by Lay. (2) Whether or not the alleged transfer of shares in favor of Lay are valid and can be ordered recorded. Held: (1) The principal function of SEC is supervision and control of corps, partnerships, associations with the view of protecting and encouraging investments for the protection of economic development. SEC has power of control & supervision over all corps to encourage active public participation in the affairs of private corps through investments. Jurisdiction over an action for mandamus lies with the SEC even if the proponent is not yet a shareholder of record, as in the case of Abejo v. de la Cruz. SEC by express mandate has absolute jurisdiction to enforce the

provisions of the Corp Code among which is the stock purchasers right to secure the corresponding certificate of stock in his name. (2) Even if Lay were not a Share Holder, he is still a member of the public whose investment in the corporate the law seeks to protect and encourage, as his purchase of shares of stock has been established. Determination of whether or not a Share Holder is entitled to exercise the rights of a Share Holder is within jurisdiction of the SEC. The SEC en banc found that TCL did not refute the validity of the transfers of the shares of stock they conceded that they could not assail the documents evincing the transfer of the shares to Lay. Lay was able to establish prima facie ownership through the deeds of transfer of shares of stock of TCL. A listing of TCLs Share Holders & their respective shares before & after the execution of a certain deed of assignment shows that Lay is indeed listed as a Share Holder of TCL. The dispute is an intra-corp controversy involving Share Holders of TCL. The duty of the corporate secretary to record transfers of stocks is ministerial. It however, cannot be compelled when the transferees title has no prima facie validity or is uncertain. Mandamus will not issue to establish a right but only to enforce one already established. Although during the trial before the SEC, TCL admitted that they ignored Lays request was based simply on the fact that they did not want to grant it. Having been capricious, whimsical & unwarranted, it constitutes bad faith. However, the SEC en banc modified & deleted the said award for damages imposed on the corp. The matter of damages now concerns only Teng, the corporate secretary. It was Tengs refusal as corp secretary to record the transfer of the shares, without evidence that such refusal was authorized by TCLs Board of Directors that caused damage

RURAL BANK OF LIPA CITY VS. COURT OF APPEALS GR No. 124535 September 28, 2001 Facts: Private respondent Reynaldo Villanueva, Sr., a stockholder of the Rural Bank of Lipa City, executed a Deed of Assignment, wherein he assigned his shares, as well as those of eight (8) other shareholders under his control with a total of 10,467 shares, in favor of the stockholders of the Bank represented by its directors Bernardo Bautista, Jaime Custodio and Octavio Katigbak. Sometime thereafter, Reynaldo Villanueva, Sr. and his wife, Avelina, executed an Agreement wherein they acknowledged their indebtedness to the Bank in the amount of Four Million Pesos (P4,000,000.00), and stipulated that said debt will be paid out of the proceeds of the sale of their real property described in the Agreement. At a meeting of the Board of Directors of the Bank on November 15, 1993, the Villanueva spouses assured the Board that their debt would be paid on or before December 31 of that same year; otherwise, the Bank would be entitled to liquidate their shareholdings, including those under their control. In such an event, should the proceeds of the sale of said shares fail to satisfy in full the obligation, the unpaid balance shall be secured by other collateral sufficient therefor. When the Villanueva spouses failed to settle their obligation to the Bank on the due date, the Board sent them a letter demanding: (1) the surrender of all the stock certificates issued to them; and (2) the delivery of sufficient collateral to secure the balance of their debt amounting to P3,346,898.54. The Villanuevas ignored the bank's demands, whereupon their shares of stock were converted into Treasury Stocks. Later, the Villanuevas, through their counsel, questioned the legality of the conversion of their shares. On January 15, 1994, the stockholders of the Bank met to elect the new directors and set of officers for the year 1994. The Villanuevas were not notified of said meeting. In a letter dated January 19, 1994, Atty. Amado Ignacio, counsel for the Villanueva spouses, questioned the legality of the said stockholders' meeting and the validity of all the proceedings therein. In reply, the new set of officers of the Bank informed Atty. Ignacio that the Villanuevas were no longer entitled to notice of the said meeting since they had relinquished their rights as stockholders in favor of the Bank. Consequently, the Villanueva spouses filed with the Securities and Exchange Commission (SEC), a petition for

annulment of the stockholders' meeting and election of directors and officers on January 15, 1994, with damages and prayer for preliminary injunction. Issue: Whether or not there is a valid transfer of shares of stocks from private respondents to petitioners resulting to the withdrawal of the rights of the former as a shareholder. Held: The Supreme Court have uniformly held that for a valid transfer of stocks, there must be strict compliance with the mode of transfer prescribed by law. The requirements are: (a) There must be delivery of the stock certificate: (b) The certificate must be endorsed by the owner or his attorney-in-fact or other persons legally authorized to make the transfer; and (c) To be valid against third parties, the transfer must be recorded in the books of the corporation. As it is, compliance with any of these requisites has not been clearly and sufficiently shown. It may be argued that despite non-compliance with the requisite endorsement and delivery, the assignment was valid between the parties, meaning the private respondents as assignors and the petitioners as assignees. While the assignment may be valid and binding on the petitioners and private respondents, it does not necessarily make the transfer effective. Consequently, the petitioners, as mere assignees, cannot enjoy the status of a stockholder, cannot vote nor be voted for, and will not be entitled to dividends, insofar as the assigned shares are concerned Parenthetically, the private respondents cannot, as yet, be deprived of their rights as stockholders, until and unless the issue of ownership and transfer of the shares in question is resolved with finality.

CORPORATION BY ESTOPPEL LIM TONG LIM V. PHILIPPINE FISHING GEAR INDUSTRIES, INC. G.R. No. 136448 Facts: On behalf of "Ocean Quest Fishing Corporation," Antonio Chua and Peter Yao entered into a Contract for the purchase of fishing nets of various sizes from the Philippine Fishing Gear Industries, Inc. (PFGI). They claimed that they were engaged in a business venture with Lim Tong Lim, who however was not a signatory to the agreement. The total price of the nets amounted to P532,045. 400 pieces of floats worth P68,000 were also sold to the Corporation. The buyers, however, failed to pay for the fishing nets and the floats; hence, PFGI filed a collection suit against Chua, Yao and Lim Tong Lim with a prayer for a writ of preliminary attachment. The suit was brought against the three in their capacities as general partners, on the allegation that "Ocean Quest Fishing Corporation" was a nonexistent corporation as shown by a Certification from the Securities and Exchange Commission. The lower court issued a Writ of Preliminary Attachment, which the sheriff enforced by attaching the fishing nets on board F/B Lourdes which was then docked at the Fisheries Port, Navotas, Metro Manila. Lim Tong Lim, on the other hand, filed an Answer with Counterclaim and Crossclaim and moved for the lifting of the Writ of Attachment. Lim argues, among others, that under the doctrine of corporation by estoppel, liability can be imputed only to Chua and Yao, and not to him, the formers being those who dealt in the name of the ostensible corporation. Issue: Held: Whether Lim should be held jointly liable with Chua and Yao.

In the first instance, an unincorporated association, which represented itself to be a corporation, will be estopped from denying its corporate capacity in a suit against it by a third person who relied in good faith on such representation. It cannot allege lack of personality to be sued to evade its responsibility for a contract it entered into and by virtue of which it received advantages and benefits. On the other hand, a third party who, knowing an association to be unincorporated, nonetheless treated it as a corporation and received benefits from it, may be barred from denying its corporate existence in a suit brought against the alleged corporation. In such case, all those who benefited from the transaction made by the ostensible corporation, despite knowledge of its legal defects, may be held liable for contracts they impliedly assented to or took advantage of. Although technically it is true that Lim did not directly act on behalf of the corporation; however, having reaped the benefits of the contract entered into by persons with whom he previously had an existing relationship, he is deemed to be part of said association and is covered by the scope of the doctrine of corporation by estoppel.

CORPORATION SOLE ROMAN CATHOLIC APOSTOLIC ADMINISTRATOR OF DAVAO VS. THE LRC G.R. No. L-8451 Facts: Mateo Rodis, a Filipino citizen and resident of Davao, executed a deed of sale of a parcel of land located in the same city in favour of the Roman Catholic Administrator of Davao, a corporation sole organized and existing in accordance with Philippine laws whose incumbent administrator is Msgr. Clovis Thibault, a Canadian citizen. When the deed was presented to the Register of Deeds for registration, it required them to

submit an affidavit stating that the ownership of the corporation is 60% Filipino citizens as required under the Constitution. The Roman Catholic stated that it was a corporation sole and that the totality of the Catholic population in Davao would become the owner of the property. The Register of Deeds doubted this and submitted the case of en consulta in the Land Registration Commission. The LRC ruled that the requirement of the Constitution must be followed. Hence, since the 60% cannot be complied with, the registration should be denied. Issue: Whether or not the Roman Catholic Apostolic Church, being a corporation sole, can lawfully acquire lands in the Philippines. Held: A corporation sole is a special form of corporation usually associated with the clergy designed to facilitate the exercise of the functions of ownership of the church which was registered as property owner. It is created not only to administer the temporalities of the church or religious society where the corporator belongs, but also to hold and transmit the same to his successor in said officer. The incumbent administrator is not the actual owner of the land but the constituents or those that make up the church, thus it is their nationality that has to be taken into consideration. The corporation sole only holds the property in trust for the benefit of the Roman Catholic faithful. The Roman Catholic Church is a corporation by prescription, with acknowledged juridical personality inasmuch as it is an institution which antedated almost a thousand years any other personality in Europe. Since it is a corporation by prescription, it has no nationality, and hence, the nationality test does not apply.

CORPORATE ACTS GOKONGWEI VS. SECURITIES AND EXCHANGE COMMISSION GR L-45911 11 April 1979 Facts: [SEC Case 1375] On 22 October 1976, John Gokongwei Jr., as stockholder of San Miguel Corporation, filed with the Securities and Exchange Commission (SEC) a petition for "declaration of nullity of amended by-laws, cancellation of certificate of filing of amended by-laws, injunction and damages with prayer for a preliminary injunction" against the majority of the members of the Board of Directors and San Miguel Corporation as an unwilling petitioner. As a first cause of action, Gokongwei alleged that on 18 September 1976, Andres Soriano, Jr., Jose M. Soriano, Enrique Zobel, Antonio Roxas, Emeterio Buao, Walthrode B. Conde, Miguel Ortigas, and Antonio Prieto amended by bylaws of the corporation, basing their authority to do so on a resolution of the stockholders adopted on 13 March 1961, when the outstanding capital stock of the corporation was only P70,139.740.00, divided into 5,513,974 common shares at P10.00 per share and 150,000 preferred shares at P100.00 per share. At the time of the amendment, the outstanding and paid up shares totalled 30,127,043, with a total par value of P301,270,430.00. It was contended that according to section 22 of the Corporation Law and Article VIII of the by-laws of the corporation, the power to amend, modify, repeal or adopt new by-laws may be delegated to the Board of Directors only by the affirmative vote of stockholders representing not less than 2/3 of the subscribed and paid up capital stock of the corporation, which 2/3 should have been computed on the basis of the capitalization at the time of the amendment. Since the amendment was based on the 1961 authorization, Gokongwei contended that the Board acted without authority and in

usurpation of the power of the stockholders. As a second cause of action, it was alleged that the authority granted in 1961 had already been exercised in 1962 and 1963, after which the authority of the Board ceased to exist. As a third cause of action, Gokongwei averred that the membership of the Board of Directors had changed since the authority was given in 1961, there being 6 new directors. As a fourth cause of action, it was claimed that prior to the questioned amendment, Gokogwei had all the qualifications to be a director of the corporation, being a substantial stockholder thereof; that as a stockholder, Gokongwei had acquired rights inherent in stock ownership, such as the rights to vote and to be voted upon in the election of directors; and that in amending the by-laws, Soriano, et. al. purposely provided for Gokongwei's disqualification and deprived him of his vested right as aforementioned, hence the amended by-laws are null and void. As additional causes of action, it was alleged that corporations have no inherent power to disqualify a stockholder from being elected as a director and, therefore, the questioned act is ultra vires and void; that Andres M. Soriano, Jr. and/or Jose M. Soriano, while representing other corporations, entered into contracts (specifically a management contract) with the corporation, which was avowed because the questioned amendment gave the Board itself the prerogative of determining whether they or other persons are engaged in competitive or antagonistic business; that the portion of the amended bylaws which states that in determining whether or not a person is engaged in competitive business, the Board may consider such factors as business and family relationship, is unreasonable and oppressive and, therefore, void; and that the portion of the amended by-laws which requires that "all nominations for election of directors shall be submitted in writing to the Board of Directors at least five (5) working days before the date of the Annual Meeting" is likewise unreasonable and oppressive. It was, therefore, prayed that the amended by-laws be declared null and void and the certificate of filing thereof be cancelled, and that Soriano, et. al. be made to pay damages, in specified amounts, to Gokongwei. On 28 October 1976, in connection with the same case, Gokongwei filed with the Securities and

Exchange Commission an "Urgent Motion for Production and Inspection of Documents", alleging that the Secretary of the corporation refused to allow him to inspect its records despite request made by Gokongwei for production of certain documents enumerated in the request, and that the corporation had been attempting to suppress information from its stockholders despite a negative reply by the SEC to its query regarding their authority to do so. The motion was opposed by Soriano, et. al. The Corporation, Soriano, et. al. filed their answer, and their opposition to the petition, respectively. Meanwhile, on 10 December 1976, while the petition was yet to be heard, the corporation issued a notice of special stockholders' meeting for the purpose of "ratification and confirmation of the amendment to the By-laws", setting such meeting for 10 February 1977. This prompted Gokongwei to ask the SEC for a summary judgment insofar as the first cause of action is concerned, for the alleged reason that by calling a special stockholders' meeting for the aforesaid purpose, Soriano, et. al. admitted the invalidity of the amendments of 18 September 1976. The motion for summary judgment was opposed by Soriano, et. al. Pending action on the motion, Gokongwei filed an "Urgent Motion for the Issuance of a Temporary Restraining Order", praying that pending the determination of Gokongwei's application for the issuance of a preliminary injunction and or Gokongwei's motion for summary judgment, a temporary restraining order be issued, restraining Soriano, et. al. from holding the special stockholders' meeting as scheduled. This motion was duly opposed by Soriano, et. al. On 10 February 1977, Cremation issued an order denying the motion for issuance of temporary restraining order. After receipt of the order of denial, Soriano, et. al. conducted the special stockholders' meeting wherein the amendments to the by-laws were ratified. On 14 February 1977, Gokongwei filed a consolidated motion for contempt and for nullification of the special stockholders' meeting. A motion for reconsideration of the order denying Gokongwei's motion for summary judgment was filed by Gokongwei before the SEC on 10 March 1977.

[SEC Case 1423] Gokongwei alleged that, having discovered that the corporation has been investing corporate funds in other corporations and businesses outside of the primary purpose clause of the corporation, in violation of section 17-1/2 of the Corporation Law, he filed with SEC, on 20 January 1977, a petition seeking to have Andres M. Soriano, Jr. and Jose M. Soriano, as well as the corporation declared guilty of such violation, and ordered to account for such investments and to answer for damages. On 4 February 1977, motions to dismiss were filed by Soriano, et. al., to which a consolidated motion to strike and to declare Soriano, et. al. in default and an opposition ad abundantiorem cautelam were filed by Gokongwei. Despite the fact that said motions were filed as early as 4 February 1977, the Commission acted thereon only on 25 April 1977, when it denied Soriano, et. al.'s motions to dismiss and gave them two (2) days within which to file their answer, and set the case for hearing on April 29 and May 3, 1977. Soriano, et. al. issued notices of the annual stockholders' meeting, including in the Agenda thereof, the "reaffirmation of the authorization to the Board of Directors by the stockholders at the meeting on 20 March 1972 to invest corporate funds in other companies or businesses or for purposes other than the main purpose for which the Corporation has been organized, and ratification of the investments thereafter made pursuant thereto." By reason of the foregoing, on 28 April 1977, Gokongwei filed with the SEC an urgent motion for the issuance of a writ of preliminary injunction to restrain Soriano, et. al. from taking up Item 6 of the Agenda at the annual stockholders' meeting, requesting that the same be set for hearing on 3 May 1977, the date set for the second hearing of the case on the merits. The SEC, however, cancelled the dates of hearing originally scheduled and reset the same to May 16 and 17, 1977, or after the scheduled annual stockholders' meeting. For the purpose of urging the Commission to act, Gokongwei filed an urgent manifestation on 3 May 1977, but this notwithstanding, no action has been taken up to the date of the filing of the instant petition.

Gokongwei filed a petition for petition for certiorari, mandamus and injunction, with prayer for issuance of writ of preliminary injunction, with the Supreme Court, alleging that there appears a deliberate and concerted inability on the part of the SEC to act. Issue: Whether the corporation has the power to provide for the (additional) qualifications of its directors. Held: It is recognized by all authorities that "every corporation has the inherent power to adopt by-laws 'for its internal government, and to regulate the conduct and prescribe the rights and duties of its members towards itself and among themselves in reference to the management of its affairs.'" In this jurisdiction under section 21 of the Corporation Law, a corporation may prescribe in its by-laws "the qualifications, duties and compensation of directors, officers and employees." This must necessarily refer to a qualification in addition to that specified by section 30 of the Corporation Law, which provides that "every director must own in his right at least one share of the capital stock of the stock corporation of which he is a director." Any person "who buys stock in a corporation does so with the knowledge that its affairs are dominated by a majority of the stockholders and that he impliedly contracts that the will of the majority shall govern in all matters within the limits of the act of incorporation and lawfully enacted by-laws and not forbidden by law." To this extent, therefore, the stockholder may be considered to have "parted with his personal right or privilege to regulate the disposition of his property which he has invested in the capital stock of the corporation, and surrendered it to the will of the majority of his fellow incorporators. It can not therefore be justly said that the contract, express or implied, between the corporation and the stockholders is infringed by any

act of the former which is authorized by a majority." Pursuant to section 18 of the Corporation Law, any corporation may amend its articles of incorporation by a vote or written assent of the stockholders representing at least two-thirds of the subscribed capital stock of the corporation. If the amendment changes, diminishes or restricts the rights of the existing shareholders, then the dissenting minority has only one right, viz.: "to object thereto in writing and demand payment for his share." Under section 22 of the same law, the owners of the majority of the subscribed capital stock may amend or repeal any by-law or adopt new by-laws. It cannot be said, therefore, that Gokongwei has a vested right to be elected director, in the face of the fact that the law at the time such right as stockholder was acquired contained the prescription that the corporate charter and the by-law shall be subject to amendment, alteration and modification. Issue [2]: Whether the disqualification of a competitor from being elected to the Board of Directors is a reasonable exercise of corporate authority. Held[2]: Although in the strict and technical sense, directors of a private corporation are not regarded as trustees, there cannot be any doubt that their character is that of a fiduciary insofar as the corporation and the stockholders as a body are concerned. As agents entrusted with the management of the corporation for the collective benefit of the stockholders, "they occupy a fiduciary relation, and in this sense the relation is one of trust." "The ordinary trust relationship of directors of a corporation and stockholders is not a matter of statutory or technical law. It springs from the fact that directors have the control and guidance of corporate affairs and property and hence of the property interests of the stockholders. Equity recognizes that stockholders are the proprietors of the corporate

interests and are ultimately the only beneficiaries thereof." A director is a fiduciary. Their powers are powers in trust. He who is in such fiduciary position cannot serve himself first and his cestuis second. He cannot manipulate the affairs of his corporation to their detriment and in disregard of the standards of common decency. He cannot by the intervention of a corporate entity violate the ancient precept against serving two masters. He cannot utilize his inside information and strategic position for his own preferment. He cannot violate rules of fair play by doing indirectly through the corporation what he could not do so directly. He cannot violate rules of fair play by doing indirectly through the corporation what he could not do so directly. He cannot use his power for his personal advantage and to the detriment of the stockholders and creditors no matter how absolute in terms that power may be and no matter how meticulous he is to satisfy technical requirements. For that power is at all times subject to the equitable limitation that it may not be exercised for the aggrandizement, preference, or advantage of the fiduciary to the exclusion or detriment of the cestuis. The doctrine of "corporate opportunity" is precisely a recognition by the courts that the fiduciary standards could not be upheld where the fiduciary was acting for two entities with competing interests. This doctrine rests fundamentally on the unfairness, in particular circumstances, of an officer or director taking advantage of an opportunity for his own personal profit when the interest of the corporation justly calls for protection. It is not denied that a member of the Board of Directors of the San Miguel Corporation has access to sensitive and highly confidential information, such as: (a) marketing strategies and pricing structure; (b) budget for expansion and diversification; (c) research and development; and (d) sources of funding, availability of personnel, proposals of mergers or tieups with other firms. It is obviously to prevent the creation of an opportunity for an officer or director of San Miguel Corporation, who is also the officer or owner of a competing corporation, from taking advantage of the information which he acquires as director to promote his individual or corporate interests to the prejudice of San Miguel Corporation and its

stockholders, that the questioned amendment of the by-laws was made. Certainly, where two corporations are competitive in a substantial sense, it would seem improbable, if not impossible, for the director, if he were to discharge effectively his duty, to satisfy his loyalty to both corporations and place the performance of his corporation duties above his personal concerns. The offer and assurance of Gokongwei that to avoid any possibility of his taking unfair advantage of his position as director of San Miguel Corporation, he would absent himself from meetings at which confidential matters would be discussed, would not detract from the validity and reasonableness of the by-laws involved. Apart from the impractical results that would ensue from such arrangement, it would be inconsistent with Gokongwei's primary motive in running for board membership which is to protect his investments in San Miguel Corporation. More important, such a proposed norm of conduct would be against all accepted principles underlying a director's duty of fidelity to the corporation, for the policy of the law is to encourage and enforce responsible corporate management. Issue [3]: Whether the SEC gravely abused its discretion in denying Gokongwei's request for an examination of the records of San Miguel International, Inc., a fully owned subsidiary of San Miguel Corporation. Held [3]: Pursuant to the second paragraph of section 51 of the Corporation Law, "(t)he record of all business transactions of the corporation and minutes of any meeting shall be open to the inspection of any director, member or stockholder of the corporation at reasonable hours." The stockholder's right of inspection of the corporation's books and records is based upon their ownership of the assets and property of the corporation. It

is, therefore, an incident of ownership of the corporate property, whether this ownership or interest be termed an equitable ownership, a beneficial ownership, or a quasi-ownership. This right is predicated upon the necessity of self-protection. It is generally held by majority of the courts that where the right is granted by statute to the stockholder, it is given to him as such and must be exercised by him with respect to his interest as a stockholder and for some purpose germane thereto or in the interest of the corporation. In other words, the inspection has to be germane to the petitioner's interest as a stockholder, and has to be proper and lawful in character and not inimical to the interest of the corporation. The "general rule that stockholders are entitled to full information as to the management of the corporation and the manner of expenditure of its funds, and to inspection to obtain such information, especially where it appears that the company is being mismanaged or that it is being managed for the personal benefit of officers or directors or certain of the stockholders to the exclusion of others." While the right of a stockholder to examine the books and records of a corporation for a lawful purpose is a matter of law, the right of such stockholder to examine the books and records of a wholly-owned subsidiary of the corporation in which he is a stockholder is a different thing. Stockholders are entitled to inspect the books and records of a corporation in order to investigate the conduct of the management, determine the financial condition of the corporation, and generally take an account of the stewardship of the officers and directors. herein, considering that the foreign subsidiary is wholly owned by San Miguel Corporation and, therefore, under Its control, it would be more in accord with equity, good faith and fair dealing to construe the statutory right of petitioner as stockholder to inspect the books and records of the corporation as extending to books and records of such wholly owned subsidiary which are in the corporation's possession and control.

Issue [4]: Whether the SEC gravely abused its discretion in allowing the stockholders of San Miguel Corporation to ratify the investment of corporate funds in a foreign corporation. Held [4]: Section 17-1/2 of the Corporation Law allows a corporation to "invest its funds in any other corporation or business or for any purpose other than the main purpose for which it was organized" provided that its Board of Directors has been so authorized by the affirmative vote of stockholders holding shares entitling them to exercise at least two-thirds of the voting power. If the investment is made in pursuance of the corporate purpose, it does not need the approval of the stockholders. It is only when the purchase of shares is done solely for investment and not to accomplish the purpose of its incorporation that the vote of approval of the stockholders holding shares entitling them to exercise at least two-thirds of the voting power is necessary. As stated by the corporation, the purchase of beer manufacturing facilities by SMC was an investment in the same business stated as its main purpose in its Articles of Incorporation, which is to manufacture and market beer. It appears that the original investment was made in 1947-1948, when SMC, then San Miguel Brewery, Inc., purchased a beer brewery in Hongkong (Hongkong Brewery & Distillery, Ltd.) for the manufacture and marketing of San Miguel beer thereat. Restructuring of the investment was made in 1970-1971 thru the organization of SMI in Bermuda as a tax free reorganization. Assuming arguendo that the Board of Directors of SMC had no authority to make the assailed investment, there is no question that a corporation, like an individual, may ratify and thereby render binding upon it the originally unauthorized acts of its officers or other agents. This is true because the questioned investment is neither contrary to law, morals, public order or public policy. It is a corporate transaction or contract which is within the corporate powers, but which is

defective from a purported failure to observe in its execution the requirement of the law that the investment must be authorized by the affirmative vote of the stockholders holding two-thirds of the voting power. This requirement is for the benefit of the stockholders. The stockholders for whose benefit the requirement was enacted may, therefore, ratify the investment and its ratification by said stockholders obliterates any defect which it may have had at the outset. Besides, the investment was for the purchase of beer manufacturing and marketing facilities which is apparently relevant to the corporate purpose. The mere fact that the corporation submitted the assailed investment to the stockholders for ratification at the annual meeting of 10 May 1977 cannot be construed as an admission that the corporation had committed an ultra vires act, considering the common practice of corporations of periodically submitting for the ratification of their stockholders the acts of their directors, officers and managers.

ISLAMIC DIRECTORATE OF THE PHILIPPINES VS. COURT OF APPEALS GR 117897 14 May 1997 Facts: Sometime in 1971, Islamic leaders of all Muslim major tribal groups in the Philippines headed by Dean Cesar Adib Majul organized and incorporated the ISLAMIC DIRECTORATE OF THE PHILIPPINES (IDP), the primary purpose of which is to establish an Islamic Center in Quezon City for, the construction of a "Mosque (prayer place, Madrasah (Arabic School), and other religious infrastructures" so as to facilitate the effective practice of Islamic faith in the area. Towards this end, that is, in the same year, the Libyan government donated money to the IDP to purchase land at Culiat, Tandang Sora, Quezon City, to be used as a Center for the Islamic populace. The land, with an area of 49,652 square meters,

we covered by two titles: TCTs RT-26520 (176616) and RT-26521 (170567), both registered in the name of IDP. In 1971, the Board of Trustees of the IDP was composed of Senator Mamintal Tamano, Congressman Ali Dimaporo, Congressman Salipada Pendatun, Dean Cesar Adib Majul, Sultan Harun Al-Rashid Lucman, Delegate Ahmad Alonto, Commissioner Datu Mama Sinsuat and Mayor Aminkadra Abubakar. In 1972, after the purchase of the land by the Libyan government in the name of IDP, Martial Law was declared by the late President Ferdinand Marcos. Most of the members of the 1971 Board of Trustees like Senators Mamintal Tamano, Salipada Pendatun, Ahmad Alonto, and Congressman Al-Rashid Lucman flew to the Middle East to escape political persecution. Thereafter, two Muslim groups sprung, the Carpizo Group, headed by Engineer Farouk Carpizo, and the Abbas Group, led by Mrs. Zorayda Tamano and Atty. Firdaussi Abbas. Both groups claimed to be the legitimate IDP. Significantly, on 3 October 1986, the SEC, in a suit between these two contending groups, came out with a Decision in SEC Case 2687 declaring the election of both the Carpizo Group and the Abbas Group as IDP board members to be null and void. Neither group, however, took the necessary steps prescribed by the SEC in its 3 October 1986 Decision, and no valid election of the members of the Board of Trustees of IDP was ever called. Although the Carpizo Group attempted to submit a set of by-laws, the SEC found that, aside from that Engineer Farouk Carpizo and Atty. Musib Buat, those who prepared and adopted the by-laws were not bona fide members of the IDP, thus rendering the adoption of the by-laws likewise null and void. On 20 April 1989, without having been properly elected as new members of the Board of Trustees of IDP, the Carpizo Group caused to be signed an alleged Board Resolution of the IDP, authorizing the sale of the subject two parcels of land to the Iglesia ni Cristo (INC) for a consideration of P22,343,400.00, which sale was evidenced by a Deed of Absolute Sale 12 dated 20 April 1989. On 30 May 1991, the 1971 IDP Board of Trustees headed by former Senator Mamintal Tamano, or the Tamano Group, filed a petition before the SEC (SEC Case 4012) seeking to declare null and void

the Deed of Absolute Sale signed by the Carpizo Group and the INC since the group of Engineer Carpizo was not the legitimate Board of Trustees of the IDP. Meanwhile, INC, pursuant to the Deed of Absolute Sale executed in its favor, filed an action for Specific Performance with Damages against the vendor, Carpizo Group, before Branch 81 of the Regional Trial Court of Quezon City (Civil Case Q-90-6937) to compel said group to clear the property of squatters and deliver complete and full physical possession thereof to INC. Likewise, INC filed a motion in the same case to compel one Mrs. Leticia P. Ligon to produce and surrender to the Register of Deeds of Quezon City the owner's duplicate copy of TCTs RT-26521 and RT26520 covering the two parcels of land, so that the sale in INC's favor may be registered and new titles issued in the name of INC. Mrs. Ligon was alleged to be the mortgagee of the two parcels of land executed in her favor by certain Abdulrahman R.T. Linzag and Rowaida Busran-Sampaco claimed to be in behalf of the Carpizo Group. Judge Celia Lipana-Reyes of Branch 81, Regional Trial Court of Quezon City, denied IDP's motion to intervene on the ground of lack of juridical personality of the IDP-Tamano Group and that the issues being raised by way of intervention are intracorporate in nature, jurisdiction thereto properly pertaining to the SEC. Apprised of the pendency of SEC Case 4012 involving the controverted status of the IDP-Carpizo Group but without waiting for the outcome of said case, Judge Reyes, on 12 September 1991, rendered Partial Judgment in Civil Case Q-90-6937 ordering the IDP-Carpizo Group to comply with its obligation under the Deed of Sale of clearing the subject lots of squatters and of delivering the actual possession thereof to INC. Thereupon Judge Reyes in another Order, dated 2 March 1992, pertaining also to Civil Case Q-90-6937, treated INC as the rightful owner of the real properties and disposed. On 6 April 1992, the Order was amended by Judge Reyes directing Ligon "to deliver the owner's duplicate copies of TCT Nos. RT26521 (170567) and RT-26520 (176616) to the Register of Deeds of Quezon City for the purposes stated in the Order of March 2, 1992." Mortgagee Ligon went to the Court of Appeals, thru a petition for certiorari

(CA-GR SP-27973), assailing the Orders of Judge Reyes. The appellate court dismissed her petition on 28 October 1992. Undaunted, Ligon filed a petition for review before the Supreme Court (GR 107751). In the meantime, the SEC, on 5 July 1993, finally came out with a Decision in SEC Case 4012, Declaring the by-laws submitted by the IDP-Caprizo group as unauthorized, and hence, null and void; declaring the sale of the two (2) parcels of land in Quezon City covered by the Deed of Absolute Sale entered into by Iglesia ni Kristo and the Islamic Directorate of the Philippines, Inc. null and void; declaring the election of the Board of Directors 23 of the corporation from 1986 to 1991 as null and void; and Declaring the acceptance of the respondents, except Farouk Carpizo and Musnib Buat, as members of the IDP null and void. The INC filed a Motion for Intervention, dated 7 September 1993, in SEC Case 4012, but the same was denied on account of the fact that the decision of the case had become final and executory, no appeal having been taken therefrom. INC elevated SEC Case 4012 to the Court of Appeals by way of a special civil action for certiorari (CA-GR SP 33295). On 28 October 1994, the appeallate court promulgated a Decision granting INC's petition. The portion of the SEC Decision in SEC Case 4012 which declared the sale of the two (2) lots in question to INC as void was ordered set aside by the Court of Appeals. Thus, the IDP-Tamano Group brought the petition for review, dated 21 December 1994, to the Supreme Court. While the petition was pending, however, the Supreme Court rendered judgment in GR 107751 on the petition filed by Mrs. Leticia P. Ligon. The Decision, dated 1 June 1995, denied the Ligon petition and affirmed the 28 October 1992 Decision of the Court of Appeals in CA-GR SP-27973 which sustained the Order of Judge Reyes compelling mortgagee Ligon to surrender the owner's duplicate copies of TCTs RT-26521 (170567) and RT-26520 (176616) to the Register of Deeds of Quezon City so that the Deed of Absolute Sale in INC's favor may be properly registered. Issue:

Whether the Tandang Sora property was legitimately sold to the INC. Held: As far back as 3 October 1986, the SEC, in Case 2687, in a suit between the Carpizo Group and the Abbas Group, already declared the election of the Carpizo Group (as well as the Abbas Group) to the IDP Board as null and void for being violative of the Articles of Incorporation. Nothing thus becomes more settled than that the IDP-Carpizo Group with whom INC contracted is a fake Board. Premises considered, all acts carried out by the Carpizo Board, particularly the sale of the Tandang Sora property, allegedly in the name of the IDP, have to be struck down for having been done without the consent of the IDP thru a legitimate Board of Trustees. Article 1318 of the New Civil Code lays down the essential requisites of contracts, and where all these elements must be present to constitute a valid contract. For, where even one is absent, the contract is void. Specifically, consent is essential for the existence of a contract, and where it is wanting, the contract is non-existent. Herein, the IDP, owner of the subject parcels of land, never gave its consent, thru a legitimate Board of Trustees, to the disputed Deed of Absolute Sale executed in favor of INC. This is, therefore, a case not only of vitiated consent, but one where consent on the part of one of the supposed contracting parties is totally wanting. Ineluctably, the subject sale is void and produces no effect whatsoever. The Carpizo Group-INC sale is further deemed null and void ab initio because of the Carpizo Group's failure to comply with Section 40 of the Corporation Code pertaining to the disposition of all or substantially all assets of the corporation. The Tandang Sora property, it appears from the records, constitutes the only property of the IDP. Hence, its sale to a thirdparty is a sale or disposition of all the corporate property and assets of IDP falling squarely within the contemplation of the foregoing section. For the sale to be valid, the majority vote of the legitimate Board of Trustees, concurred in by the vote of at least 2/3 of the bona fide members of the

corporation should have been obtained. These twin requirements were no met as the Carpizo Group which voted to sell the Tandang Sora property was a fake Board of Trustees, and those whose names and signatures were affixed by the Carpizo Group together with the sham Board Resolution authorizing the negotiation for the sale were, from all indications, not bona fide members of the IDP as they were made to appear to be. Apparently, there are only 15 official members of the IDP including the 8 members of the Board of Trustees. All told, the disputed Deed of Absolute Sale executed by the fake Carpizo Board and INC was intrinsically void ab initio.

LIABILITY OF CORPORATION PEPSI-COLA DISTRIBUTORS VS. NLRC G.R. No. 100686 Facts: This case involves a maintenance electrician (private respondent Yute), an employee of petitioner PCD, who was dismissed from his employment on the alleged ground of abandonment and/or absence without leave. When the private respondent filed with the Labor Arbiter a case for illegal dismissal, the latter rendered a favorable decision to Yute and against his employer. He was reinstated and included in the payroll from May 22, 1989 pending PDC's appeal with the NLRC, only to be dismissed again on July 24, 1989 on the alleged ground that his employer, PCD, sold its business interest to PCPPI which, however, denied liability on the ground that it is a new entity separate and distinct from PCD. The NLRC rendered judgment in favour of private respondent. It is the contention of petitioner PCD that the dismissal of private respondent was premised on a just cause after affording him due process

because as early as the first two years of his employment, he was twice reprimanded for being absent without permission and when he was required to explain his absenc, he failed to appear before the administrative committee despite personal service of notice which he refused to sign. While petitioner asserts that the second dismissal of private respondent was due to closure of PCD as a result of business losses, it however argues that public respondent NLRC gravely abused its discretion when it assumed jurisdiction and ruled on the validity of the second dismissal. Petitioner maintains that its right to due process of law was violated considering that there was no formal complaint as regard's the second dismissal and no hearing was ever conducted to enable petitioner PCD to present evidence on an issue which is separate and distinct from the first dismissal. Corollary to the argument on violation of its right to due process of law, petitioner PCD further contends that Pepsi Cola Products Philippines, Inc. (PCPPI), a corporation separate and distinct from PCDPI, should not be held liable for reinstatement with backwages of private respondent since it is not a party to this case. Issue: Held: Pepsi Cola Distributors of the Philippines may have ceased business operations and Pepsi-Cola Products Philippines, Inc. may be a new company but it does not necessarily follow that no one may now be held liable for illegal acts committed by the earlier firm. The complaint was filed when PCD was still in existence. Pepsi-Cola never stopped doing business in the Philippines. The same soft drinks products sold when the complaint was initiated continue to be sold now. The sale of products, purchases of materials, payment of obligations, and other business acts did not stop at the time PCD bowed out and PCPPI came into being. There is no evidence presented showing that PCPPI, as the new entity or purchasing company is free from any liabilities incurred by the former corporation. Who may be held liable for illegal acts committed by the PCD?

DR. ALFAFARA VS. ACEBEDO OPTICAL G.R. No. 148384 Facts: Petitioners were duly licensed optometrists and were all members of the Samahan ng Optometrists ng Pilipinas (SOP)-Cebu Chapter. The SOPCebu Chapter was a chapter of SOP Incorporated, a national organization which had a program called Sight Saving Month. This program is implemented nationwide which provided free consultations. Respondent was a corporation with several outlets in Cebu, selling optical products and ready-to-wear eyeglasses of limited grades, advertising its services and products. Respondent hired optometrists who conducted eye examinations, prescribed ophthalmic lenses, and rendered other optometry services. While the hired optometrists received their salary from respondent, they are not precluded from seeking other sources of income. Petitioners brought an injunctive suit in the Regional Trial Court to enjoin respondent Acebedo from practicing optometry in the province of Cebu on the main ground of violation of the Code of Ethics for Optometrists. In addition, petitioners contend that respondent is engaged in the practice of optometry by its act of employing licensed agents. Thus, an optometrist who is employed by a corporation, such as Acebedo, is not acting on his own capacity but as an employee or agent of the corporation. They contend that, as a mere employee or agent, such optometrist cannot be held personally liable for his acts done in the course of his employment as an optometrist under the provisions of the Civil Code. Respondent denies being engaged in the practice of optometry and argues that incidental to its business of selling optical products, it hired duly

licensed optometrists who conducted eye examination, prescribed ophthalmic lenses, and rendered other services; that it exercised neither control nor supervision over the optometrists under its employ; and that the hired optometrists exercised neither control nor supervision in the sale of optical products and accessories by respondent. The trial court at first dismissed the suit but, on motion of petitioners, reinstated the action and granted their prayer for a writ of preliminary injunction and/or restraining order. After hearing, judgment was rendered in favor of petitioners. The Court of Appeals reversed the trial court holding that respondent was illegally engaged in the practice of Optometry. Issue: Whether or not respondent corporation Acebedo was engaged in the practice of optometry and is thus liable for the acts of its optometrist employees. Held: An optometrist is a person who has been certified by the Board of Optometry and registered with the Professional Regulation Commission as qualified to practice optometry in the Philippines. Thus, only natural persons can engage in the practice of optometry and not corporations. Respondent, which is not a natural person, cannot take the licensure examinations for optometrist and, therefore, it cannot be registered as an optometrist under R.A. No. 1998. There is no reason to deviate from the ruling that a duly licensed optometrist is not prohibited from being employed by respondent and that respondent cannot be said to be exercising the optometry profession by reason of such employment. While the optometrists are employees of respondent, their practice of optometry is separate and distinct from the business of respondent of selling optical products. They are personally liable for acts done in the

course of their practice in the same way that if respondent is sued in court in connection with its business of selling optical products, the optometrists need not be impleaded as party defendants. In that regard, the Board of Optometry and the Professional Regulation Commission regulate their practice and have exclusive original jurisdiction over them.

already the corporate property of FLADC for which the Tius were not entitled to the issuance of new shares of stock. Issue: Whether the Tuis were correct in choosing rescission as remedy. Held:

ONG YONG vs. DAVID S. TIU G.R. No. 144476 April 8, 2003 Facts: The construction of the Masagana Citimall was threatened with stoppage and incompletion when its owner, the First Landlink Asia Development Corporation (FLADC), which was owned by the Tius, encountered dire financial difficulties. To prevent foreclosure of the mortgage on the two lots where the mall was being built, the Tius invited the Ongs, to invest in FLADC through a Pre-Subscription Agrrement. Tius committed to contribute a four-storey building and two parcels of land to cover their additional stock subscription. The Tuis subsequently rescinded the Pre-Subscription Agreement.and alleged that the Ongs refused to give them the shares corresponding to their property contributions of a four-story building, a 1,902.30 square-meter lot and a 151 square-meter lot. Hence, they felt they were justified in setting aside their Pre-Subscription Agreement with the Ongs who allegedly refused to comply with their undertakings. The Ongs later on discovered that FLADC had in reality owned the property all along, even before their Pre-Subscription Agreement was executed. This meant that the 151 square-meter property was at that time

The parties' Pre-Subscription Agreement was in fact a subscription contract as defined under Section 60, Title VII of the Corporation Code: Any contract for the acquisition of unissued stock in an existing corporation or a corporation still to be formed shall be deemed a subscription within the meaning of this Title, notwithstanding the fact that the parties refer to it as a purchase or some other contract. A subscription contract necessarily involves the corporation as one of the contracting parties since the subject matter of the transaction is property owned by the corporation its shares of stock. Considering therefore that the real contracting parties to the subscription agreement were FLADC and the Ongs alone, a civil case for rescission on the ground of breach of contract filed by the Tius in their personal capacities will not prosper. Assuming it had valid reasons to do so, only FLADC had the legal personality to file suit rescinding the subscription agreement with the Ongs inasmuch as it was the real party in interest therein. Granting that the Tius possess the legal standing to sue for rescission based on breach of contract, said action will still not prosper since rescission will violate the Trust Fund Doctrine and the procedures for the valid distribution of assets and property under the Corporation Code. In the instant case, the rescission of the Pre-Subscription Agreement will effectively result in the unauthorized distribution of the capital assets and

property of the corporation, thereby violating the Trust Fund Doctrine and the Corporation Code, since rescission of a subscription agreement is not one of the instances when distribution of capital assets and property of the corporation is allowed.

PHILIPPINE NATIONAL BANK & NATIONAL SUGAR DEVELOPMENT CORPORATION VS. ANDRADA ELECTRIC & ENGINEERING COMPANY G.R. No. 142936 April 17, 2002 Facts: Pampanga Sugar Mills (PASUMIL) hired the services of respondent, since the latter is engaged in the business of general construction for repairs of buildings and machineries. The Development Bank of the Philippines (DBP) has foreclosed the assets of PASUMIL. At the foreclosure sale, Philippine National Bank (PNB) was the highest bidder, hence it acquired the assets of PASUMIL. PNB organized the National Sugar Development Corporation (NASUDECO) to take ownership and possession of the assets of the PASUMIL. The latter failed to pay the balance of its total obligation to the respondent when PASUMIL hired the services of the respondent. Consequently, the respondent filed a case against PNB, NASUDECO and PASUMIL. Issue: Whether or not PNB is liable for corporate debts. Held: As a rule, a corporation that purchases the assets of another will not be liable for the debts of the selling corporation, provided the former acted in good faith and paid adequate consideration for such assets, except when any of the following circumstances is present: (1) where the purchaser expressly or impliedly agrees to assume the debts, (2) where the transaction

amounts to a consolidation or merger of the corporations, (3) where the purchasing corporation is merely a continuation of the selling corporation, and (4) where the transaction is fraudulently entered into in order to escape liability for those debts. While we agree with respondents claim that the assets of the National Sugar Development Corporation (NASUDECO) can be easily traced to PASUMIL, we are not convinced that the transfer of the latters assets to petitioners was fraudulently entered into in order to escape liability for its debt to respondent. A careful review of the records reveals that DBP foreclosed the mortgage executed by PASUMIL and acquired the assets as the highest bidder at the public auction conducted. The bank was justified in foreclosing the mortgage, because the PASUMIL account had incurred arrearages of more than 20 percent of the total outstanding obligation. Thus, DBP had not only a right, but also a duty under the law to foreclose the subject properties.

MANUEL R. DULAY ENTERPRISES, INC., VIRGILIO E. DULAY AND NEPOMUCENO REDOVAN VS. COURT OF APPEALS, EDGARDO D. PABALAN, MANUEL A. TORRES, JR., MARIA THERESA V. VELOSO AND CASTRENSE C. VELOSO G.R. No. 91889 August 27, 1993 225 SCRA 678 FACTS: Manuel Dulay Enterprises, Inc. (Dulay Corporation, for brevity) obtained several loans for the construction of its hotel project. Dulay Corporation borrowed money from Atty. Virgilio Dulay, its vice-president for the continuance of said project. As a result of the said loan, Virgilio occupied one of the unit apartments of the subject property. Thereafter, by virtue of a board resolution, Dulay Corporation, through its president Manuel Dulay, sold the subject property in favor of spouses Maria Theresa

and Castrense Veloso in the amount of P300,000.00 as evidenced by the Deed of Absolute Sale. The parties executed a Memorandum to the Absolute Deed of Sale giving Manuel Dulay two years within which to repurchase the property from Spouses Veloso. However, a day after the property was sold to the spouses, the spouses, in order to obtain a loan, mortgaged the same to Manuel Torres without the knowledge of Manuel Dulay. By reason of the failure of Spouses Veloso to pay Torres, the said property was sold to the latter, as the highest bidder in an extrajudicial foreclosure sale. Maria Veloso executed an Absolute Deed of Assignment of the Right to Redeem in favor Manuel Dulay, but neither of them was able to redeem the property within one year. As a result, Torres filed an action against Dulay Corporation for the recovery of possession. Dulay Corporation, in turn, filed an action for the cancellation of the Certificate of Sheriffs Sale against Spouses Veloso and Torres. Issue: Whether or not the sale of the subject property by its president, Manuel Dulay in favor of spouses Veloso is null and void as the alleged Board Resolution was passed without the knowledge and consent of the other members of the board of directors Held: Consequently, petitioner corporation is liable for the act of Manuel Dulay and the sale of the subject property to private respondents by Manuel Dulay is valid and binding. Petitioners' contention that private respondent Torres never acquired ownership over the subject property since the latter was never in actual possession of the subject property nor was the property ever delivered to him is also without merit.

Paragraph 1, Article 1498 of the New Civil Code provides: When the sale is made through a public instrument, the execution thereof shall be equivalent to the delivery of the thing which is the object of the contract, if from the deed the contrary do not appear or cannot clearly be inferred. Under the aforementioned article, the mere execution of the deed of sale in a public document is equivalent to the delivery of the property.Therefore, prior physical delivery or possession is not legally required since the execution of the Deed of Sale in deemed equivalent to delivery.

SIA VS. PEOPLE Facts: Accused Jose 0. Sia sometime prior to 24 May, 1963, was General Manager of the Metal Manufacturing Company of the Philippines, Inc. engaged in the manufacture of steel office equipment; on 31 May, 1963, because his company was in need of raw materials to be imported from abroad, he applied for a letter of credit to import steel sheets from Mitsui Bussan Kaisha, Ltd. of Tokyo, Japan, the application being directed to the Continental Bank, herein complainant, Exhibit B and his application having been approved, the letter of credit was opened on 5 June, 1963 in the amount of $18,300, Exhibit D; and the goods arrived sometime in July, 1963 according to accused himself, tsn. II:7; now from here on there is some debate on the evidence; according to Complainant Bank, there was permitted delivery of the steel sheets only upon execution of a trust receipt, Exhibit A; while according to the accused, the goods were delivered to him sometime before he executed that trust receipt in fact they had already been converted into steel office equipment by the time he signed said trust receipt, tsn. II:8; but there is no question - and this is not debated - that the bill of exchange issued for the purpose of collecting the unpaid account thereon having fallen due (see Exh. B) neither accused nor his company

having made payment thereon notwithstanding demands, Exh. C and C-1, dated 17 and 27 December, 1963, and the accounts having reached the sum in pesos of P46,818.68 after deducting his deposit valued at P28,736.47; that was the reason why upon complaint by Continental Bank, the Fiscal filed the information after preliminary investigation as has been said on 22 October, 1964. Issue: Whether petitioner Jose O. Sia, having only acted for and in behalf of the Metal Manufacturing Company of the Philippines (Metal Company, for short) as President thereof in dealing with the complainant, the Continental Bank, (Bank for short) may be liable for the crime charged. Held: The case cited by the Court of Appeals in support of its stand-Tan Boon Kong case, supra-may however not be squarely applicable to the instant case in that the corporation was directly required by law to do an act in a given manner, and the same law makes the person who fails to perform the act in the prescribed manner expressly liable criminally. The performance of the act is an obligation directly imposed by the law on the corporation. Since it is a responsible officer or officers of the corporation who actually perform the act for the corporation, they must of necessity be the ones to assume the criminal liability; otherwise this liability as created by the law would be illusory, and the deterrent effect of the law, negated. In the present case, a distinction is to be found with the Tan Boon Kong case in that the act alleged to be a crime is not in the performance of an act directly ordained by law to be performed by the corporation. The act is imposed by agreement of parties, as a practice observed in the usual pursuit of a business or a commercial transaction. The offense may arise, if at all, from the peculiar terms and condition agreed upon by the parties to the transaction, not by direct provision of the law. The intention of the parties, therefore, is a factor determinant of whether a crime was committed or

whether a civil obligation alone intended by the parties. With this explanation, the distinction adverted to between the Tan Boon Kong case and the case at bar should come out clear and meaningful. In the absence of an express provision of law making the petitioner liable for the criminal offense committed by the corporation of which he is a president as in fact there is no such provisions in the Revised Penal Code under which petitioner is being prosecuted, the existence of a criminal liability on his part may not be said to be beyond any doubt. In all criminal prosecutions, the existence of criminal liability for which the accused is made answerable must be clear and certain. The maxim that all doubts must be resolved in favor of the accused is always of compelling force in the prosecution of offenses. This Court has thus far not ruled on the criminal liability of an officer of a corporation signing in behalf of said corporation a trust receipt of the same nature as that involved herein. In the case of Samo vs. People, L-17603-04, May 31, 1962, the accused was not clearly shown to be acting other than in his own behalf, not in behalf of a corporation.

THE BOARD OF DIRECTORS GRACE CHRISTIAN HIGH SCHOOL vs. COURT OF APPEALS, GRACE VILLAGE ASSOCIATION, INC., ALEJANDRO G. BELTRAN, and ERNESTO L. GO G.R. No. 108905 October 23, 1997 281 SCRA 133 Facts: Grace Christian High School (Grace School, for brevity) is an educational institution located in Quezon City while Grace Village Association, Inc. (Grace Association, for short) on the other hand, is an organization of lot and/or building owners, lessees and residents at Grace

Village, while private respondents Alejandro G. Beltran and Ernesto L. Go were its president and chairman of the committee on election, respectively, in 1990, when this suit was brought. Sometime in 1968, the by-laws of Grace Association provided that the Board of Directors were composed of 11 members to serve for 1 year until their successors are duly elected. However, in 1975, the board of directors prepared a draft of an amendment to the by-laws, which granted Grace School representative to be a permanent director of Grace Association.The said draft was never presented to the general membership for approval but it was presumably submitted to the board. Until the year 1990, Grace School was given a permanent seat in the board of directors of the association. However, on February 13, 1990, Grace Associations committed on election informed the principal of Grace School that to make an entity a permanent Director would deprive the right of the voters to vote for 15 members of the Board and it is undemocratic for an entity to hold office in perpetuity. Grace School made a request on the committee on election that the latter should follow the procedures in the previous elections claiming that the notice issued for the 1990 elections will run counter to the practice in the previous years and was in violation of the by-laws of 1975. It further claimed that the proposal of the committee on election would unlawfully deprive Grace School of its vested right to a permanent seat in the Board. As a result of the denial of Grace Association of its request, Grace School brought suit for mandamus in the Home Insurance and Guaranty Corporation to compel the board of directors of the association to recognize its right to a permanent seat in the board. The officer of the Home Insurance and Guaranty Corporation dismissed the action filed by Grace school stating among others that the by-

laws of 1975 was merely a proposal although implemented in the past, had not yet been ratified by the members of the association nor approved by competent authority. In 1990, the Board of Directors of Grace Association declared the said proposed by-laws as null and void. Issue: Whether or not Grace School has acquired a vested right to a permanent seat in the Board of Directors. Held: The present Corporation Code (B.P. Blg. 68), which took effect on May 1, 1980, similarly provides: 23. The Board of Directors or Trustees. Unless otherwise provided in this Code, the corporate powers of all corporations formed under this Code shall be exercised, all business conducted and all property of such corporations controlled and held by the board of directors or trustees to be elected from among the holders of stocks, or where there is no stock, from among the members of the corporation, who shall hold office for one (1) year and until their successors are elected and qualified. These provisions of the former and present corporation law leave no room for doubt as to their meaning: the board of directors of corporations must be elected from among the stockholders or members. There may be corporations in which there are unelected members in the board but it is clear that in the examples cited by petitioner the unelected members sit as ex officio members, i.e., by virtue of and for as long as they hold a particular office. But in the case of petitioner, there is no reason at all for its representative to be given a seat in the board. Nor does petitioner claim a right to such seat by virtue of an office held. In fact it was not given such seat in the beginning. It was only in 1975 that a proposed amendment to the by-laws sought to give it one.

Since the provision in question is contrary to law, the fact that for fifteen years it has not been questioned or challenged but, on the contrary, appears to have been implemented by the members of the association cannot forestall a later challenge to its validity. Neither can it attain validity through acquiescence because, if it is contrary to law, it is beyond the power of the members of the association to waive its invalidity. For that matter the members of the association may have formally adopted the provision in question, but their action would be of no avail because no provision of the by-laws can be adopted if it is contrary to law. It is probable that, in allowing petitioners representative to sit on the board, the members of the association were not aware that this was contrary to law. It should be noted that they did not actually implement the provision in question except perhaps insofar as it increased the number of directors from 11 to 15, but certainly not the allowance of petitioners representative as an unelected member of the board of directors. It is more accurate to say that the members merely tolerated petitioners representative and tolerance cannot be considered ratification. Nor can petitioner claim a vested right to sit in the board on the basis of practice. Practice, no matter how long continued, cannot give rise to any vested right if it is contrary to law. Even less tenable is petitioners claim that its right is coterminus with the existence of the association.

Mid-Pasig Land Development Corporation, represented by its Chairman and President, Ronaldo Salonga, and ECRM Enterprises, represented by its proprietor, Mario P. Tablante, executed a lease agreement over a parcel of land. When the lease agreement expired, Tablante assigned all his rights and interests under the said agreement to Litam and/or Rockland Construction Company, Inc. under a Deed of Assignment. MidPasig Land Development Corporation later on discovered that Tablante had executed a Contract of Lease with respondent MC Home Depot, Inc. over the same parcel of land and consequently demanded that MC Home Depot vacate the land. When the matter reached the Court of Appeals via petition for certiorari, the appellate court dismissed the petition because the verification and certification against non-forum shopping was signed by Antonio A. Merelos as General Manager of the petitioner-corporation without attaching therewith a Corporate Secretarys certificate or board resolution that he is authorized to sign for and on behalf of the petitioner.

Issue: Whether or not a verification and certification by a General Manager requires a board resolution and/or a secretarys certificate for its validity and admissibility Held:

MID-PASIG LAND DEVELOPMENT CORPORATION vs. MARIO TABLANTE G.R. No. 162924 February 4, 2010 Facts:

Sec. 23, in relation to Sec. 25 of the Corporation Code, clearly enunciates that all corporate powers are exercised, all business conducted, and all properties controlled by the board of directors. A corporation has a separate and distinct personality from its directors and officers and can only exercise its corporate powers through the board of directors. Thus, it is clear that an individual corporate officer cannot solely exercise any corporate

power pertaining to the corporation without authority from the board of directors. However, the Supreme Court has recognized the authority of the following corporate officers to sign the verification and certification against forum shopping without need of a board resolution: (1) the Chairperson of the Board of Directors, (2) the President of a corporation, (3) the General Manager or Acting General Manager, (4) Personnel Officer, and (5) an Employment Specialist in a labor case. The determination of the sufficiency of the authority was done on a case to case basis. This is to justify the authority of corporate officers or representatives of the corporation to sign the verification or certificate against forum shopping, being in a position to verify the truthfulness and correctness of the allegations in the petition. From the foregoing, it is thus clear that the failure to attach the Secretarys Certificate, attesting to General Manager Antonio Mereloss authority to sign the Verification and Certification of Non-Forum Shopping, should not be considered fatal to the filing of the petition. Nonetheless, the requisite board resolution was subsequently submitted to the CA, together with the pertinent documents.

St. Marys Farm, Inc. was the registered owner of an originally twenty-five thousand five hundred ninety-eight (25,598) square meters of land at, Las Pias City. In compliance with a final court decision of the RTC in a civil case, St. Marys Farm Inc. passed and approved a board resolution authorizing defendant Agana to cede to T.S. Cruz Subdivision four thousand (4,000) square meters of the land. However, Agana then did not return to St. Mary the borrowed title and forged a board resolution of the corporation making it appear that he was authorized to sell the remaining twenty-one thousand five hundred ninety-eight (21,598) square meters of the subject property. Agana and Prima Real Properties, Inc. later on signed an absolute deed of sale, with Prima as the vendee. Prima then caused the transfer of the title in its name. In its complaint, St. Mary alleged that the authorization certified by its Corporate Secretary and used by Agana in selling the subject property to defendant Prima was a forged and that Prima wasnt a buyer in good faith. Issue: Whether or not a Corporation is liable for the acts of its agent and must necessarily bear whatever damage may have been caused by this alleged breach of trust to buyer in good faith and for value, who relied on the authority of a notarized board resolution Held: A buyer in good faith has every reason to rely on a persons authority to sell a particular property owned by a corporation on the basis of a notarized board resolution. Being a buyer in good faith, he buys the property with the well-founded belief that the person from whom he receives the thing had title to the property and capacity to convey it. Agana presented to Prima:

AUTHORITY OF AN OFFICER TO ACT ON BEHALF OF THE CORPORATION ST. MARYS FARM, INC., VS. PRIMA REAL PROPERTIES, INC.ET.AL. G.R. No. 158144 July 31, 2008 Facts:

1.

2.

3.

A notarized board resolution which stated that at a special meeting held on June 27, 1988 the board of directors authorized Agana, Treasurer, to sell the subject property; A separate Certification by the St. Marys president, authorizing its Treasurer, Agana, to sell said property; and The title of the subject property.

When a document under scrutiny is a special power of attorney that is duly notarized, acknowledgement is prima facie evidence of the fact of its due execution a buyer presented with such document would have no choice between knowing and finding out whether a forger lurks beneath the signature on it. A person dealing with a seller who has [in his] possession title to the property but whose capacity to sell is restricted, qualifies as a buyer in good faith if he proves that he inquired into the title of the seller as well as into the latter's capacity to sell; and that in his inquiry, he relied on the notarial acknowledgment found in the seller's duly notarized special power of attorney. He need not prove anything more for it is already the function of the notarial acknowledgment to establish the appearance of the parties to the document, its due execution and authenticity. The board resolution further averred that he was "authorized and empowered to sign any and all documents, instruments, papers or writings which may be required and necessary for this purpose to bind the Corporation in this undertaking." The certification of St. Marys President also attests to this fact. With this notarized board resolution, respondent Agana, undeniably, had the authority to cede the subject property, carrying with it all the concomitant powers necessary to implement said transaction.

ESPIRITU JR. et. al vs. PETRON CORPORATION and CARMEN J. DOLOIRAS, doing business under the name "KRISTINA PATRICIA ENTERPRISES (KPE)" GR No. 170891 November 24, 200 Facts: KPE and Bicol Gas Refilling Plant Corporation (Bicol Gas) were in the business of selling and distributing LPGs in Sorsogon. The former was owned and operated by Carmen Dolorais but was manged by Jose Dolorais. The latter carried the trademark "Bicol Savers Gas" managed by Llona. In the course of trade and competition, any given distributor of LPGs at times acquired possession of LPG cylinder tanks belonging to other distributors operating in the same area. They called these "captured cylinders." In one occasion, Jose saw a particular Bicol Gas truck on the Maharlika Highway. While the truck carried mostly Bicol Savers LPG tanks, it had on it one unsealed 50-kg Gasul tank and one 50-kg Shellane tank. Jose followed the truck and when it stopped at a store, he asked the driverand the Bicol Gas sales representative about the Gasul tank in their truck who later admitted that the Gasul and Shellane tanks on their truck belonged to a customer who had them filled up by Bicol Gas. KPE filed a complaint for violations of Republic Act (R.A.) 623 (illegally filling up registered cylinder tanks), as amended, and Sections 155 (infringement of trade marks) and 169.1 (unfair competition) of the Intellectual Property Code (R.A. 8293) against the directors, officers, and stockholders of Bicol Gas. Issue:

Whether or not the stockholders and members of the board of directors of Bicol Gas are liable with respect to the charge of unlawfully filling up a steel cylinder or tank that belonged to Petron

SAN MIGUEL BUKID HOMEOWNERS ASSOCIATION, INC. vs. THE CITY OF MANDALUYONG G.R. No. 153653 October 2, 2009 Facts: San Miguel Bukid Homeowners Association, Inc. entered into a Memorandum of Agreement with the City of Mandaluyong whereby the City purchased lots and then transferred the same to petitioner with a first real estate mortgage in favor of the City. When the work on the project stopped, petitioner filed the complaint praying that the City and its contractor be ordered to perform their respective undertakings and obligations under the Contract Agreement and to pay petitioner attorneys fees, exemplary damages and litigation expenses. The City filed an Answer within the extended period granted by the trial court. When the trial court dismissed petitioners petition to declare the City in default, the matter was elevated by petitioner to the Court of Appeals via a petition for certiorari. However Court of Appeals dismissed the petition because the person who signed the Verification/Certification of Non-Forum Shopping thereof did not appear to be authorized by San Miguel Bukid Homeowners Association, Inc. Issue: Whether or not a Board Resolution authorizing an officer to initiate, sign, file and prosecute a complaint sufficient for a special civil action of certiorari Held: Certiorari is an original and independent action that is not part of the trial or the proceedings on the complaint filed before the trial court

Held: Corporate officers or employees through whose act, default or omission the corporation commits a crime may themselves be answerable for the crime. This is an exception to the general rule that a corporation has entity separate and distinct from the persons of its officers, directors, and stockholders. Before a stockholder may be held criminally liable for acts committed by a corporation, it must be shown that he had knowledge of the criminal act committed in the name of the corporation and that he took part in the same or gave his consent to its commission whether by action or inaction. The "owners" of a corporate organization are its stockholders and they are to be distinguished from its directors and officers. The petitioners here, with the exception of Audie Llona, are being charged in their capacities as stockholders of Bicol Gas. But the Court of Appeals forgets that in a corporation, the management of its business is generally vested in its board of directors, not its stockholders. Stockholders are basically investors in a corporation. They do not have a hand in running the day-today business operations of the corporation unless they are at the same time directors or officers of the corporation. Before a stockholder may be held criminally liable for acts committed by the corporation, therefore, it must be shown that he had knowledge of the criminal act committed in the name of the corporation and that he took part in the same or gave his consent to its commission, whether by action or inaction.

hence, a Board Resolution authorizing an officer to initiate, sign, file and prosecute the complaint is not and cannot be considered as an authorization to initiate, sign, file, and prosecute another case for the special civil action of certiorari. Evidently, petitioner only authorized its President, Evelio Barata, to initiate, sign, file and prosecute the Complaint for specific performance. Certiorari, as a special civil action, is an original action invoking the original jurisdiction of a court to annul or modify the proceedings of a tribunal, board or officer exercising judicial or quasi-judicial functions. It is an original and independent action that is not part of the trial or the proceedings on the complaint filed before the trial court. The petition for certiorari before the CA is, therefore, a separate and distinct action from the action for specific performance instituted before the RTC, as the writ of certiorari being prayed for is directed against the judicial or quasi-judicial body, not against the private parties in the original action for specific performance. Such being the case, the Resolution of the Board of Directors of petitioner association is not and cannot be considered as an authorization for its President, Evelio Barata, to initiate, sign, file and prosecute another case for the special civil action of certiorari. The submission of a Secretarys Certificate with the Motion for Reconsideration is also insufficient to cure the initial defect because there was no certification as to when petitioners Board of Trustees originally granted Mr. Barata authority to show that as of the date of the filing of the petition for certiorari, Mr. Barata had been authorized to perform such acts.

LIABILITY OF BOARD OF DIRECTORS FLIGHT ATTENDANTS AND STEWARDS ASSOCIATION OF THE PHILIPPINES VS. PHILIPPINE AIRLINES, PATRICIA CHIONG AND COURT OF APPEALS G.R. No. 178083 July 23, 2008 569 SCRA 252 Facts: The Flight Attendants and Stewards Association of the Philippines (FASAP, for brevity) is the duly certified collective bargaining representative of PALs flight attendants and stewards, while PAL, is a domestic corporation operating as a common carrier transporting passengers and cargo through aircraft. PAL adopted the retrenchment scheme due to its alleged financial losses and as a result of the adoption of such retrenchment scheme, PAL retrenched 5,000 of its employees and a thousand of its cabin crew personnel. During the said period, PAL was placed under corporate rehabilitation and a rehabilitation plan was approved by the Securities and Exhange Commission (SEC, for short). Thereafter, PAL, through its Chairman and C.E.O, Lucio Tan, made an offer to transfer shares of stock to its employees and three seats in its Board of Directors, on the condition that all the existing Collective Bargaining Agreements (CBAs) with its employees would be suspended for 10 years, however, the proposal was rejected by its employees. Consequently, PAL ceased its operations and sent notices of termination to its employees. After sometime, PAL submitted a "stand-alone" rehabilitation plan to the SEC by which it undertook a recovery on its own while keeping its options open for the entry of a strategic partner in the future. Accordingly, it submitted an amended rehabilitation plan to the SEC

with a proposed revised business and financial restructuring plan, which required the infusion of US$200 million in new equity into the airline. The SEC approved the proposed "Amended and Restated Rehabilitation Plan" of PAL and appointed a permanent rehabilitation receiver for the latter. The former, later on, issued an Order approving the foregoing plan. However, FASAP, filed a complaint against PAL and Patricia Chiong for unfair labor practice, illegal retrenchment with claims for reinstatement and payment of salaries, allowances and backwages of affected FASAP members and damages, and prayer to enjoin the retrenchment program then being implement. Issue: Whether or not PAL was suffering from substantial actual and imminent losses to justify the adoption of the retrenchment program. Held: What further belied PAL's allegation that it was suffering from substantial actual and imminent losses was the fact that in December 1998, PAL submitted a "stand-alone" rehabilitation plan to the SEC, and on June 4, 1999, or less than a year after the retrenchment, the amount of US$200 million was invested directly into PAL by way of additional capital infusion for its operations.[81] These facts betray PAL's claim that it was in dire financial straits. By submitting a "stand-alone" rehabilitation plan, PAL acknowledged that it could undertake recovery on its own and that it possessed enough resources to weather the financial storm, if any. Thus said, it was grave error for the Labor Arbiter, the NLRC and the Court of Appeals, to have simply assumed that PAL was in grievous financial state, without requiring the latter to substantiate such claim. It bears stressing that in retrenchment cases, the presentation of proof of

financial difficulties through the required documents, preferably audited financial statements prepared by independent auditors, may not summarily be done away with.

ANTONIO C. CARAG vs. NATIONAL LABOR RELATIONS COMMISSION, ISABEL G. PANGANIBAN-ORTIGUERRA, as Executive Labor Arbiter, NAFLU and MARIVELES APPAREL CORPORATION LABOR UNION G.R. No. 147590 April 2, 2007 520 SCRA 28 Facts: The National Federation of Labor Unions (NAFLU) and Mariveles Apparel Corporation Labor Union (MACLU), on behalf of the rank and file employees of Mariveles Apparel Corporation (MAC) filed a complaint against MAC for illegal dismissal brought about by its illegal closure of business. The dispute started when, in the absence of the required notice of cessation of its business, MAC ceased its operations with the intention of completely closing its shop or factory. Such intention was manifested in a letter, allegedly claimed by MAC as its notice was filed only on the same day that the operations closed. As a result of the said business closure, the workers who rendered their services were not paid their salaries or wages. Hence, the rank and file employees claimed that the manner of closure of the operations of MAC was illegal. While the complaint was lodged before the Labor Arbiter, the complaintant filed a motion to implead MACs Chairman of the Board Antonio Carag and MACs President Armando David. The inclusion of Carag and David in the complaint is to guarantee the satisfaction of any judgment favorable to the complainants.

However, the counsel for respondents, submitted a position paper and stated that complainants should not have impleaded Carag and David because MAC is actually owned by a consortium of banks. Carag and David own shares in MAC only to qualify them to serve as MAC's officers. Issue: Whether or not Carag and David could be held personally liable for corporate debts. Held: This case also raises this issue: when is a director personally liable for the debts of the corporation? The rule is that a director is not personally liable for the debts of the corporation, which has a separate legal personality of its own. Section 31 of the Corporation Code lays down the exceptions to the rule, as follows: Liability of directors, trustees or officers. - Directors or trustees who wilfully and knowingly vote for or assent to patently unlawful acts of the corporation or who are guilty of gross negligence or bad faith in directing the affairs of the corporation or acquire any personal or pecuniary interest in conflict with their duty as such directors or trustees shall be liable jointly and severally for all damages resulting therefrom suffered by the corporation, its stockholders or members and other persons. Section 31 makes a director personally liable for corporate debts if he wilfully and knowingly votes for or assents to patently unlawful acts of the corporation. Section 31 also makes a director personally liable if he is guilty of gross negligence or bad faith in directing the affairs of the corporation. Complainants did not allege in their complaint that Carag wilfully and knowingly voted for or assented to any patently unlawful act of MAC. Complainants did not present any evidence showing that Carag wilfully and knowingly voted for or assented to any patently unlawful act of MAC.

Neither did Arbiter Ortiguerra make any finding to this effect in her Decision. Complainants did not also allege that Carag is guilty of gross negligence or bad faith in directing the affairs of MAC. Complainants did not present any evidence showing that Carag is guilty of gross negligence or bad faith in directing the affairs of MAC. Neither did Arbiter Ortiguerra make any finding to this effect in her Decision. To hold a director personally liable for debts of the corporation, and thus pierce the veil of corporate fiction, the bad faith or wrongdoing of the director must be established clearly and convincingly. Bad faith is never presumed. Bad faith does not connote bad judgment or negligence. Bad faith imports a dishonest purpose. Bad faith means breach of a known duty through some ill motive or interest. Bad faith partakes of the nature of fraud. Neither does bad faith arise automatically just because a corporation fails to comply with the notice requirement of labor laws on company closure or dismissal of employees. The failure to give notice is not an unlawful act because the law does not define such failure as unlawful. Such failure to give notice is a violation of procedural due process but does not amount to an unlawful or criminal act. For a wrongdoing to make a director personally liable for debts of the corporation, the wrongdoing approved or assented to by the director must be a patently unlawful act. Mere failure to comply with the notice requirement of labor laws on company closure or dismissal of employees does not amount to a patently unlawful act. Patently unlawful acts are those declared unlawful by law which imposes penalties for commission of such unlawful acts. There must be a law declaring the act unlawful and penalizing the act.

GARCIA vs. SOCIAL SECURITY COMMISSION LEGAL AND COLLECTION, SOCIAL SECURITY SYSTEM G. R. No . 1 7 0 7 3 5 Dece mb e r 1 7 , 2 0 0 7 Facts: Garcia was one of the directors of Impact Corporation, a corporation engaged in manufacturing aluminum tube containers and operated two factories. Due to financial problems, Impact filed with the SEC a Petition for Suspension of Payments. SSS, through its Legal and Collection Division (LCD), filed a case before the SSC for the collection of unremitted SSS premium contributions withheld by Impact Corporation from its employees. Impact Corporation, through its then Vice President Ricardo de Leon, explained that its operations had been suspended and that it was waiting for the resolution on its Petition for Suspension of Payments by the SEC. The Petition for Suspension of Payments, however, was dismissed. Impact Corporation resumed operations but only for its winding up and dissolution. SSS-LCD later filed an amended Petition impleading the directors of Impact including Garcia who insisted that she was a mere director without managerial functions, and she ceased to be such in 1982. Even as a stockholder and director of Impact Corporation, she contended that she cannot be made personally liable for the corporate obligations of Impact Corporation since her liability extended only up to the extent of her unpaid subscription, of which she had none since her subscription was already fully paid.

Whether or not the only surviving director of a corporation can be made solely liable for the corporate obligations pertaining to unremitted SSS premium contributions and penalties Held: A director, officer, and employee of a corporation are generally not held liable for obligations incurred by the corporation because corporation is invested by law with a personality separate and distinct from that of the persons composing it as well as from that of any other legal entity to which it may be related. However, a corporate director, a trustee, or an officer, maybe held solidarily liable with the corporation when he is made, by specific provision of a law, personally liable for his corporate action. As provided for in Section 31 of the Corporation Law, a corporate director, a trustee or an officer, may be held solidarily liable with the corporation in the following instances: 1. When directors and trustees or, in appropriate cases, the officers of a corporation-(a) vote for or assent to patently unlawful acts of the corporation; (b) act in bad faith or with gross negligence in directing the corporate affairs; (c) are guilty of conflict of interest to the prejudice of the corporation, its stockholders or members, and other persons.

Issue:

2. When a director or officer has consented to the issuance of watered stocks or who, having knowledge thereof, did not forthwith file with the corporate secretary his written objection thereto. 3. When a director, trustee or officer has contractually agreed or stipulated to hold himself personally and solidarily liable with the Corporation. 4. When a director, trustee or officer is made, by specific provision of law, personally liable for his corporate action. When said corporation failed to remit the SSS premium contributions falls exactly under the fourth situation. Section 28(f) of the Social Security Law imposes a civil liability for any act or omission pertaining to the violation of the Social Security Law against its managing head, directors or partners shall be liable to the penalties provided in this Act for the offense. Criminal actions for violations of the Social Security Law are also provided under the Revised Penal Code. The Social Security Law provides, in Section 28, that any employer who, after deducting the monthly contributions or loan amortizations from his employees compensation, fails to remit the said deductions to the SSS within thirty (30) days from the date they became due shall be presumed to have misappropriated such contributions or loan amortizations and shall suffer the penalties provided in Article Three hundred fifteen of the Revised Penal Code.

The personal liability for the unremitted SSS premium contributions and the late penalty thereof attaches to the petitioner as a director of Impact Corporation during the period the amounts became due and demandable by virtue of a direct provision of law. Although a corporation once formed is conferred a judicial personality separate and distinct from the persons comprising it, it is but a legal fiction introduced for purposes of convenience and to subserve the ends of justice the concept cannot be extended to a point beyond its reasons and policy.

RIGHTS OF SHARE HOLDERS REPUBLIC PLANTERS BANK VS. AGANA, SR G.R. No. 51765 March 3, 1997 Facts: Private respondent Robes-Francisco Realty & Development Corporation secured a loan from petitioner in the amount of P120,000.00. As part of the proceeds of the loan, preferred shares of stocks were issued to private respondent Corporation, through its officers then, private respondent Adalia F. Robes and one Carlos F. Robes. In other words, instead of giving the legal tender totaling to the full amount of the loan, which is P120,000.00, petitioner lent such amount partially in the form of money and partially in the form of stock certificates numbered 3204 and 3205, each for 400 shares with a par value of P10.00 per share, or for P4,000.00 each, for a total of P8,000.00. Said stock certificates were in the name of private respondent Adalia F. Robes and Carlos F. Robes, who subsequently, however, endorsed his shares in favor of Adalia F. Robes.

Private respondents proceeded against petitioner and filed a Complaint anchored on private respondents alleged rights to collect dividends under the preferred shares in question and to have petitioner redeem the same under the terms and conditions of the stock certificates. Private respondents attached to their complaint, a letter-demand dated January 5, 1979 which, significantly, was not formally offered in evidence. Petitioner filed a Motion to Dismiss but was denied by the trial court in an Order. The trial court rendered the herein assailed decision in favor of private respondents ordering petitioner to pay private respondents the face value of the stock certificates as redemption price, plus 1% quarterly interest thereon until full payment. Issue: Whether or not redemption of shares should be allowed Held: Preferred share of stock, on one hand, is one which entitles the holder thereof to certain preferences over the holders of common stock. The preferences are designed to induce persons to subscribe for shares of a corporation. Preferred shares take a multiplicity of forms. The most common forms may be classified into two: (1) preferred shares as to assets; and (2) preferred shares as to dividends. The former is a share which gives the holder thereof preference in the distribution of the assets of the corporation in case of liquidation; the latter is a share the holder of which is entitled to receive dividends on said share to the extent agreed upon before any dividends at all are paid to the holders of common stock. There is no guaranty, however, that the share will receive any dividends. The present Corporation Code provides that the board of directors of a stock corporation may declare dividends only out of unrestricted retained earnings. The Code, in Section 43, adopting the change made in accounting terminology, substituted the phrase unrestricted retained earnings, which may be a more precise term, in place of surplus profits arising from its business in

the former law. Thus, the declaration of dividends is dependent upon the availability of surplus profit or unrestricted retained earnings, as the case may be. Preferences granted to preferred stockholders, moreover, do not give them a lien upon the property of the corporation nor make them creditors of the corporation, the right of the former being always subordinate to the latter. Dividends are thus payable only when there are profits earned by the corporation and as a general rule, even if there are existing profits, the board of directors has the discretion to determine whether or not dividends are to be declared. 15 Shareholders, both common and preferred, are considered risk takers who invest capital in the business and who can look only to what is left after corporate debts and liabilities are fully paid. Redeemable shares, on the other hand, are shares usually preferred, which by their terms are redeemable at a fixed date, or at the option of either issuing corporation, or the stockholder, or both at a certain redemption price. A redemption by the corporation of its stock is, in a sense, a repurchase of it for cancellation. The present Code allows redemption of shares even if there are no unrestricted retained earnings on the books of the corporation. This is a new provision which in effect qualifies the general rule that the corporation cannot purchase its own shares except out of current retained earnings. However, while redeemable shares may be redeemed regardless of the existence of unrestricted retained earnings, this is subject to the condition that the corporation has, after such redemption, assets in its books to cover debts and liabilities inclusive of capital stock. Redemption, therefore, may not be made where the corporation is insolvent or if such redemption will cause insolvency or inability of the corporation to meet its debts as they mature.

CECILIA CASTILLO, ET AL. VS. ANGELES BALINGHASAY, ET AL. G.R. No. 150976 October 18, 2004 Facts: Petitioners and respondents are stockholders of Medical Center Paranaque, Inc. (MCPI, for brevity) with the former holding class B shares while the latter owning class A shares. MCPI was incorporated when the Old Corporation Code was then in force. Under Article VII of its Articles of Incorporation, only holders of class A shares have the right to vote and be voted for during their annual stockholders election. The said Articles of Incorporation was amended three times, however, only holders of class B shares are still deprived to vote and be voted for. The petitioners protested claiming that Article VII was null and void for depriving them, as holders of class B shares, of their right to vote and be voted upon, in violation of the New Corporation Code. They alleged that the phrase, under Artice VII, except when otherwise provided by law grants them the right to vote under the New Corporation Code. Issue: Whether or not the law referred to in the amendment to Article VII refers to the New Corporation Code and as such holders of class B shares have the right to vote and be voted upon Held: The Supreme Court found and so held that the law referred to in the amendment to Article VII refers to the Corporation Code and no other law. At the time of the incorporation of MCPI in 1977, the right of a corporation to classify its shares of stock was sanctioned by Section 5 of Act No. 1459. The law repealing Act No. 1459, B.P. Blg. 68, retained the same grant of right of classification of stock shares to corporations, but with a significant change. Under Section 6 of B.P. Blg. 68, the requirements and

restrictions on voting rights were explicitly provided for, such that "no share may be deprived of voting rights except those classified and issued as "preferred" or "redeemable" shares, unless otherwise provided in this Code" and that "there shall always be a class or series of shares which have complete voting rights." One of the rights of a stockholder is the right to participate in the control and management of the corporation that is exercised through his vote. The right to vote is a right inherent in and incidental to the ownership of corporate stock, and as such is a property right. The stockholder cannot be deprived of the right to vote his stock nor may the right be essentially impaired, either by the legislature or by the corporation, without his consent, through amending the charter, or the bylaws.

ANG-ABAYA vs ANG G.R. No. 178511 December 4, 2008 Facts: Vibelle Manufacturing Corporation and Genato Investments, Inc. are family-owned corporations, where petitioners Ma. Belen Flordeliza C. Ang-Abaya , Jason A. Ang, Vincent G. Genato, Hanna Ang and private respondent Eduardo G. Ang (Eduardo) are shareholders, officers and members of the board of directors. Prior to the instant controversy, VMC, Genato, and Oriana Manufacturing Corporation (Oriana) filed a case for damages with prayer for issuance of a TRO and/or writ of preliminary injunction against Eduardo for allegedly conniving to fraudulently wrest control/management of the corporations. Eduardo allegedly borrowed substantial amounts of money from the said corporations without any intention to repay; that he repeatedly demanded for increases in his monthly allowance and for more cash

advances contrary to existing corporate policies; that he harassed petitioner Flordeliza to transfer and/or sell certain corporate and personal properties in order to pay off his personal obligations etc. Eduardo sought permission to inspect the corporate books of VMC and Genato on account of petitioners alleged failure and/or refusal to update him on the financial and business activities of these family corporations. Petitioners denied the request claiming that Eduardo would use the information obtained from said inspection for purposes inimical to the corporations interests, considering that: a) he is harassing the corporation into writing off his advances; b) he is unjustly demanding that he be given an office/position already occupied and usurping corporate powers as well as making demands with regard to corporate properties. Because of petitioners refusal to grant his request to inspect the corporate books of VMC and Genato, Eduardo filed an Affidavit-Complaint against petitioners Flordeliza and Jason, charging them with violation (two counts) of Section 74, in relation to Section 144, of the Corporation Code of the Philippines. The City Prosecutor issued a Resolution recommending that petitioners be charged with two counts of violation of Section 74 of the Corporation Code, but dismissed the complaint against Belinda for lack of evidence. Petitioners filed a Petition for Review before the DOJ, which reversed the recommendation of the City Prosecutor. The DOJ denied Eduardos Motion for Reconsideration. The Court of Appeals reversed the DOJ. Issue: Whether or not the DOJ committed GADALEJ in reversing the resolution of the prosecutor finding probable cause against petitioners after

preliminary investigation for violation of section 74 of the corporation code of the Philippines Held: In order for the penal provision under Section 144 of the Corporation Code to apply in a case of violation of a stockholder or members right to inspect the corporate books/records as provided for under Section 74 of the Corporation Code, the following elements must be present: First. A director, trustee, stockholder or member has made a prior demand in writing for a copy of excerpts from the corporations records or minutes; Second. Any officer or agent of the concerned corporation shall refuse to allow the said director, trustee, stockholder or member of the corporation to examine and copy said excerpts; Third. If such refusal is made pursuant to a resolution or order of the board of directors or trustees, the liability under this section for such action shall be imposed upon the directors or trustees who voted for such refusal; and, Fourth. Where the officer or agent of the corporation sets up the defense that the person demanding to examine and copy excerpts from the corporations records and minutes has improperly used any information secured through any prior examination of the records or minutes of such corporation or of any other corporation, or was not acting in good faith or for a legitimate purpose in making his demand, the contrary must be shown or proved.

Thus, in a criminal complaint for violation of Section 74 of the Corporation Code, the defense of improper use or motive is in the nature of a justifying circumstance that would exonerate those who raise and are able to prove the same. Accordingly, where the corporation denies inspection on the ground of improper motive or purpose, the burden of proof is taken from the shareholder and placed on the corporation. This being the case, it would be improper for the prosecutor, during preliminary investigation, to refuse or fail to address the defense of improper use or motive, given its express statutory recognition. Thus, contrary to Eduardos insistence, the stockholders right to inspect corporate books is not without limitations. It is now expressly required as a condition for such examination that the one requesting it must not have been guilty of using improperly any information secured through a prior examination, or that the person asking for such examination must be acting in good faith and for a legitimate purpose in making his demand. The serious allegations against Eduardo are supported by official and other documents, such as board resolutions, treasurers affidavits and written communication from the respondent Eduardo himself, who appears to have withheld his objections to these charges. His silence virtually amounts to an acquiescence. Taken together, all these serve to justify petitioners allegation that Eduardo was not acting in good faith and for a legitimate purpose in making his demand for inspection of the corporate books. Otherwise stated, there is lack of probable cause to support the allegation that petitioners violated Section 74 of the Corporation Code in refusing respondents request for examination of the corporation books.

RIGHT OF FIRST REFUSAL J.G. SUMMIT HOLDINGS, INC. VS. COURT OF APPEALS, ET.AL. G.R. No. 124293 January 31, 2005 Facts: The National Investment and Development Corporation (NIDC), a government corporation, entered into a Joint Venture Agreement (JVA) with Kawasaki Heavy Industries, Ltd. of Kobe, Japan (KAWASAKI) for the construction, operation and management of the Subic National Shipyard, Inc. (SNS) which subsequently became the Philippine Shipyard and Engineering Corporation (PHILSECO). Under the JVA, the NIDC and KAWASAKI will contribute P330 million for the capitalization of PHILSECO in the proportion of 60%-40% respectively. One of its salient features was the grant to the parties of the right of first refusal should either of them decide to sell, assign or transfer its interest in the joint venture. NIDC later transferred all its rights, title and interest in PHILSECO to the Philippine National Bank (PNB). Such interests were subsequently transferred to the National Government pursuant to Administrative Order No. 14. President Corazon C. Aquino then issued Proclamation No. 50 establishing the Committee on Privatization (COP) and the Asset Privatization Trust (APT) to take title to, and possession of, conserve, manage and dispose of non-performing assets of the National Government. Thereafter, on February 27, 1987, a trust agreement was entered into between the National Government and the APT wherein the latter was named the trustee of the National Government's share in PHILSECO. In 1989, as a result of a quasi-reorganization of PHILSECO to settle its huge obligations to PNB, the National Government's shareholdings

in PHILSECO increased to 97.41% thereby reducing KAWASAKI's shareholdings to 2.59%. In the interest of the national economy and the government, the COP and the APT deemed it best to sell the National Government's share in PHILSECO to private entities. After a series of negotiations between the APT and KAWASAKI, they agreed that the latter's right of first refusal under the JVA be "exchanged" for the right to top by five percent (5%) the highest bid for the said shares. They further agreed that KAWASAKI would be entitled to name a company in which it was a stockholder, which could exercise the right to top. On September 7, 1990, KAWASAKI informed APT that Philyards Holdings, Inc. (PHI) would exercise its right to top. At the pre-bidding conference, interested bidders were given copies of the JVA between NIDC and KAWASAKI, and of the Asset Specific Bidding Rules (ASBR) drafted for the National Government's 87.6% equity share in PHILSECO. As petitioner was declared the highest bidder, the COP approved the sale on December 3, 1993 "subject to the right of Kawasaki Heavy Industries, Inc./[PHILYARDS] Holdings, Inc. to top JGSMI's bid by 5% as specified in the bidding rules." Petitioner protested the offer of PHI to top its bid because it had no right of first refusal can be exercised in a public bidding or auction sale. Petitioner was notified that PHI had fully paid the balance of the purchase price of the subject bidding. The APT notified petitioner that PHI had exercised its option to top the highest bid and that the COP had approved the same. The APT and PHI executed a Stock Purchase Agreement. Issue: Whether KAWASAKI had a valid right of first refusal over PHILSECO shares under the JVA considering that PHILSECO owned land

until the time of the bidding and KAWASAKI already held 40% of PHILSECOs equity

Held: The Supreme Court upheld the validity of the mutual rights of first refusal under the JVA between KAWASAKI and NIDC. First of all, the right of first refusal is a property right of PHILSECO shareholders, KAWASAKI and NIDC, under the terms of their JVA. This right allows them to purchase the shares of their co-shareholder before they are offered to a third party. The agreement of co-shareholders to mutually grant this right to each other, by itself, does not constitute a violation of the provisions of the Constitution limiting land ownership to Filipinos and Filipino corporations. As PHILYARDS correctly puts it, if PHILSECO still owns land, the right of first refusal can be validly assigned to a qualified Filipino entity in order to maintain the 60%-40% ratio. This transfer, by itself, does not amount to a violation of the Anti-Dummy Laws, absent proof of any fraudulent intent. The transfer could be made either to a nominee or such other party which the holder of the right of first refusal feels it can comfortably do business with. Alternatively, PHILSECO may divest of its landholdings, in which case KAWASAKI, in exercising its right of first refusal, can exceed 40% of PHILSECOs equity. In fact, it can even be said that if the foreign shareholdings of a landholding corporation exceeds 40%, it is not the foreign stockholders ownership of the shares which is adversely affected but the capacity of the corporation to own land that is, the corporation becomes disqualified to own land. This finds support under the basic corporate law principle that the corporation and its stockholders are separate juridical entities. In this vein, the right of first refusal over shares pertains to the shareholders whereas the capacity to own land pertains to the corporation. Hence, the fact that PHILSECO owns land cannot deprive stockholders of their right of first

refusal. No law disqualifies a person from purchasing shares in a landholding corporation even if the latter will exceed the allowed foreign equity, what the law disqualifies is the corporation from owning land. As correctly observed by the public respondents, the prohibition in the Constitution applies only to ownership of land. It does not extend to immovable or real property as defined under Article 415 of the Civil Code. Otherwise, we would have a strange situation where the ownership of immovable property such as trees, plants and growing fruit attached to the land would be limited to Filipinos and Filipino corporations only.

DISTRIBUTION OF DIVIDENDS COMMISSIONER OF INTERNAL REVENUE VS. COURT OF APPEALS GR NO. 108576 JANUARY 20, 1999

and records, Revenue examiners issued a report proposing that ANSCOR be assessed for deficiency withholding tax-at-source based on the transactions of exchange and redemption of stocks. ANSCOR filed a petition for review with the CTA assailing the tax assessments on the redemptions and exchange of stocks. The CTA ruled that ANSCORs redemption and exchange of the stocks of its foreign stockholders cannot be considered as essentially equivalent to a distribution of taxable dividends under Section 83(b) of the then 1939 Internal Revenue Act. ANSCOR avers that it has no duty to withhold any tax either from the Don Andres estate or from Doa Carmen based on the two transactions, because the same were done for legitimate business purposes which are (a) to reduce its foreign exchange remittances in the event the company would declare cash dividends, and to (b) subsequently filipinized ownership of ANSCOR, as allegedly, envisioned by Don Andres. It likewise invoked the amnesty provisions of P.D. 67.

Issues: Facts: Don Andres Soriano, a citizen and resident of the United States, formed the corporation A. Soriano Y Cia, predecessor of ANSCOR. ANSCOR is wholly owned and controlled by the family of Don Andres, who are all non-resident aliens. Don Andres died, but his estate continued to receive stock dividends as well as his wife Doa Carmen Soriano. Pursuant to a board resolution, ANSCOR redeemed a considerable number of common shares from Don Andres estate. As stated in the Board Resolutions, ANSCORs business purpose for both redemptions of stocks is to partially retire said stocks as treasury shares in order to reduce the companys foreign exchange remittances in case cash dividends are declared. ANSCOR also reclassified some of Doa Carmens common shares to preferred shares. After examining ANSCORs books of account Whether ANSCORs redemption of stocks from its stockholder and the exchange of stocks can be considered as essentially equivalent to the distribution of taxable dividend making the proceeds thereof taxable under the provisions of the above-quoted law

Held: The three elements in the imposition of income tax are: (1) there must be gain or and profit, (2) that the gain or profit is realized or received, actually or constructively, and (3) it is not exempted by law or treaty from income tax. The existence of legitimate business purposes in support of the redemption of stock dividends is immaterial in income taxation. The test of taxability under the exempting clause of Section 83(b) is whether income was realized through the redemption of stock dividends. The redemption

converts into money the stock dividends which become a realized profit or gain and consequently, the stockholders separate property. Profits derived from the capital invested cannot escape income tax. As realized income, the proceeds of the redeemed stock dividends can be reached by income taxation regardless of the existence of any business purpose for the redemption. Hence, the proceeds are essentially considered equivalent to a distribution of taxable dividends. As taxable dividend under Section 83(b), it is part of the entire income subject to tax under Section 22 ( tax on non-resident alien individual) in relation to Section 21 (rates of tax on citizens or residents) of the then 1939 Code. As income, it is subject to income tax which is required to be withheld at source.

other benefits. He alleged that respondents offered him compromise settlement of only P300,000.00 which he rejected. Respondent alleged that the assets of PEGGY MILLS were acquired by Sta. Rosa Textile Corporation but PEGGY MILLS still exists. Respondents alleged that except for Peggy Mills, the other respondents are not proper persons in interest due to the lack of employer-employee relationship between them and complainant. The Labor Arbiter awarded complainant his back wages plus moral damages which was deleted by the NLRC. The Court of Appeals rejected McLeods theory that all respondent corporations are the same corporate entity which should be held solidarily liable for the payment of his monetary claims. It upheld the NLRCs finding that no employer-employee relationship existed between McLeod and respondent corporations except PEGGY MILLS and stated that considering that PEGGY MILLS was no longer in operation, its officer should be held liable for acting on behalf of the corporation. Issue: Whether there was a merger or consolidation resulting into the assumption by Sta. Rosa of the liabilities of PEGGY Mills. Held:

MERGER AND CONSOLIDATION JOHN F. MCLEOD VS.NLRC, FILIPINAS SYNTHETIC FIBER CORP. (FILSYN), FAR EASTERN TEXTILE MILLS, INC., STA. ROSA TEXTILES, INC., (PEGGY MILLS, INC.), PATRICIO L. LIM, AND ERIC HU.

Facts: John F. McLeod filed a complaint for retirement and several benefits plus damages against respondents. Complainant alleged that respondent FILSYN has controlling interest in PEGGY MILLS and that he was absorbed by Peggy Mills as its Vice President and Plant Manager. He likewise alleged that Filsyn sold Peggy Mills, Inc. to Far Eastern Textile Mills, Inc and it was renamed Sta. Rosa Textile. The owners of Far Eastern Textiles decided for cessation of operations of Sta. Rosa Textiles consequently complainant wrote letters requesting for his retirement and What took place between PEGGY MILLS and Sta. RosaI was dation in payment with lease. As a rule, a corporation that purchases the assets of another will not be liable for the debts of the selling corporation, provided the former acted in good faith and paid adequate consideration for such assets, except when any of the following circumstances is present: (1) where the purchaser expressly or impliedly agrees to assume the debts, (2) where the transaction amounts to a consolidation or merger of the corporations, (3) where the purchasing corporation is merely a continuation of the selling corporation, and (4) where the selling corporation fraudulently enters into the transaction to escape liability for those debts.PEGGY MILLS

transferred its assets to Sta. RosaI to settle its obligation. PMI did not fraudulently transferred its assets to escape its liability for any of its debts. PMI had already paid its employees, except McLeod, their money claims. In the present case, there is no showing that the subject dation in payment involved any corporate merger or consolidation. Neither is there any showing of those indicative factors that SRTI is a mere instrumentality of PMI. Moreover, SRTI did not expressly or impliedly agree to assume any of PMIs debts. There being no proof of employer-employee relationship between McLeod and respondent corporations, McLeods cause of action is only against his former employer, PMI.

for the promissory note and, in support thereof, cites petitioner's failure to submit any proof of his loan application and of his actual receipt of the amount loaned.

Issues: 1.) Whether or not Associated Bank, the surviving corporation, may enforce the promissory note made by private respondent in favor of CBTC, the absorbed company, after the merger agreement had been signed, but before a certificate of merger was issued? 2.) Whether or not the promissory note was a contract pour autrui was issued without consideration?

ASSOCIATED BANK vs. CA G. R. No. 123793 June 29, 1998 Facts: Associated Banking Corporation and Citizens Bank and Trust Company (CBTC) merged to form just one banking corporation known as Associated Citizens Bank (later renamed Associated Bank), the surviving bank. After the merger agreement had been signed, but before a certificate of merger was issued, respondent Lorenzo Sarmiento, Jr. executed in favor of Associated Bank a promissory note, promising to pay the bank P2.5 million on or before due date at 14% interest per annum, among other accessory dues. For failure to pay the amount due, Sarmiento was sued by Associated Bank. Respondent argued that the plaintiff is not the proper party in interest because the promissory note was executed in favor of CBTC. Also, while respondent executed the promissory note in favor of CBTC, said note was a contract pour autrui, one in favor of a third person who may demand its fulfillment. Also, respondent claimed that he received no consideration

and Held:

1.) Associated Bank assumed all the rights of CBTC. Although absorbed corporations are dissolved, there is no winding up of their affairs or liquidation of their assets, because the surviving corporation automatically acquires all their rights, privileges and powers, as well as their liabilities. The merger, however, does not become effective upon the mere agreement of the constituent corporations. The Securities and Exchange Commission (SEC) and majority of the respective stockholders of the constituent corporations must have approved the merger. (Section 79, Corporation Code) It will be effective only upon the issuance by the SEC of a certificate of merger. Records do not show when the SEC approved the merger. But assuming that the effectivity date of the merger was the date of its execution, we still cannot agree that petitioner no longer has any interest in the promissory note. The agreement itself clearly provides that all contracts irrespective of the date of execution entered into in the name of CBTC shall be understood as pertaining to the surviving bank,

herein petitioner. Such must have been deliberately included in the agreement in order to avoid giving the merger agreement a farcical interpretation aimed at evading fulfillment of a due obligation. Thus, although the subject promissory note names CBTC as the payee, the reference to CBTC in the note shall be construed, under the very provisions of the merger agreement, as a reference to petitioner bank. 2.) On the issue that the promissory note was a contract pour autrui and was issued without consideration, the Supreme Court held it was not. In a contract pour autrui, an incidental benefit or interest, which another person gains, is not sufficient. The contracting parties must have clearly and deliberately conferred a favor upon a third person. The "fairest test" in determining whether the third person's interest in a contract is a stipulation pour autrui or merely an incidental interest is to examine the intention of the parties as disclosed by their contract. It did not indicate that a benefit or interest was created in favor of a third person. The instrument itself says nothing on the purpose of the loan, only the terms of payment and the penalties in case of failure to pay. Private respondent also claims that he received no consideration for the promissory note, citing petitioner's failure to submit any proof of his loan application and of his actual receipt of the amount loaned. These arguments deserve no merit. Res ipsa loquitur. The instrument, bearing the signature of private respondent, speaks for itself. Respondent Sarmiento has not questioned the genuineness and due execution thereof. That he partially paid his obligation is itself an express acknowledgment of his obligation.

BPI vs. BPI Employees Union GR NO. 164301 Facts: On March 23, 2000, the Bangko Sentral ng Pilipinas approved the Articles of Merger executed on January 20, 2000 by and between BPI, herein petitioner, and FEBTC.[ This Article and Plan of Merger was approved by the Securities and Exchange Commission on April 7, 2000.Pursuant to the Article and Plan of Merger, all the assets and liabilities of FEBTC were transferred to and absorbed by BPI as the surviving corporation. FEBTC employees, including those in its different branches across the country, were hired by petitioner as its own employees, with their status and tenure recognized and salaries and benefits maintained. Respondent BPI Employees Union-Davao Chapter - Federation of Unions in BPI Unibank is the exclusive bargaining agent of BPIs rank and file employees in Davao City. The former FEBTC rank-and-file employees in Davao City did not belong to any labor union at the time of the merger. Prior to the effectivity of the merger, or on March 31, 2000, respondent Union invited said FEBTC employees to a meeting regarding the Union Shop Clause of the existing CBA between petitioner BPI and respondent Union After the meeting called by the Union, some of the former FEBTC employees joined the Union, while others refused. Later, however, some of those who initially joined retracted their membership. Respondent Union then sent notices to the former FEBTC employees who refused to join, as well as those who retracted their membership, and called them to a hearing regarding the matter. When these former FEBTC employees refused to attend the hearing, the president of the Union requested BPI to implement the Union Shop Clause of the CBA and to terminate their employment pursuant thereto. After two months of management inaction on the request, respondent Union informed petitioner BPI of its decision to refer the issue of the implementation of the Union Shop Clause of the CBA to the Grievance Committee. However, the issue remained unresolved at this

level and so it was subsequently submitted for voluntary arbitration by the parties. Issue: Whether or not the former FEBTC employees that were absorbed by petitioner upon the merger between FEBTC and BPI should be covered by the Union Shop Clause found in the existing CBA between petitioner and respondent Union. Held: All employees in the bargaining unit covered by a Union Shop Clause in their CBA with management are subject to its terms. However, under law and jurisprudence, the following kinds of employees are exempted from its coverage, namely, employees who at the time the union shop agreement takes effect are bona fide members of a religious organization which prohibits its members from joining labor unions on religious grounds; employees already in the service and already members of a union other than the majority at the time the union shop agreement took effect confidential employees who are excluded from the rank and file bargaining unit;[ and employees excluded from the union shop by express terms of the agreement. When certain employees are obliged to join a particular union as a requisite for continued employment, as in the case of Union Security Clauses, this condition is a valid restriction of the freedom or right not to join any labor organization because it is in favor of unionism. This Court, on occasion, has even held that a union security clause in a CBA is not a restriction of the right of freedom of association guaranteed by the Constitution. Moreover, a closed shop agreement is an agreement whereby an employer binds himself to hire only members of the contracting union who must continue to remain members in good standing to keep their jobs. It is the most prized achievement of unionism. It adds membership and

compulsory dues. By holding out to loyal members a promise of employment in the closed shop, it wields group solidarity. Indeed, the situation of the former FEBTC employees in this case clearly does not fall within the first three exceptions to the application of the Union Shop Clause discussed earlier. No allegation or evidence of religious exemption or prior membership in another union or engagement as a confidential employee was presented by both parties. The sole category therefore in which petitioner may prove its claim is the fourth recognized exception or whether the former FEBTC employees are excluded by the express terms of the existing CBA between petitioner and respondent. To reiterate, petitioner insists that the term new employees, as the same is used in the Union Shop Clause of the CBA at issue, refers only to employees hired by BPI as non-regular employees who later qualify for regular employment and become regular employees, and not those who, as a legal consequence of a merger, are allegedly automatically deemed regular employees of BPI. However, the CBA does not make a distinction as to how a regular employee attains such a status. Moreover, there is nothing in the Corporation Law and the merger agreement mandating the automatic employment as regular employees by the surviving corporation in the merger.

DOCUMENTARY STAMP TAX JOSE ANTONIO GABUCAN vs.HON. JUDGE LUIS D. MANTA JOSEFA G. VDA. DE YSALINA and NELDA G. ENCLONAR Facts: This case is about the dismissal of a petition for the probate of a notarial will on the ground that it does not bear a thirty-centavo documentary stamp. The Court of First Instance in its "decision" in Special Proceeding for the probate of the will of the late Rogaciano Gabucan, dismissed the proceeding (erroneously characterizes as an "action"). The proceeding was dismissed because the requisite documentary stamp was not affixed to the notarial acknowledgment in the will and, hence, according to respondent Judge, it was not admissible in evidence. Respondent Judge refused to reconsider the dismissal in spite of petitioner's manifestation that he had already attached the documentary stamp to the original of the will. The case was brought to the Supreme by means of a petition for mandamus to compel the lower court to allow petitioner's appeal from its decision Issue: Whether the probate of the will should be dismissed. Held: The lower court manifestly erred in declaring that, because no documentary stamp was affixed to the will, there was "no will and testament to probate" and, consequently, the alleged "action must of necessity be dismissed".

What the probate court should have done was to require the petitioner or proponent to affix the requisite thirty-centavo documentary stamp to the notarial acknowledgment of the will which is the taxable portion of that document. That procedure may be implied from the provision of section 238 that the non-admissibility of the document, which does not bear the requisite documentary stamp, subsists only "until the requisite stamp or stamps shall have been affixed thereto and cancelled." Thus, it was held that the documentary stamp may be affixed at the time the taxable document is presented in evidence. If the promissory note does not bear a documentary stamp, the court should have allowed plaintiff's tender of a stamp to supply the deficiency. Note that the lack of the documentary stamp on a document does not invalidate such document.

VOTING TRUST AGREEMENT LEE VS. COURT OF APPEALS GR 93695 4 February 1992

Facts: On 15 November 1985, a complainant for sum of money was filed by the International Corporate Bank, Inc. against Sacoba Manufacturing Corp., Pablo Gonzales Jr., and Tomas Gonzales who, in turn, filed a third party complaint against Alfa Integrated Textile Mills (ALFA), Ramon C. Lee (ALFA's president) and Antonio DM. Lacdao (ALFA's vice president) on 17 March 1986. On 17 September 1987, Lee and Lacdao filed a motion to dismiss the third party complaint which the Regional Trial Court of

Makati, Branch 58 denied in an Order dated 27 June 1988. On 18 July 1988, Lee and Lacdao filed their answer to the third party complaint. Meanwhile, on 12 July 1988, the trial issued an order requiring the issuance of an alias summons upon ALFA through the DBP as a consequence of Lee and Lacdao's letter informing the court that the summons for ALFA was erroneously served upon them considering that the management of ALFA had been transferred to the DBP. In a manifestation dated 22 July 1988, the DBP claimed that it was not authorized to receive summons on behalf of ALFA since the DBP had not taken over the company which has a separate and distinct corporate personality and existence. On 4 August 1988, the trial court issued an order advising Sacoba Manufacturing, et. al. to take the appropriate steps to serve the summons to ALFA. On 16 August 1988, Sacoba Manufacturing, et. al. filed a Manifestation and Motion for the Declaration of Proper Service of Summons which the trial court granted on 17 August 1988. On 12 September 1988, Lee and Lacdao filed a motion for reconsideration submitting that the Rule 14, section 13 of the Revised Rules of Court is not applicable since they were no longer officers of ALFA and Sacoba Manufacturing, et. al. should have availed of another mode of service under Rule 14, Section 16 of the said Rules, i.e., through publication to effect proper service upon ALFA. On 2 January 1989, the trial court upheld the validity of the service of summons on ALFA through Lee and Lacdao, thus, denying the latter's motion for reconsideration and requiring ALFA to file its answer through Lee and Lacdao as its corporate officers. On 19 January 1989, a second motion for reconsideration was filed by Lee and Lacdao reiterating their stand that by virtue of the voting trust agreement they ceased to be officers and directors of ALFA, hence, they could no longer receive summons or any court processes for or on behalf of ALFA. In support of their second motion for reconsideration, Lee and Lacdao attached thereto a copy of the voting trust agreement between all the stockholders of ALFA (Lee and Lacdao included), on the one hand, and the DBP, on the other hand, whereby the management and control of ALFA became vested upon the DBP. On 25 April 1989, the trial court reversed

itself by setting aside its previous Order dated 2 January 1989 and declared that service upon Lee and Lacdao who were no longer corporate officers of ALFA cannot be considered as proper service of summons on ALFA. On 15 May 1989, Sacoba Manufacturing, et. al. moved for a reconsideration of the Order which was affirmed by the court in is Order dated 14 August 1989 denying Sacoba Manufacturing, et. al.'s motion for reconsideration. On 18 September 1989, a petition for certiorari was belatedly submitted by Sacoba Manufacturing, et. al. before the Court of Appeals which, nonetheless, resolved to give due course thereto on 21 September 1989. On 17 October 1989, the trial court, not having been notified of the pending petition for certiorari with the appellate court issued an Order declaring as final the Order dated 25 April 1989. Sacoba Manufacturing, et. al. in the said Order were required to take positive steps in prosecuting the third party complaint in order that the court would not be constrained to dismiss the same for failure to prosecute. Subsequently, on 25 October 1989 Sacoba Manufacturing, et. al. filed a motion for reconsideration on which the trial court took no further action. On 19 March 1990, after Lee and Lacdao filed their answer to Sacoba Manufacturing, et. al.'s petition for certiorari, the appellate court rendered its decision, setting aside the orders of trial court judge dated 25 April 1989 and 14 August 1989. On 11 April 1990, Lee and Lacdao moved for a reconsideration of the decision of the appellate court which resolved to deny the same on 10 May 1990. Lee and Lacdao filed the petition for certiorari. In the meantime, the appellate court inadvertently made an entry of judgment on 16 July 1990 erroneously applying the rule that the period during which a motion for reconsideration has been pending must be deducted from the 15-day period to appeal. However, in its Resolution dated 3 January 1991, the appellate court set aside the aforestated entry of judgment after further considering that the rule it relied on applies to appeals from decisions of the Regional Trial Courts to the Court of Appeals, not to appeals from its decision to the Supreme Court pursuant to the Supreme Court's ruling in the case of Refractories

Corporation of the Philippines v. Intermediate Appellate Court, 176 SCRA 539 [1989]. Issue: (1) Whether the execution of the voting trust agreement by Lee and Lacdao whereby all their shares to the corporation have been transferred to the trustee deprives the stockholder of their positions as directors of the corporation. Whether the five-year period of the voting trust agreement in question had lapsed in 1986 so that the legal title to the stocks covered by the said voting trust agreement ipso facto reverted to Lee and Lacdao as beneficial owners pursuant to the 6th paragraph of section 59 of the new Corporation Code.

DBP had caused to be transferred in their names one share of stock for the purpose of qualifying as directors of ALFA, Lee and Lacdao can no longer be deemed to have retained their status as officers of ALFA which was the case before the execution of the subject voting trust agreement. There is no dispute from the records that DBP has taken over full control and management of the firm. (2) The 6th paragraph of section 59 of the new Corporation Code reads that "Unless expressly renewed, all rights granted in a voting trust agreement shall automatically expire at the end of the agreed period, and the voting trust certificates as well as the certificates of stock in the name of the trustee or trustees shall thereby be deemed cancelled and new certificates of stock shall be reissued in the name of the transferors." However, it is manifestly clear from the terms of the voting trust agreement between ALFA and the DBP that the duration of the agreement is contingent upon the fulfillment of certain obligations of ALFA with the DBP. Had the five-year period of the voting trust agreement expired in 1986, the DBP would not have transferred an its rights, titles and interests in ALFA "effective June 30, 1986" to the national government through the Asset Privatization Trust (APT) as attested to in a Certification dated 24 January 1989 of the Vice President of the DBP's Special Accounts Department II. In the same certification, it is stated that the DBP, from 1987 until 1989, had handled s account which included ALFA's assets pursuant to a management agreement by and between the DBP and APT. Hence, there is evidence on record that at the time of the service of summons on ALFA through Lee and Lacdao on 21 August 1987, the voting trust agreement in question was not yet terminated so that the legal title to the stocks of ALFA, then, still belonged to the DBP.

(2)

Held: (1) Lee and Lacdao, by virtue of the voting trust agreement executed in 1981 disposed of all their shares through assignment and delivery in favor of the DBP, as trustee. Consequently, Lee and Lacdao ceased to own at least one share standing in their names on the books of ALFA as required under Section 23 of the new Corporation Code. They also ceased to have anything to do with the management of the enterprise. Lee and Lacdao ceased to be directors. Hence, the transfer of their shares to the DBP created vacancies in their respective positions as directors of ALFA. The transfer of shares from the stockholders of ALFA to the DBP is the essence of the subject voting trust agreement. Considering that the voting trust agreement between ALFA and the DBP transferred legal ownership of the stocks covered by the agreement to the DBP as trustee, the latter because the stockholder of record with respect to the said shares of stocks. In the absence of a showing that the

In Re: Petition for Assistance in the Liquidation in the Rural Bank of Bokod (Benguet), PDIC vs. Bureau of Internal Revenue GR No. 158261 December 18, 2006 511 SCRA 123 Facts: A special examination of Rural Bank of Bokod (Benguet), Inc. (RBBI) was conducted by the Supervision and Examination Sector (SES) Department III of what is now the Bangko Sentral ng Pilipinas (BSP), wherein various loan irregularities were uncovered. SES Department III required the RRBI management to infuse fresh capital into the bank, within 30 days from date of the advice, and to correct all the exceptions noted. However, up to the termination of the subsequent general examination conducted by the SES Department III, no concrete action was taken by the RBBI management. In view of the irregularities noted and the insolvent condition of RBBI, the members of the RBBI Board of Directors were called for a conference at the BSP. Only one RBBI Director, a certain Mr. Wakit, attended the conference, and the examination findings and related recommendations were discussed with him. In a letter, receipt of which was acknowledged by Mr. Wakit, the SES Department III warned the RBBI Board of Directors that, unless substantial remedial measures are taken to rehabilitate the bank, it will recommend that the bank be placed under receivership. In a subsequent letter, a copy of which was sent to every member of th RBBI Board of Directors via registered mail, the SES Department III reiterated its warning that it would recommend the closure of the bank, unless the needed fresh capital was immediately infused. Despite these notices, the SES Department III received no word from RBBI or from any of its Directors. In a meeting, the Monetary Board of the BSP decided to take the following: forbid the bank to do business in the Philippines and place its assets and affairs under receivership in accordance with Section 29 of RA No. 265 as amended; designate the Special Assistant to the Governor and Head, SES Department III, as Receiver of the bank; refer the cases of irregularities/frauds to the Office of Special Investigation for further

investigation and possible filing of appropriate charges against the following present/former officers and employees of the bank. A memorandum and report, were submitted by the Director of the SES Department III concluding that the RBBI remained in insolvent financial condition and it can no longer safely resume business with the depositors, creditors, and the general public. The Monetary Board, after determining and confirming the said memorandum and report, ordered the liquidation of the bank and designated the Director of the SES Department III as liquidator. The designated BSP liquidator of RBBI caused the filing with the RTC of a Petition for Assistance in the Liquidation of RBBI. Subsequently, the Monetary Board transferred to herein Philippine Deposit Insurance Corporation (PDIC) the receivership/liquidation of RBBI. PDIC then filed a Motion for Approval of Project of Distribution of the assets of RBBI, in accordance with Section 31, in relation to Section 30 of Republic Act No. 7653, otherwise known as the New Central Bank Act. During the hearing, the Bureau of Internal Revenue (BIR), through Atty. Justo Reginaldo, manifested that PDIC should secure a tax clearance certificate from the appropriate BIR Regional Office, pursuant to Section 52(C) of the Tax Code of 1997 within 30 days from receipt of a copy of the said order. Pending compliance therewith, the RTC held in abeyance the Motion for Approval of Project of Distribution. In order therefore that all taxes due the government should be paid, petitioner should secure a tax clearance from the BIR. Hence, PDIC filed the present Petition for Review on Certiorari, under Rule 45 of the revised Rules of Court, raising pure questions of law. PDIC argues that the closure of banks under Section 30 of the New Central Bank Act is summary in nature and procurement of tax clearance as required under Section 52(C) of the Tax Code of 1997 is not a condition precedent thereto; that under Section 30, in relation to Section 31, of the New Central Bank Act, asset distribution of a closed bank requires only the approval of the liquidation court; and that the BIR is not without recourse since, subject to the applicable provisions of the Tax Code of 1997, it may therefore assess the closed RBBI for tax liabilities, if any. Issue:

Whether or not RBBI, as represented by its liquidator, PDIC, still needs to secure a tax clearance from the BIR before the RTC could approve the Project of Distribution of the assets of RBBI. Held: Section 30 of the New Central Bank Act lays down the proceedings for receivership and liquidation of a bank. The said provision is silent as regards the securing of a tax clearance from the BIR. The omission, nonetheless, cannot compel this Court to apply by analogy the tax clearance requirement of the SEC, as stated in Section 52(C) of the Tax Code of 1997 and BIR-SEC Regulations No. 1, since, again, the dissolution of a corporation by the SEC is a totally different proceeding from the receivership and liquidation of a bank by the BSP. This Court cannot simply replace any reference by Section 52(C) of the Tax Code of 1997 and the provisions of the BIR-SEC Regulations No. 1 to the "SEC" with the "BSP." To do so would be to read into the law and the regulations something that is simply not there and would be tantamount to judicial legislation. It should be noted that there are substantial differences in the procedure for involuntary dissolution and liquidation of a corporation under the Corporation Code, and that of a banking corporation under the New Central Bank Act, so that the requirements in one cannot simply be imposed in the other.

allied purposes, and in the purchase, sale and/or exchange of said subdivision and properties through network marketing. Noel Manero requested public respondent SEC to investigate petitioners business. He claimed that he attended a seminar conducted by petitioner where the latter claimed to sell properties that were inexistent and without any brokers license. One Romulo E. Munsayac, Jr. inquired from public respondent SEC whether petitioners business involves "legitimate network marketing." To address the issue, SEC conducted a conference with the incorporators. Also, thepetitioner submitted their marketing modules and their letters of accreditation authority from Crown Asia, FilEstate Network and Pioneer 29 Realty Corporation. SEC visited the business premises of petitioner wherein it gathered documents such as certificates of accreditation to several real estate companies, list of members with web sites, sample of member mail box, webpages of two (2) members, and lists of Business Center Owners who are qualified to acquire real estate properties and materials on computer tutorials. SEC found that the petitioner is engaged in the sale or offer for sale or distribution of investment contracts, which are considered securities under Sec. 3.1 (b) of Republic Act (R.A.) No. 8799 (The Securities Regulation Code), but failed to register them in violation of Sec. 8.1 of the same Act, and issued a Cease and Desist Order. The order was appealed but the Court of Appeals affirmed the SEC. Issue: Whether the business of petitioner involves an investment contract that is considered security11 and thus, must be registered prior to sale or offer for sale or distribution to the public pursuant to Section 8.1 of R.A. No. 8799 Held: Yes. The business of the petitioner involves an investment contract. It is a contract, transaction or scheme (collectively contract) whereby a person invests his money in a common enterprise and is led to expect profits primarily from the efforts of others. (R.A. 8799) The HOWEY TEST is the test established to determine whether a transaction falls within the scope of an investment contract. It requires that a person:

POWER HOMES UNLIMITED CORPORATION vs.SEC G.R. No. 164182 February 26, 2008 546 SCRA 567

Facts: Petitioner is a domestic corporation duly registered with public respondent SEC. Its primary purpose is: To engage in the transaction of promoting, acquiring, managing, leasing, obtaining options on, development, and improvement of real estate properties for subdivision and

1. Makes an investment of money; 2. In a common enterprise; 3. With the expectation of profits; 4. To be derived primarily from the efforts of others. We therefore rule that the business operation or the scheme of petitioner constitutes an investment contract that is a security under R.A. No. 8799. Thus, it must be registered with public respondent SEC before its sale or offer for sale or distribution to the public. As petitioner failed to register the same, its offering to the public was rightfully enjoined by public respondent SEC. The CDO was proper even without a finding of fraud. As an investment contract that is security under R.A. No. 8799, it must be registered with public respondent SEC, otherwise the SEC cannot protect the investing public from fraudulent securities. The strict regulation of securities is founded on the premise that the capital markets depend on the investing publics level of confidence in the system.

the Philippine-American Drug, Co., Inc. instituted an action in the Court of First Instance (CFI) of Manila against Anacleto Mangaliman, Florencio Mangaliman and the Director of the Bureau of Commerce for infringement of trade mark and unfair competition. Mentholatum prayed for the issuance of an order restraining Mangalimans from selling their product "Mentholiman," and directing them to render an accounting of their sales and profits and to pay damages. After a protracted trial, the Court of First Instance of Manila rendered judgment in favor of Mentholatum. In the Court of Appeals, the decision of the trial court was reversed, said tribunal holding that the activities of the Mentholatum Inc., were business transactions in the Philippines, and that by section 69 of the Corporation Law, it may not maintain the suit. Hence, Mentholatum filed the instant petition for certiorari. Issue: Whether Mentholatum could prosecute the instant action without having secured the license required in section 69 of the Corporation Law.

The Mentholatum Co. Inc. vs. Mangaliman GR 47701 27 June 1941 78 Phil. 525 Facts: The Mentholatum Co., Inc., is a Kansas corporation which manufactures "Mentholatum," a medicament and salve adapted for the treatment of colds, nasal irritations, chapped skin, insect bites, rectal irritation and other external ailments of the body. The Philippine-American Drug Co., Inc., is its exclusive distributing agent in the Philippines authorized by it to look after and protect its interests. On 26 June 1919 and on 21 January 1921, the Mentholatum Co., Inc., registered with the Bureau of Commerce and Industry the word, "Mentholatum", as trade mark for its products. The Mangaliman brothers prepared a medicament and salve named "Mentholiman" which they sold to the public packed in a container of the same size, color and shape as "Mentholatum." As a consequence of these acts of the Mangalimans, Mentholatum suffered damages from the diminution of their sales and the loss of goodwill and reputation of their product in the market. On 1 October 1935, the Mentholatum Co., Inc., and

Held: No general rule or governing principle can be laid down as to what constitutes "doing" or "engaging in" or "transacting" business. The true test, however, seems to be whether the foreign corporation is continuing the body or substance of the business or enterprise for which it was organized or whether it has substantially retired from it and turned it over to another. The term implies a continuity of commercial dealings and arrangements, and contemplates, to that extent, the performance of acts or works or the exercise of some of the functions normally incident to, and in progressive prosecution of, the purpose and object of its organization. Herein, Mentholatum Co., through its agent, the Philippine-American Drug Co., Inc., has been doing business in the Philippines by selling its products here since the year 1929, at least. Whatever transactions the Philippine-American Drug Co., Inc., had executed in view of the law, the Mentholatum Co., Inc., being a foreign corporation doing business in the Philippines without the license required by section 68 of the Corporation Law, it may not prosecute this action for violation of trade mark and unfair competition. Neither may the Philippine-American Drug Co., Inc., maintain the action here for the reason that the distinguishing features of the agent being his representative

character and derivative authority, it cannot now, to the advantage of its principal, claim an independent standing in court. Further, the recognition of the legal status of a foreign corporation is a matter affecting the policy of the forum, and the distinction drawn in Philippine Corporation Law is an expression of the policy. Agilent Technologies Singapore vs. Integrated Silicon Techngology Philippines Corp. GR 154618 14 April 2004 427 SCRA 593 Facts: Agilent Technologies Singapore (Pte.), Ltd. is a foreign corporation, which, by its own admission, is not licensed to do business in the Philippines. Integrated Silicon Technology Philippines Corporation is a private domestic corporation, 100% foreign owned, which is engaged in the business of manufacturing and assembling electronics components. Teoh Kiang Hong, Teoh Kiang Seng and Anthony Choo, Malaysian nationals, are current members of Integrated Silicons board of directors, while Joanne Kate M. dela Cruz, Jean Kay M. dela Cruz, and Rolando T. Nacilla are its former members. The juridical relation among the various parties in the case can be traced to a 5-year Value Added Assembly Services Agreement (VAASA), entered into on 2 April 1996 between Integrated Silicon and the HewlettPackard Singapore (Pte.) Ltd., Singapore Components Operation (HPSingapore). Under the terms of the VAASA, Integrated Silicon was to locally manufacture and assemble fiber optics for export to HP-Singapore. HP-Singapore, for its part, was to consign raw materials to Integrated Silicon; transport machinery to the plant of Integrated Silicon; and pay Integrated Silicon the purchase price of the finished products. The VAASA had a five-year term, beginning on 2 April 1996, with a provision for annual renewal by mutual written consent. On 19 September 1999, with the consent of Integrated Silicon, HP-Singapore assigned all its rights and obligations in the VAASA to Agilent. On 25 May 2001, Integrated Silicon filed a complaint for Specific Performance and Damages against Agilent and its officers Tan Bian Ee, Lim Chin Hong, Tey Boon Teck and Francis

Khor alleging that Agilent breached the parties oral agreement to extend the VAASA. Integrated Silicon thus prayed that Agilent be ordered to execute a written extension of the VAASA for a period of five years as earlier assured and promised; to comply with the extended VAASA; and to pay actual, moral, exemplary damages and attorneys fees. On 1 June 2001, summons and a copy of the complaint were served on Atty. Ramon Quisumbing, who returned these processes on the claim that he was not the registered agent of Agilent. Later, he entered a special appearance to assail the courts jurisdiction over the person of Agilent. On 2 July 2001, Agilent filed a separate complaint against Integrated Silicon, Teoh Kang Seng, Teoh Kiang Gong, Anthony Choo, Joanne Kate M. dela Cruz, Jean Kay M. dela Cruz and Rolando T. Nacilla, for Specific Performance, Recovery of Possession, and Sum of Money with Replevin, Preliminary Mandatory Injunction, and Damages, before the Regional Trial Court, Calamba, Laguna, Branch 9). Agilent prayed that a writ of replevin or, in the alternative, a writ of preliminary mandatory injunction, be issued ordering Integrated Silicon, et. al. to immediately return and deliver to Agilent its equipment, machineries and the materials to be used for fiber-optic components which were left in the plant of Integrated Silicon; and that the latter be ordered to pay actual and exemplary damages and attorneys fees. Integrated Silicon, et. al. filed a Motion to Dismiss in Civil Case No. 31232001-C, on the grounds of lack of Agilents legal capacity to sue; litis pendentia; forum shopping; and failure to state a cause of action. On 4 September 2001, the trial court denied the Motion to Dismiss and granted Agilents application for a writ of replevin. Without filing a motion for reconsideration, Integrated Silicon, et. al. filed a petition for certiorari with the Court of Appeals. In the meantime, upon motion filed by Integrated Silicon, et. al., Judge Antonio S. Pozas of Branch 92 voluntarily inhibited himself in Civil Case 3123-2001-C. The case was re-raffled and assigned to Branch 35, the same branch where Civil Case 3110-2001-C is pending. On 12 August 2002, the Court of Appeals granted Integrated Silicon, et. al.s petition for certiorari, set aside the assailed Order of the trial court dated 4 September 2001, and ordered the dismissal of Civil Case 3123-2001-C. Agilent filed the petition for review. Issue [1]: Whether a foreign corporation without a license is incapacitated from bringing an action in Philippine courts.

Issue [2]: Whether Agilent was doing business in the Philippines. Held [1]: A foreign corporation without a license is not ipso facto incapacitated from bringing an action in Philippine courts. A license is necessary only if a foreign corporation is transacting or doing business in the country. Section 133 of the Corporation Code provides that "No foreign corporation transacting business in the Philippines without a license, or its successors or assigns, shall be permitted to maintain or intervene in any action, suit or proceeding in any court or administrative agency of the Philippines; but such corporation may be sued or proceeded against before Philippine courts or administrative tribunals on any valid cause of action recognized under Philippine laws." The aforementioned provision prevents an unlicensed foreign corporation doing business in the Philippines from accessing our courts. In a number of cases, however, the Court held that an unlicensed foreign corporation doing business in the Philippines may bring suit in Philippine courts against a Philippine citizen or entity who had contracted with and benefited from said corporation. Such a suit is premised on the doctrine of estoppel. A party is estopped from challenging the personality of a corporation after having acknowledged the same by entering into a contract with it. This doctrine of estoppel to deny corporate existence and capacity applies to foreign as well as domestic corporations. The application of this principle prevents a person contracting with a foreign corporation from later taking advantage of its noncompliance with the statutes chiefly in cases where such person has received the benefits of the contract. The principles regarding the right of a foreign corporation to bring suit in Philippine courts may thus be condensed in four statements: (1) if a foreign corporation does business in the Philippines without a license, it cannot sue before the Philippine courts; (2) if a foreign corporation is not doing business in the Philippines, it needs no license to sue before Philippine courts on an isolated transaction or on a cause of action entirely independent of any business transaction; (3) if a foreign corporation does business in the Philippines without a license, a Philippine citizen or entity which has contracted with said corporation may be estopped from challenging the foreign corporations corporate personality in a suit brought before Philippine courts; and (4) if a foreign corporation does business in the Philippines with the required license, it can sue before Philippine courts on any transaction. Held [2]: The challenge to Agilents legal capacity to file suit hinges on whether or not it is doing business in the Philippines. However, there is no definitive rule on what constitutes doing, engaging in, or transacting business in the Philippines, the Corporation Code itself is silent as to what acts constitute doing or transacting business in the Philippines. An analysis of the relevant case law, in conjunction with Section 1 of the Implementing Rules and Regulations of the Foreign Investments Act of 1991 (FIA, as amended by RA 8179), would demonstrate that the acts enumerated in the VAASA do not constitute doing business in the Philippines. By and large, to constitute doing business, the activity to be undertaken in the Philippines is one that is for profit-making. Herein, by the clear terms of the VAASA, Agilents activities in the Philippines were confined to (1) maintaining a stock of goods in the Philippines solely for the purpose of having the same processed by Integrated Silicon; and (2) consignment of equipment with Integrated Silicon to be used in the processing of products for export. As such, Agilent cannot be deemed to be doing business in the Philippines. Integrated Silicon, et. al.s contention that Agilent lacks the legal capacity to file suit is therefore devoid of merit. As a foreign corporation not doing business in the Philippines, it needed no license before it can sue before our court.

GELANO vs COURT of APPEALS G.R. No. L-39050 February 24, 1981 Facts: Private respondent Insular Sawmill, Inc. is a corporation organized on September 17, 1945 with a corporate life of fifty (50) years, or up to September 17, 1995, with the primary purpose of carrying on a general lumber and sawmill business. To carry on this business, private respondent leased the paraphernal property of petitioner-wife Guillermina M. Gelano at the corner of Canonigo and Otis, Paco, Manila for P1,200.00 a month. It was while private respondent was leasing the aforesaid property that its officers and directors had come to know petitioner-husband Carlos Gelano

who received from the corporation cash advances on account of rentals to be paid by the corporation on the land. Between November 19, 1947 to December 26, 1950 petitioner Carlos Gelano obtained from private respondent cash advances of P25,950.00. The said sum was taken and received by petitioner Carlos Gelano on the agreement that private respondent could deduct the same from the monthly rentals of the leased premises until said cash advances are fully paid. Out of the aforementioned cash advances in the total sum of P25,950.00, petitioner Carlos Gelano was able to pay only P5,950.00 thereby leaving an unpaid balance of P20,000.00 which he refused to pay despite repeated demands by private respondent. Petitioner Guillermina M. Gelano refused to pay on the ground that said amount was for the personal account of her husband asked for by, and given to him, without her knowledge and consent and did not benefit the family. On various occasions from May 4, 1948 to September 11, 1949 petitioners husband and wife also made credit purchases of lumber materials from private respondent with a total price of P1,120.46 in connection with the repair and improvement of petitioners' residence. On November 9, 1949 partial payment was made by petitioners in the amount of P91.00 and in view of the cash discount in favor of petitioners in the amount of P83.00, the amount due private respondent on account of credit purchases of lumber materials is P946.46 which petitioners failed to pay. On July 14, 1952, in order to accommodate and help petitioners renew previous loans obtained by them from the China Banking Corporation, private respondent, through Joseph Tan Yoc Su, executed a joint and several promissory note with Carlos Gelano in favor of said bank in the amount of P8,000.00 payable in sixty (60) days. For failure of Carlos Gelano to pay the promissory note upon maturity, the bank collected from the respondent corporation the amount of P9,106.00 including interests, by debiting it from the corporation's current account with the bank. Petitioner

Carlos Gelano was able to pay private respondent the amount of P5,000.00 but the balance of P4,106.00 remained unsettled. Guillermina M. Gelano refused to pay on the ground that she had no knowledge about the accommodation made by the corporation in favor of her husband. On May 29, 1959 the corporation, thru Atty. German Lee, filed a complaint for collection against herein petitioners before the Court of First Instance of Manila. Trial was held and when the case was at the stage of submitting memorandum, Atty. Lee retired from active law practice and Atty. Eduardo F. Elizalde took over and prepared the memorandum. In the meantime, private respondent amended its Articles of Incorporation to shorten its term of existence up to December 31, 1960 only. The amended Articles of Incorporation was filed with, and approved by the Securities and Exchange Commission, but the trial court was not notified of the amendment shortening the corporate existence and no substitution of party was ever made. On November 20, 1964 and almost four (4) years after the dissolution of the corporation, the trial court rendered a decision in favor of private respondent.

Issue: Whether or not a corporation, whose corporate life had ceased by the expiration of its term of existence, could still continue prosecuting and defending suits after its dissolution and beyond the period of three years provided for under Act No. 1459, otherwise known as the Corporation law, to wind up its affairs, without having undertaken any step to transfer its assets to a trustee or assignee.

Held: Yes. The complaint in this case was filed on May 29, 1959 when private respondent Insular Sawmill, Inc. was still existing. While the case was being tried, the stockholders amended its Articles of Incorporation by

shortening the term of its existence from December 31, 1995 to December 31, 1960, which was approved by the Securities and Exchange Commission. In American corporate law, upon which our Corporation Law was patterned, it is well settled that, unless the statutes otherwise provide, all pending suits and actions by and against a corporation are abated by a dissolution of the corporationSection 77 of the Corporation Law provides that the corporation shall "be continued as a body corporate for three (3) years after the time when it would have been ... dissolved, for the purpose of prosecuting and defending suits By or against it ...," so that, thereafter, it shall no longer enjoy corporate existence for such purpose. For this reason, Section 78 of the same law authorizes the corporation, "at any time during said three years ... to convey all of its property to trustees for the benefit of members, Stockholders, creditors and other interested," evidently for the purpose, among others, of enabling said trustees to prosecute and defend suits by or against the corporation begun before the expiration of said period. Commenting on said sections, Justice Fisher said: It is to be noted that the time during which the corporation, through its own officers, may conduct the liquidation of its assets and sue and be sued as a corporation is limited to three years from the time the period of dissolution commences; but that there is no time limited within which the trustees must complete a liquidation placed in their hands. It is provided only (Corp. Law, Sec. 78) that the conveyance to the trustees must be made within the three-year period. It may be found impossible to complete the work of liquidation within the three-year period or to reduce disputed claims to judgment. The authorities are to the effect that suits by or against a corporation abate when it ceased to be an entity capable of suing or being sued (7 R.C.L. Corps., Par. 750); but trustees to whom the corporate assets have been conveyed pursuant to the authority of Section 78 may sue and be sued as such in all matters connected with the liquidation. By the terms of the statute the effect of the conveyance is to make the trustees the legal owners of the property conveyed, subject to the beneficial interest therein of creditors and stockholders.

When Insular Sawmill, Inc. was dissolved on December 31, 1960, under Section 77 of the Corporation Law, it still has the right until December 31, 1963 to prosecute in its name the present case. After the expiration of said period, the corporation ceased to exist for all purposes and it can no longer sue or be sued. However, a corporation that has a pending action and which cannot be terminated within the three-year period after its dissolution is authorized under Section 78 to convey all its property to trustees to enable it to prosecute and defend suits by or against the corporation beyond the Threeyear period although private respondent (did not appoint any trustee, yet the counsel who prosecuted and defended the interest of the corporation in the instant case and who in fact appeared in behalf of the corporation may be considered a trustee of the corporation at least with respect to the matter in litigation only. Said counsel had been handling the case when the same was pending before the trial court until it was appealed before the Court of Appeals and finally to this Court. We therefore hold that there was a substantial compliance with Section 78 of the Corporation Law and as such, private respondent Insular Sawmill, Inc. could still continue prosecuting the present case even beyond the period of three (3) years from the time of its dissolution. From the above quoted commentary of Justice Fisher, the trustee may commence a suit which can proceed to final judgment even beyond the three-year period. No reason can be conceived why a suit already commenced By the corporation itself during its existence, not by a mere trustee who, by fiction, merely continues the legal personality of the dissolved corporation should not be accorded similar treatment allowed to proceed to final judgment and execution thereof. The word "trustee" as sued in the corporation statute must be understood in its general concept which could include the counsel to whom was entrusted in the instant case, the prosecution of the suit filed by the corporation. The purpose in the transfer of the assets of the corporation to a trustee upon its dissolution is

more for the protection of its creditor and stockholders. Debtors like the petitioners herein may not take advantage of the failure of the corporation to transfer its assets to a trustee, assuming it has any to transfer which petitioner has failed to show, in the first place. To sustain petitioners' contention would be to allow them to enrich themselves at the expense of another, which all enlightened legal systems condemn.

PHILIPPINE STOCK EXCHANGE, INC. vs. COURT OF APPEALS G.R. No. 125469 October 27, 1997 Facts: The Puerto Azul Land, Inc. (PALI), a domestic real estate corporation, had sought to offer its shares to the public in order to raise funds allegedly to develop its properties and pay its loans with several banking institutions. In January, 1995, PALI was issued a Permit to Sell its shares to the public by the Securities and Exchange Commission (SEC). To facilitate the trading of its shares among investors, PALI sought to course the trading of its shares through the Philippine Stock Exchange, Inc. (PSE), for which purpose it filed with the said stock exchange an application to list its shares, with supporting documents attached. On February 8, 1996, the Listing Committee of the PSE, upon a perusal of PALI's application, recommended to the PSE's Board of Governors the approval of PALI's listing application. On February 14, 1996, before it could act upon PALI's application, the Board of Governors of the PSE received a letter from the heirs of Ferdinand E. Marcos, claiming that the late President Marcos was the legal and beneficial owner of certain properties forming part of the Puerto Azul Beach Hotel and Resort Complex which PALI claims to be among its assets and that the Ternate Development Corporation, which is among the stockholders of PALI, likewise appears to have been held and continue to be held in trust by one Rebecco Panlilio for then President Marcos and now, effectively for his estate, and requested PALI's application to be deferred. PALI was requested to comment upon the said letter.

PALI's answer stated that the properties forming part of the Puerto Azul Beach Hotel and Resort Complex were not claimed by PALI as its assets. On the contrary, the resort is actually owned by Fantasia Filipina Resort, Inc. and the Puerto Azul Country Club, entities distinct from PALI. Furthermore, the Ternate Development Corporation owns only 1.20% of PALI. The Marcoses responded that their claim is not confined to the facilities forming part of the Puerto Azul Hotel and Resort Complex, thereby implying that they are also asserting legal and beneficial ownership of other properties titled under the name of PALI. On February 20, 1996, the PSE wrote Chairman Magtanggol Gunigundo of the Presidential Commission on Good Government (PCGG) requesting for comments on the letters of the PALI and the Marcoses. On March 4, 1996, the PSE was informed that the Marcoses received a Temporary Restraining Order on the same date, enjoining the Marcoses from, among others, "further impeding, obstructing, delaying or interfering in any manner by or any means with the consideration, processing and approval by the PSE of the initial public offering of PALI." The TRO was issued by Judge Martin S. Villarama, Executive Judge of the RTC of Pasig City in Civil Case No. 65561, pending in Branch 69 thereof. In its regular meeting held on March 27, 1996, the Board of Governors of the PSE reached its decision to reject PALI's application, citing the existence of serious claims, issues and circumstances surrounding PALI's ownership over its assets that adversely affect the suitability of listing PALI's shares in the stock exchange. On April 11, 1996, PALI wrote a letter to the SEC addressed to the then Acting Chairman, Perfecto R. Yasay, Jr., bringing to the SEC's attention the action taken by the PSE in the application of PALI for the listing of its shares with the PSE, and requesting that the SEC, in the exercise of its supervisory and regulatory powers over stock exchanges under Section 6(j) of P.D. No. 902-A, review the PSE's action on PALI's listing application and institute such measures as are just and proper under the circumstances. On the same date, or on April 11, 1996, the SEC wrote to the PSE, attaching thereto the letter of PALI and directing the PSE to file its comments thereto within five days from its receipt and for its authorized representative to appear for an "inquiry" on the matter. On April 22, 1996, the PSE submitted a letter to the SEC containing its comments to the April

11, 1996 letter of PALI. On April 24, 1996, the SEC rendered its Order, reversing the PSE's decision. Issue: Whether or not SEC has the power or jurisdiction to reverse the ruling of Philippine Stock Exchange in this case. Held: No. The role of the SEC in our national economy cannot be minimized. The legislature, through the Revised Securities Act, Presidential Decree No. 902-A, and other pertinent laws, has entrusted to it the serious responsibility of enforcing all laws affecting corporations and other forms of associations not otherwise vested in some other government office. This is not to say, however, that the PSE's management prerogatives are under the absolute control of the SEC. The PSE is, alter all, a corporation authorized by its corporate franchise to engage in its proposed and duly approved business. One of the PSE's main concerns, as such, is still the generation of profit for its stockholders. Moreover, the PSE has all the rights pertaining to corporations, including the right to sue and be sued, to hold property in its own name, to enter (or not to enter) into contracts with third persons, and to perform all other legal acts within its allocated express or implied powers. A corporation is but an association of individuals, allowed to transact under an assumed corporate name, and with a distinct legal personality. In organizing itself as a collective body, it waives no constitutional immunities and perquisites appropriate to such a body. 11 As to its corporate and management decisions, therefore, the state will generally not interfere with the same. Questions of policy and of management are left to the honest decision of the officers and directors of a corporation, and the courts are without authority to substitute their judgment for the judgment of the board of directors. The board is the business manager of the corporation, and so long as it acts in good faith, its orders are not reviewable by the courts. Thus, notwithstanding the regulatory power of the SEC over the PSE, and the resultant authority to reverse the PSE's decision in matters of application for listing in the market, the SEC may exercise such power only if the PSE's judgment is attended by bad faith. In Board of Liquidators vs.

Kalaw, 13 it was held that bad faith does not simply connote bad judgment or negligence. It imports a dishonest purpose or some moral obliquity and conscious doing of wrong. It means a breach of a known duty through some motive or interest of ill will, partaking of the nature of fraud. In reaching its decision to deny the application for listing of PALI, the PSE considered important facts, which, in the general scheme, brings to serious question the qualification of PALI to sell its shares to the public through the stock exchange. During the time for receiving objections to the application, the PSE heard from the representative of the late President Ferdinand E. Marcos and his family who claim the properties of the private respondent to be part of the Marcos estate. In time, the PCGG confirmed this claim. In fact, an order of sequestration has been issued covering the properties of PALI, and suit for reconveyance to the state has been filed in the Sandiganbayan Court. How the properties were effectively transferred, despite the sequestration order, from the TDC and MSDC to Rebecco Panlilio, and to the private respondent PALI, in only a short span of time, are not yet explained to the Court, but it is clear that such circumstances give rise to serious doubt as to the integrity of PALI as a stock issuer. The petitioner was in the right when it refused application of PALI, for a contrary ruling was not to the best interest of the general public. The purpose of the Revised Securities Act, after all, is to give adequate and effective protection to the investing public against fraudulent representations, or false promises, and the imposition of worthless ventures. Also, as the primary market for securities, the PSE has established its name and goodwill, and it has the right to protect such goodwill by maintaining a reasonable standard of propriety in the entities who choose to transact through its facilities. It was reasonable for the PSE, therefore, to exercise its judgment in the manner it deems appropriate for its business identity, as long as no rights are trampled upon, and public welfare is safeguarded. In this connection, it is proper to observe that the concept of government absolutism is a thing of the past, and should remain so. In any case, for the purpose of determining whether PSE acted correctly in refusing the application of PALI, the true ownership of the

properties of PALI need not be determined as an absolute fact. What is material is that the uncertainty of the properties' ownership and alienability exists, and this puts to question the qualification of PALI's public offering. In sum, the Court finds that the SEC had acted arbitrarily in arrogating unto itself the discretion of approving the application for listing in the PSE of the private respondent PALI, since this is a matter addressed to the sound discretion of the PSE, a corporation entity, whose business judgments are respected in the absence of bad faith. The question as to what policy is, or should be relied upon in approving the registration and sale of securities in the SEC is not for the Court to determine, but is left to the sound discretion of the Securities and Exchange Commission. In mandating the SEC to administer the Revised Securities Act, and in performing its other functions under pertinent laws, the Revised Securities Act, under Section 3 thereof, gives the SEC the power to promulgate such rules and regulations as it may consider appropriate in the public interest for the enforcement of the said laws. The second paragraph of Section 4 of the said law, on the other hand, provides that no security, unless exempt by law, shall be issued, endorsed, sold, transferred or in any other manner conveyed to the public, unless registered in accordance with the rules and regulations that shall be promulgated in the public interest and for the protection of investors by the Commission. Presidential Decree No. 902-A, on the other hand, provides that the SEC, as regulatory agency, has supervision and control over all corporations and over the securities market as a whole, and as such, is given ample authority in determining appropriate policies. Pursuant to this regulatory authority, the SEC has manifested that it has adopted the policy of "full material disclosure" where all companies, listed or applying for listing, are required to divulge truthfully and accurately, all material information about themselves and the securities they sell, for the protection of the investing public, and under pain of administrative, criminal and civil sanctions. In connection with this, a fact is deemed material if it tends to induce or otherwise effect the sale or purchase of its securities. 15 While the employment of this policy is recognized and sanctioned by the laws, nonetheless, the Revised Securities Act sets substantial and procedural standards which a proposed issuer of securities must satisfy.

A reading of the foregoing grounds reveals the intention of the lawmakers to make the registration and issuance of securities dependent, to a certain extent, on the merits of the securities themselves, and of the issuer, to be determined by the Securities and Exchange Commission. This measure was meant to protect the interests of the investing public against fraudulent and worthless securities, and the SEC is mandated by law to safeguard these interests, following the policies and rules therefore provided. The absolute reliance on the full disclosure method in the registration of securities is, therefore, untenable. As it is, the Court finds that the private respondent PALI, on at least two points (nos. 1 and 5) has failed to support the propriety of the issue of its shares with unfailing clarity, thereby lending support to the conclusion that the PSE acted correctly in refusing the listing of PALI in its stock exchange. This does not discount the effectivity of whatever method the SEC, in the exercise of its vested authority, chooses in setting the standard for public offerings of corporations wishing to do so. However, the SEC must recognize and implement the mandate of the law, particularly the Revised Securities Act, the provisions of which cannot be amended or supplanted by mere administrative issuance. In resume, the Court finds that the PSE has acted with justified circumspection, discounting, therefore, any imputation of arbitrariness and whimsical animation on its part. Its action in refusing to allow the listing of PALI in the stock exchange is justified by the law and by the circumstances attendant to this case.

LITTON MILLS INC. vs COURT of APPEALS GR NO. 9488 May 15,1996 Facts: Petitioner Litton Mills, Inc. (Litton) entered into an agreement with Empire Sales Philippines Corporation (Empire), as local agent of private respondent Gelhaar Uniform Company (Gelhaar), a corporation organized under the laws of the United States, whereby Litton agreed to supply Gelhaar 7,770 dozens of soccer jerseys. The agreement stipulated that

before it could collect from the bank on the letter of credit, Litton must present an inspection certificate issued by Gelhaar's agent in the Philippines, Empire Sales, that the goods were in satisfactory condition. Litton sent four shipments totalling 4,770 dozens of the soccer jerseys between December 2 and December 30, 1983. A fifth shipment, consisting of 2,110 dozens of the jerseys, was inspected by Empire from January 9 to January 19, 1984, but Empire refused to issue the required certificate of inspection. Alleging that Empire's refusal to issue a certificate was without valid reason, Litton filed a complaint with the Regional Trial Court of Pasig (Branch 158) on January 23, 1984, for specific performance and prayed for the issuance of writ of preliminary injunction to compel Empire to issue the inspection certificate covering the 2,110 dozens of jerseys and for damages. The trial court issued the writ which compelled Empire to issue the inspection certificate. Atty. Noval filed on behalf of Gelhaar Company a motion for extension of time to file an answer/responsive pleading. Afterwards, the law firm of Sycip, Salazar, Feliciano and Hernandez entered a special appearance for the purpose of objecting to the jurisdiction of the court over Gelhaar. On February 4, 1985, it moved to dismiss the case and to quash the summons on the ground that Gelhaar was a foreign corporation not doing business in the Philippines, and as such, was beyond the reach of the local courts. It contended that Litton failed to allege and prove that Gelhaar was doing business in the Philippines, which they argued was required by the ruling in Pacific Micronisian Lines, Inc., v. Del Rosario, 1 before summons could be served under Rule 14, 14. It likewise denied the authority of Atty. Noval to appear for Gelhaar and contended that the answer filed by Atty. Noval on June 15, 1984 could not bind Gelhaar and its filing did not amount to Gelhaar's submission to the jurisdiction of the court. Litton opposed the motion. On the other hand, Empire moved to dismiss on the ground of failure of the complaint to state a cause of action since the complaint alleged that Empire only acted as agent of Gelhaar; that it was made party-defendant only for the purpose of securing the issuance of an inspection certificate; and that it had already issued such certificate and the shipment had already been shipped on time. the trial court issued an order denying for lack of merit Gelhaar's motion to dismiss and to quash the summons. It held that Gelhaar was doing business in the Philippines, and that the service of summons on Gelhaar was

therefore valid. Gelhaar filed a motion for reconsideration, but its motion was denied. On appeal, the Court of Appeals reversed the ruling of trial court. The appellate court held that neither did the trial court acquire jurisdiction over Gelhaar through voluntary submission because the authority of Atty. Noval to represent Gelhaar had been questioned. Hence, this appeal. Issue: Whether or not respondent Gelhaar Uniform Company is doing business for the purpose of applying Rule 14, Section 14 of Rules of Court? Held: Yes. The trial court acquired jurisdiction over Gelhaar by service of summons upon its agent pursuant to Rule 14, 14. First. The appellate court invoked the ruling in Pacific Micronisian, in which it was stated that the fact of doing business must first be established before summons can be served in accordance with Rule 14, 14. The Court of Appeals quoted the following portion of the opinion in that case: The above section [referring to Rule 14, Section 14] provides for three modes of effecting service upon a private corporation, namely: [enumerates the three modes of service of summons]. But, it should be noted, in order that service may be effected in the manner above stated said section also requires that the foreign corporation be one which is doing business in the Philippines. This is a sine qua non requirement. This fact must first be established in order that summons can made and jurisdiction acquired. (Emphasis by the Court of Appeals) In the later case of Signetics Corporation v. Court of Appeals, 3 however, we clarified the holding in Pacific Micronisian, thus: The petitioner opines that the phrase, "(the) fact (of doing business in the Philippines) must first be established in order that summons be made and jurisdiction acquired," used in the above pronouncement, would indicate that a mere allegation to that effect in the complaint is not enough there must instead be proof of doing business. In any case, the petitioner points out, the allegations themselves did not sufficiently show the fact of its doing business in the Philippines.

It should be recalled that jurisdiction and venue of actions are, as they should so be, initially determined by the allegations of the complaint. Jurisdiction cannot be made to depend on independent pleas set up in a mere motion to dismiss, otherwise jurisdiction would become dependent almost entirely upon the defendant. The fact of doing business must then, in the first place, be established by appropriate allegations in the complaint. This is what the Court should be seen to have meant in the Pacific Micronisian case. The complaint, it is true, may have been vaguely structured but, taken correlatively, not disjunctively as the petitioner would rather suggest, it is not really so weak as to be fatally deficient in the above requirement. Hence, a court need not go beyond the allegations in the complaint to determine whether or not a defendant foreign corporation is doing business for the purpose of Rule 14, 14. In the case at bar, the allegation that Empire, for and in behalf of Gelhaar, ordered 7,770 dozens of soccer jerseys from Litton and for this purpose Gelhaar caused the opening of an irrevocable letter of credit in favor of Litton is a sufficient allegation that Gelhaar was doing business in the Philippines. Second. Gelhaar contends that the contract with Litton was a single, isolated transaction and that it did not constitute "doing business." Reference is made to Pacific Micronisian in which the only act done by the foreign company was to employ a Filipino as a member of the crew on one of its ships. This court held that the act was an isolated, incidental or casual transaction, not sufficient to indicate a purpose to engage in business. It is not really the fact that there is only a single act done that is material. The other circumstances of the case must be considered. Thus, in Wang Laboratories, Inc. v. Mendoza, 4 it was held that where a single act or transaction of a foreign corporation is not merely incidental or casual but is of such character as distinctly to indicate a purpose on the part of the foreign corporation to do other business in the state, such act will be considered as constituting doing business. 5 This Court referred to acts which were in the ordinary course of business of the foreign corporation.

In the case at bar, the trial court was certainly correct in holding that Gelhaar' s act in purchasing soccer jerseys to be within the ordinary course of business of the company considering that it was engaged in the manufacture of uniforms. The acts noted above are of such a character as to indicate a purpose to do business. In accordance with Rule 14, 14, service upon Gelhaar could be made in three ways: (1) by serving upon the agent designated in accordance with law to accept service of summons; (2) if there is no resident agent, by service on the government official designated by law to that effect; and (3) by serving on any officer or agent of said corporation within the Philippines. 6 Here, service was made through Gelhaar's agent, the Empire Sales Philippines Corp. There was, therefore, a valid service of summons on Gelhaar, sufficient to confer on the trial court jurisdiction over the person of Gelhaar. MR HOLDINGS INC. vs. BAJAR GR No. 138104 April 11,2002 Facts: Marcopper Mining Corporation, a domestic corporation whose 40% share capital are owned by Placer Dome Inc., obtained a loan from Asian Development Bank through the support of Place Dome Inc. Placer Dome Inc. and its subsidiary MR Holdings Inc. are both foreign corporations. To secure the loan, Marcopper executed in favor of ADB a deed of real estate and chattel mortgage. Upon default of payment by Marcopper, Placer Dome in fulfillment of its undertaking to support the loan agreed to have its subsidiary MR Holdings assumed Marcoppers obligation. Consequently, in an "Assignment Agreement"7 dated March 20, 1997, ADB assigned to petitioner all its rights, interests and obligations under the principal and complementary loan agreements. Meanwhile, it appeared that Solidbank Corporation obtained partial judgment against Marcopper from another RTC and moved for the issuance of writ of execution pending appeal. Thereafter, respondent Bajar issued two notices of levy on Marcoppers personal and real properties, and over all its stocks of scrap iron and unserviceable mining equipment. Together with sheriff Ferdinand M.

Jandusay (also a respondent) of the RTC, Branch 94, Boac, Marinduque, respondent Bajar issued two notices setting the public auction sale of the levied properties on August 27, 1998 at the Marcopper mine site. Having learned of the scheduled auction sale, petitioner MR Holdings Inc. served an "Affidavit of Third-Party Claim" upon respondent sheriffs on August 26, 1998, asserting its ownership over all Marcoppers mining properties, equipment and facilities by virtue of the "Deed of Assignment." Upon the denial of its "Affidavit of ThirdParty Claim" by the RTC of Manila, petitioner commenced with the RTC of Boac, Marinduque, presided by Judge Leonardo P. Ansaldo, a complaint for reivindication of properties, etc., with prayer for preliminary injunction and temporary restraining order against respondents Solidbank, Marcopper, and sheriffs Bajar and Jandusay. In an Order dated October 6, 1998, Judge Ansaldo denied petitioners application for a writ of preliminary injunction on the ground that a) petitioner has no legal capacity to sue, it being a foreign corporation doing business in the Philippines without license; b) an injunction will amount "to staying the execution of a final judgment by a court of co-equal and concurrent jurisdiction;" and c) the validity of the "Assignment Agreement" and the "Deed of Assignment" has been "put into serious question by the timing of their execution and registration." Unsatisfied, petitioner appealed before the Court of Appeals which affirmed the decision of RTC. Hence, this appeal. Issue: Whether or not petitioner has legal capacity to sue and seek redress from Philippine courts as it is a non-resident foreign corporations not doing business in the Philippines and suing on isolated transactions. Held: Yes. The Court of Appeals ruled that petitioner has no legal capacity to sue in the Philippine courts because it is a foreign corporation doing business here without license. A review of this ruling does not pose much complexity as the principles governing a foreign corporations right to sue in local courts have long been settled by our Corporation Law. These principles may be condensed in three statements, to wit: a) if a foreign corporation does business in the Philippines without a license, it cannot sue before the Philippine courts; b) if a foreign corporation is not doing business in the Philippines, it needs no license to sue before Philippine courts on an isolated transaction or on a cause of action entirely

independent of any business transaction; and c) if a foreign corporation does business in the Philippines with the required license, it can sue before Philippine courts on any transaction. Apparently, it is not the absence of the prescribed license but the "doing (of) business" in the Philippines without such license which debars the foreign corporation from access to our courts. The task at hand requires us to weigh the facts vis--vis the established principles. The question whether or not a foreign corporation is doing business is dependent principally upon the facts and circumstances of each particular case, considered in the light of the purposes and language of the pertinent statute or statutes involved and of the general principles governing the jurisdictional authority of the state over such corporations. Batas Pambansa Blg. 68, otherwise known as "The Corporation Code of the Philippines," is silent as to what constitutes doing" or "transacting" business in the Philippines. Fortunately, jurisprudence has supplied the deficiency and has held that the term "implies a continuity of commercial dealings and arrangements, and contemplates, to that extent, the performance of acts or works or the exercise of some of the functions normally incident to, and in progressive prosecution of, the purpose and object for which the corporation was organized." 23 In Mentholatum Co. Inc., vs. Mangaliman,24 this Court laid down the test to determine whether a foreign company is "doing business," thus: " x x x The true test, however, seems to be whether the foreign corporation is continuing the body or substance of the business or enterprise for which it was organized or whether it has substantially retired from it and turned it over to another. (Traction Cos. vs. Collectors of Int. Revenue [C.C.A., Ohio], 223 F. 984,987.) x x x." The traditional case law definition has metamorphosed into a statutory definition, having been adopted with some qualifications in various pieces of legislation in our jurisdiction. For instance, Republic Act No. 7042, otherwise known as the "Foreign Investment Act of 1991," defines "doing business" as follows:

"d) The phrase doing business shall include soliciting orders, service contracts, opening offices, whether called liaison offices or branches; appointing representatives or distributors domiciled in the Philippines or who in any calendar year stay in the country for a period or periods totalling one hundred eight(y) (180) days or more; participating in the management, supervision or control of any domestic business, firm, entity, or corporation in the Philippines; and any other act or acts that imply a continuity of commercial dealings or arrangements, and contemplate to that extent the performance of acts or works; or the exercise of some of the functions normally incident to, and in progressive prosecution of, commercial gain or of the purpose and object of the business organization; Provided, however, That the phrase doing business shall not be deemed to include mere investment as a shareholder by a foreign entity in domestic corporations duly registered to do business, and/or the exercise of rights as such investor, nor having a nominee director or officer to represent its interests in such corporation, nor appointing a representative or distributor domiciled in the Philippines which transacts business in its own name and for its own account." (Emphasis supplied) 25 Likewise, Section 1 of Republic Act No. 5455,26 provides that: "SECTION. 1. Definition and scope of this Act. - (1) x x x the phrase doing business shall include soliciting orders, purchases, service contracts, opening offices, whether called liaison offices or branches; appointing representatives or distributors who are domiciled in the Philippines or who in any calendar year stay in the Philippines for a period or periods totaling one hundred eighty days or more; participating in the management, supervision or control of any domestic business firm, entity or corporation in the Philippines; and any other act or acts that imply a continuity of commercial dealings or arrangements, and contemplate to that extent the performance of acts or works, or the exercise of some of the functions normally incident to, and in progressive prosecution of, commercial gain or of the purpose and object of the business organization."

There are other statutes27 defining the term "doing business" in the same tenor as those above-quoted, and as may be observed, one common denominator among them all is the concept of "continuity." In the case at bar, the Court of Appeals categorized as "doing business" petitioners participation under the "Assignment Agreement" and the "Deed of Assignment." This is simply untenable. The expression "doing business" should not be given such a strict and literal construction as to make it apply to any corporate dealing whatever.28 At this early stage and with petitioners acts or transactions limited to the assignment contracts, it cannot be said that it had performed acts intended to continue the business for which it was organized. It may not be amiss to point out that the purpose or business for which petitioner was organized is not discernible in the records. No effort was exerted by the Court of Appeals to establish the nexus between petitioners business and the acts supposed to constitute "doing business." Thus, whether the assignment contracts were incidental to petitioners business or were continuation thereof is beyond determination. We cannot apply the case cited by the Court of Appeals, Far East Intl Import and Export Corp. vs. Nankai Kogyo Co., Ltd.,29 which held that a single act may still constitute "doing business" if "it is not merely incidental or casual, but is of such character as distinctly to indicate a purpose on the part of the foreign corporation to do other business in the state." In said case, there was an express admission from an official of the foreign corporation that he was sent to the Philippines to look into the operation of mines, thereby revealing the foreign corporations desire to continue engaging in business here. But in the case at bar, there is no evidence of similar desire or intent. Unarguably, petitioner may, as the Court of Appeals suggested, decide to operate Marcoppers mining business, but, of course, at this stage, that is a mere speculation. Or it may decide to sell the credit secured by the mining properties to an offshore investor, in which case the acts will still be isolated transactions. To see through the present facts an intention on the part of petitioner to start a series of business transaction is to rest on assumptions or probabilities falling short of actual proof. Courts should never base its judgments on a state of facts so inadequately developed that it cannot be determined where inference ends and conjecture begins.

Indeed, the Court of Appeals holding that petitioner was determined to be "doing business" in the Philippines is based mainly on conjectures and speculation. In concluding that the "unmistakable intention" of petitioner is to continue Marcoppers business, the Court of Appeals hangs on the wobbly premise that "there is no other way for petitioner to recover its huge financial investments which it poured into Marcoppers rehabilitation without it (petitioner) continuing Marcoppers business in the country." 30 This is a mere presumption. Absent overt acts of petitioner from which we may directly infer its intention to continue Marcoppers business, we cannot give our concurrence. Significantly, a view subscribed upon by many authorities is that the mere ownership by a foreign corporation of a property in a certain state, unaccompanied by its active use in furtherance of the business for which it was formed, is insufficient in itself to constitute doing business.31 In Chittim vs. Belle Fourche Bentonite Products Co.,32 it was held that even if a foreign corporation purchased and took conveyances of a mining claim, did some assessment work thereon, and endeavored to sell it, its acts will not constitute the doing of business so as to subject the corporation to the statutory requirements for the transacting of business. On the same vein, petitioner, a foreign corporation, which becomes the assignee of mining properties, facilities and equipment cannot be automatically considered as doing business, nor presumed to have the intention of engaging in mining business. One important point. Long before petitioner assumed Marcoppers debt to ADB and became their assignee under the two assignment contracts, there already existed a "Support and Standby Credit Agreement" between ADB and Placer Dome whereby the latter bound itself to provide cash flow support for Marcoppers payment of its obligations to ADB. Plainly, petitioners payment of US$ 18,453,450.12 to ADB was more of a fulfillment of an obligation under the "Support and Standby Credit Agreement" rather than an investment. That petitioner had to step into the shoes of ADB as Marcoppers creditor was just a necessary legal consequence of the transactions that transpired. Also, we must hasten to add that the "Support and Standby Credit Agreement" was executed four (4) years prior to Marcoppers insovency, hence, the alleged "intention of petitioner to continue Marcoppers business" could have no basis for at that time, Marcoppers fate cannot yet be determined.

In the final analysis, we are convinced that petitioner was engaged only in isolated acts or transactions. Single or isolated acts, contracts, or transactions of foreign corporations are not regarded as a doing or carrying on of business. Typical examples of these are the making of a single contract, sale, sale with the taking of a note and mortgage in the state to secure payment therefor, purchase, or note, or the mere commission of a tort.33 In these instances, there is no purpose to do any other business within the country.

ABACUS SECURITIES CORPORATION VS. RUBEN AMPIL G.R. NO. 160016 Panganiban, C.J. : Facts: Ruben Ampil opened a cash account with Abacus Securities Corporation (Abacus, for brevity). Abacus is engaged in a business as a broker and dealer of securities of listed companies at the Philippine Stock Exchange Center. Ampil actively traded his account, and as a result of such trading activities, he accumulated an outstanding obligation in favor of Abacus in the sum of P6,617,036.22. The purchases of Ampil were continuously unpaid, but notwithstanding Ampils failure to cover his initial deficiency, Abacus subsequently purchased and sold securities for Ampils account. Abacus did not cancel or liquidate a substantial amount of Ampils stock transactions. The RTC of Makati City ruled that Abacus violated Secs. 23 and 25 of the Revised Securities Act (SRA) when Abacus failed to require Ampil to pay for his stock purchases within 3 or 4 days from trading and request from the appropriate authority an extension of time for the payment of Ampils cash purchases. According to the lower court, by allowing Ampli to trade his account actively without cash, Abacus effectively induced him to purchase securities thereby incurring excessive credits, hence, both Abacus and Ampil were in pari delicto and they are without recourse against each other. FEBRUARY 27, 2006

Issue: Whether or not both Abacus and Ampil violated the provisions of RSA, hence the pari delicto rule can be applied. Ruling: Yes. In the present case, respondent cannot escape payment of stocks validly traded by petitioner on his behalf. These transactions took place before both parties violated the trading law and rules. Hence, they fall outside the purview of the pari delicto rule. The law places the burden of compliance with margin requirements primarily upon the brokers and dealers. Sections 23 and 25 and Rule 25-1, otherwise known as the "mandatory close-out rule, clearly vest upon petitioner the obligation, not just the right, to cancel or otherwise liquidate a customers order, if payment is not received within three days from the date of purchase. For transactions subsequent to an unpaid order, the broker should require its customer to deposit funds into the account sufficient to cover each purchase transaction prior to its execution. These duties are imposed upon the broker to ensure faithful compliance with the margin requirements of the law, which forbids a broker from extending undue credit to a customer. The obligation of respondent for stock transactions made and entered into on April 10 and 11, 1997 remains outstanding. These transactions were valid and the obligations incurred by respondent concerning his stock purchases on these dates subsist. At the time, there was no violation of the RSA yet. In the final analysis, both parties acted in violation of the law and did not come to court with clean hands with regard to transactions subsequent to the initial trades made on April 10 and 11, 1997. Thus, the peculiar facts of the present case bar the application of the pari delicto rule -- expressed in the maxims "Ex dolo malo non oritur action" and "In pari delicto potior est conditio defendentis" -- to all the transactions entered into by the parties. The pari delecto rule refuses legal remedy to either party to an illegal agreement and leaves them where they were. In this case, the pari delicto rule applies only to transactions entered into after the initial trades made on April 10 and 11, 1997. Since the initial trades are valid and subsisting obligations, respondent is liable for them. Justice and good conscience require all persons to satisfy their debts. Ours are courts of both law and equity; they compel fair dealing; they do not abet clever attempts to escape just obligations. Ineludibly, this Court would not hesitate to grant relief in accordance with good faith and conscience.

Fallo: WHEREFORE, the assailed Decision and Resolution of the Court of Appeals are hereby MODIFIED. Respondent is ordered to pay petitioner the difference between the formers outstanding obligation as of April 11, 1997 less the proceeds from the mandatory sell out of shares pursuant to the RSA Rules, with interest thereon at the legal rate until fully paid. EXPERTRAVEL & TOURS, INC. vs. COURT OF APPEALS and KOREAN AIRLINES G.R. No. 152392 May 26, 2005 CALLEJO, SR., J.: Facts: Korean Airlines (KAL) is a corporation established and registered in the Republic of South Korea and licensed to do business in the Philippines. Its general manager in the Philippines is Suk Kyoo Kim, while its appointed counsel was Atty. Mario Aguinaldo and his law firm. KAL, through Atty. Aguinaldo, filed a Complaint against Expert Travel & Tours, Inc. (ETI) with the RTC of Manila, for the collection of the principal amount of P260,150.00, plus attorneys fees and exemplary damages. The verification and certification against forum shopping was signed by Atty. Aguinaldo, who indicated therein that he was the resident agent and legal counsel of KAL and had caused the preparation of the complaint. ETI, on the other hand, filed a motion to dismiss the complaint on the ground that Atty. Aguinaldo was not authorized to execute the verification and certificate of non-forum shopping as required by Section 5, Rule 7 of the Rules of Court. KAL opposed the motion, contending that Atty. Aguinaldo was its resident agent and was registered as such with the Securities and Exchange Commission (SEC) as required by the Corporation Code of the Philippines. During the hearing of the case, Atty. Aguinaldo claimed that he had been authorized to file the complaint through a resolution of the KAL Board of Directors approved during a special meeting.

Finally, KAL submitted an Affidavit executed by its general manager Suk Kyoo Kim, alleging that the board of directors conducted a special teleconference and in that same teleconference, the board of directors approved a resolution authorizing Atty. Aguinaldo to execute the certificate of non-forum shopping and to file the complaint. Suk Kyoo Kim also alleged, however, that the corporation had no written copy of the aforesaid resolution. Issue: Whether or not Atty. Aguinaldo, as the resident agent of KAL has the authority to sign the certificate on non-forum shopping. Ruling: No. In a case where the plaintiff is a private corporation, the certification may be signed, for and on behalf of the said corporation, by a specifically authorized person, including its retained counsel, who has personal knowledge of the facts required to be established by the documents. In turn, a corporation exercises said powers through its board of directors and/or its duly-authorized officers and agents. Physical acts, like the signing of documents, can be performed only by natural persons dulyauthorized for the purpose by corporate by-laws or by specific act of the board of directors. Indeed, the certificate of non-forum shopping may be incorporated in the complaint or appended thereto as an integral part of the complaint. The rule is that compliance with the rule after the filing of the complaint, or the dismissal of a complaint based on its non-compliance with the rule, is impermissible. However, in exceptional circumstances, the court may allow subsequent compliance with the rule. If the authority of a partys counsel to execute a certificate of non-forum shopping is disputed by the adverse party, the former is required to show proof of such authority or representation. In this case, the petitioner, as the defendant in the RTC, assailed the authority of Atty. Aguinaldo to execute the requisite verification and certificate of non-forum shopping as the resident agent and counsel of the

respondent. It was, thus, incumbent upon the respondent, as the plaintiff, to allege and establish that Atty. Aguinaldo had such authority to execute the requisite verification and certification for and in its behalf. The respondent, however, failed to do so. As gleaned from the aforequoted certification, there was no allegation that Atty. Aguinaldo had been authorized to execute the certificate of non-forum shopping by the respondents Board of Directors; moreover, no such board resolution was appended thereto or incorporated therein. While Atty. Aguinaldo is the resident agent of the respondent in the Philippines, this does not mean that he is authorized to execute the requisite certification against forum shopping. Under Section 127, in relation to Section 128 of the Corporation Code, the authority of the resident agent of a foreign corporation with license to do business in the Philippines is to receive, for and in behalf of the foreign corporation, services and other legal processes in all actions and other legal proceedings against such corporation. Under the law, Atty. Aguinaldo was not specifically authorized to execute a certificate of non-forum shopping as required by Section 5, Rule 7 of the Rules of Court. This is because while a resident agent may be aware of actions filed against his principal (a foreign corporation doing business in the Philippines), such resident may not be aware of actions initiated by its principal, whether in the Philippines against a domestic corporation or private individual, or in the country where such corporation was organized and registered, against a Philippine registered corporation or a Filipino citizen. The respondent knew that its counsel, Atty. Aguinaldo, as its resident agent, was not specifically authorized to execute the said certification. It attempted to show its compliance with the rule subsequent to the filing of its complaint by submitting, on March 6, 2000, a resolution purporting to have been approved by its Board of Directors during a teleconference held on

June 25, 1999, allegedly with Atty. Aguinaldo and Suk Kyoo Kim in attendance. NARVASA, C.J.: However, such attempt of the respondent casts veritable doubt not only on its claim that such a teleconference was held, but also on the approval by the Board of Directors of the resolution authorizing Atty. Aguinaldo to execute the certificate of non-forum shopping. In the Philippines, teleconferencing and videoconferencing of members of board of directors of private corporations is a reality, in light of Republic Act No. 8792. The Securities and Exchange Commission issued SEC Memorandum Circular No. 15, on November 30, 2001, providing the guidelines to be complied with related to such conferences. Thus, the Court agrees with the RTC that persons in the Philippines may have a teleconference with a group of persons in South Korea relating to business transactions or corporate governance. Even given the possibility that Atty. Aguinaldo and Suk Kyoo Kim participated in a teleconference along with the respondents Board of Directors, the Court is not convinced that one was conducted; even if there had been one, the Court is not inclined to believe that a board resolution was duly passed specifically authorizing Atty. Aguinaldo to file the complaint and execute the required certification against forum shopping. Fallo: IN LIGHT OF ALL THE FOREGOING, the petition is GRANTED. The Decision of the Court of Appeals in CA-G.R. SP No. 61000 is REVERSED and SET ASIDE. The Regional Trial Court of Manila is hereby ORDERED to dismiss, without prejudice, the complaint of the respondent.

G.R. No. 97816, July 24, 1992

The capacity of a foreign corporation to maintain an action in the Philippines against residents thereof, is the principal question in the appellate proceedings at bar. Facts: Merrill Lynch Futures, Inc. (MLF) filed a complaint with the RTC of Quezon City against the Spouses Pedro Lara and Elisa Lara for the recovery of a debt and interest thereon, damages, and attorney's fees. It also defined a "futures contract" as a "contractual commitment to buy and sell a standardized quantity of a particular item at a specified future settlement date and at a price agreed upon, with the purchase or sale being executed on a regulated futures exchange." MLF alleged that: (1) MLF entered into a Futures Customer Agreement with spouses Lara, in virtue of which it agreed to act as the latter's broker for the purchase and sale of futures contracts in the U.S.; (2) that pursuant to the contract, orders to buy and sell futures contracts were transmitted to MLFby the Lara Spouses "through the facilities of Merrill Lynch Philippines, Inc., a Philippine corporation and a company servicing plaintiffs customers;" (3) that from the outset, the Lara Spouses "knew and were duly advised that Merrill Lynch Philippines, Inc. was not a broker in futures contracts," and that it "did not have a license from the Securities and Exchange Commission to operate as a commodity trading advisor (i.e., an entity which, not being a broker, furnishes advice on commodity futures to persons who trade in futures contracts);

MERRILL LYNCH FUTURES, INC. VS. HON. COURT OF APPEALS, AND THE SPOUSES PEDRO M. LARA AND ELISA G. LARA

(4) that in line with the above mentioned agreement and through said Merrill Lynch Philippines, Inc., the Lara Spouses actively traded in futures contracts, including "stock index futures" for four years or so; (5) that because of a loss amounting to US$160,749.69 incurred in respect of three (3) transactions involving "index futures," and after setting this off against an amount of US$75,913.42 then owing by ML FUTURES to the Lara Spouses, said spouses became indebted to ML FUTURES for the ensuing balance of US$84,836.27, which the latter asked them to pay; and (6) that the Lara Spouses however refused to pay this balance, alleging that the transactions were null and void because Merrill Lynch Philippines, Inc., the Philippine company servicing accounts of plaintiff, ** had no license to operate as a commodity and/or financial futures broker." However, spouses Lara filed a motion to dismiss based on the following grounds: a) although not licensed to do so, MLF had been doing business in the Philippines "at least for the last 4 years," this being clear from the very allegations of the complaint; consequently, ML FUTURES is prohibited by law "to maintain or intervene in any action, suit or proceeding in any court or administrative agency of the Philippines;" and b) they had never been informed that Merrill Lynch Philippines, Inc. was not licensed to do business in this country; and contrary to the allegations of the complaint, all their transactions had actually been with MERRILL LYNCH PIERCE FENNER & SMITH, INC., and not with ML FUTURES (Merrill Lynch Futures, Inc.), in proof of which they attached to their motion to dismiss copies of 8 agreements, receipts or reminders, etc., executed on standard printed forms of said Merrill Lynch Pierce Fenner & Smith Inc. The RTC sustained the motion to dismiss, directing the dismissal of the case and discharging the writ of preliminary attachment.

On appeal to the Court of Appeals, the appellate court the decision of the RTC. Issue: Whether or not Merrill Lynch Futures, Inc. (MLF), being a foreign corporation not licensed to do business in the Philippines, has the capacity to maintain an action in the Philippines against residents thereof (spouses Lara). Ruling: Yes. The Court is satisfied that the facts on record adequately establish that ML FUTURES, operating in the United States, had indeed done business with the Lara Spouses in the Philippines over several years, had done so at all times through Merrill Lynch Philippines, Inc. (MLPI), a corporation organized in this country and had executed all these transactions without ML FUTURES being licensed to so transact business here, and without MLPI being authorized to operate as a commodity futures trading advisor. These are the factual findings of both the Trial Court and Appeals. These, too, are the conclusions of the Securities Commission which denied MLPI's application to operate as futures trading advisor, a denial subsequently affirmed by Appeals. the Court of & Exchange a commodity the Court of

Prescinding from the proposition that factual findings of the Court of Appeals are generally conclusive, this Court has been cited to no circumstance of substance to warrant reversal of said Appellate Court's findings or conclusions in this case. The Court is satisfied, too, that the Laras did transact business with ML FUTURES through its agent corporation organized in the Philippines, it being unnecessary to determine whether this domestic firm was MLPI (Merrill Lynch Philippines, Inc.) or Merrill Lynch Pierce Fenner & Smith, (MLPI's alleged predecessor). The fact is that ML FUTURES did deal with futures contracts in exchanges in the United States in behalf and for the account of the Lara Spouses, and

that on several occasions the latter received account documents and money in connection with those transactions. In other words, if it be true that during all the time that they were transacting with ML FUTURES, the Laras were fully aware of its lack of license to do business in the Philippines, and in relation to those transactions had made payments to, and received money from it for several years, the question is whether or not the Lara Spouses are now estopped to impugn ML FUTURES capacity to sue them in the courts of the forum. The rule is that a party is estopped to challenge the personality of a corporation after having acknowledged the same by entering into a contract with it. And the "doctrine of estoppel to deny corporate existence applies to foreign as well as to domestic corporations;" one who has dealt with a corporation of foreign origin as a corporate entity is estopped to deny its corporate existence and capacity. The principle "will be applied to prevent a person contracting with a foreign corporation from later taking advantage of its noncompliance with the statutes, chiefly in cases where such person has received the benefits of the contract where such person has acted as agent for the corporation and has violated his fiduciary obligations as such, and where the statute does not provide that the contract shall be void, but merely fixes a special penalty for violation of the statute. The general rule that in the absence of fraud a person who has contracted or otherwise dealt with an association in such a way as to recognize and in effect admit its legal existence as a corporate body is thereby estopped to deny its corporate existence in any action leading out of or involving such contract or dealing, unless its existence is attacked for causes which have arisen since making the contract or other dealing relied on as an estoppel and this applies to foreign as well as domestic corporations. Considerations of equity dictate that, at the very least, the issue of whether the Laras are in truth liable to ML FUTURES and if so in what amount, and

whether they were so far aware of the absence of the requisite licenses on the part of ML FUTURES and its Philippine correspondent, MLPI, as to be estopped from alleging that fact as a defense to such liability, should be ventilated and adjudicated on the merits by the proper trial court. Fallo: WHEREFORE, the decision of the Court of Appeals in CA-G.R. CV No. 16478 dated November 27, 1990 and its Resolution of March 7, 1991 are REVERSED and SET ASIDE, and the Regional Trial Court at Quezon City, Branch 84, is ORDERED to reinstate Civil Case No. Q-52360 and forthwith conduct a hearing to adjudicate the issues set out in the preceding paragraph on the merits. B. VAN ZUIDEN BROS., LTD vs. GTVL MANUFACTURING INDUSTRIES, INC., G.R. No. 147905 May 28, 2007

CARPIO, J.: Facts: On several occasions, GTVL purchased lace products from Zuiden, a corporation, incorporated under the laws of Hong Kong, not engaged in business in the Philippines, but is suing before the Philippine Courts. The procedure for these purchases, as per the instructions of GTVL, was that ZUIDEN delivers the products purchased by GTVL, to a certain Hong Kong corporation, known as Kenzar Ltd. and the products are then considered as sold, upon receipt by KENZAR of the goods purchased by GTVL. KENZAR had the obligation to deliver the products to the Philippines and/or to follow whatever instructions GTVL had on the matter. Insofar as ZUIDEN is concerned, upon delivery of the goods to KENZAR in Hong Kong, the transaction is concluded; and GTVL became obligated to pay the agreed purchase price. However, commencing October 31, 1994 up to the present, GTVL has failed and refused to pay the agreed purchase price for several deliveries ordered by it and delivered by ZUIDEN.

Instead of filing an answer, respondent filed a Motion to Dismiss on the ground that petitioner has no legal capacity to sue. Respondent alleged that petitioner is doing business in the Philippines without securing the required license. Accordingly, petitioner cannot sue before Philippine courts. The trial court issued an Order on 10 November 1999 dismissing the complaint. On appeal, the Court of Appeals sustained the trial courts dismissal of the complaint. Issue: Whether or not Zuiden is doing business in the Philippines

countries. The mere act of exporting from ones own country, without doing any specific commercial act within the territory of the importing country, cannot be deemed as doing business in the importing country. The importing country does not acquire jurisdiction over the foreign exporter who has not performed any specific commercial act within the territory of the importing country. Without jurisdiction over the foreign exporter, the importing country cannot compel the foreign exporter to secure a license to do business in the importing country. Otherwise, Philippine exporters, by the mere act alone of exporting their products, could be considered by the importing countries to be doing business in those countries. This will require Philippine exporters to secure a business license in every foreign country where they usually export their products, even if they do not perform any specific commercial act within the territory of such importing countries. Such a legal concept will have a deleterious effect not only on Philippine exports, but also on global trade. To be doing or transacting business in the Philippines for purposes of Section 133 of the Corporation Code, the foreign corporation must actually transact business in the Philippines, that is, perform specific business transactions within the Philippine territory on a continuing basis in its own name and for its own account. Actual transaction of business within the Philippine territory is an essential requisite for the Philippines to acquire jurisdiction over a foreign corporation and thus require the foreign corporation to secure a Philippine business license. If a foreign corporation does not transact such kind of business in the Philippines, even if it exports its products to the Philippines, the Philippines has no jurisdiction to require such foreign corporation to secure a Philippine business license.

Held: Section 133 of the Corporation Code is clear. An unlicensed foreign corporation doing business in the Philippines cannot sue before Philippine courts. On the other hand, an unlicensed foreign corporation not doing business in the Philippines can sue before Philippine courts. In the present controversy, petitioner is a foreign corporation which claims that it is not doing business in the Philippines. As such, it needs no license to institute a collection suit against respondent before Philippine courts. The series of transactions between petitioner and respondent cannot be classified as doing business in the Philippines under Section 3(d) of RA 7042. An essential condition to be considered as doing business in the Philippines is the actual performance of specific commercial acts within the territory of the Philippines for the plain reason that the Philippines has no jurisdiction over commercial acts performed in foreign territories. Here, there is no showing that petitioner performed within the Philippine territory the specific acts of doing business mentioned in Section 3(d) of RA 7042. Petitioner did not also open an office here in the Philippines, appoint a representative or distributor, or manage, supervise or control a local business. While petitioner and respondent entered into a series of transactions implying a continuity of commercial dealings, the perfection and consummation of these transactions were done outside the Philippines. An exporter in one country may export its products to many foreign importing countries without performing in the importing countries specific commercial acts that would constitute doing business in the importing

AVON INSURANCE PLC, BRITISH RESERVE INSURANCE. CO. LTD. et.al. vs. COURT OF APPEALS, et.al. G.R. No. 97642 August 29, 1997 TORRES, JR., J.:

Facts: Yupangco Cotton Mills engaged to secure with Worldwide Security and Insurance Co. Inc., several of its properties for the periods July 6, 1979 to July 6, 1980 as under Policy No. 20719 for a coverage of P100,000,000.00 and from October 1, 1980 to October 1, 1981, under Policy No. 25896, also forP100,000,000.00. Both contracts were covered by reinsurance treaties between Worldwide Surety and Insurance and several foreign reinsurance companies, including the petitioners. The reinsurance arrangements had been made through international broker C.J. Boatright and Co. Ltd., acting as agent of Worldwide Surety and Insurance. The properties therein insured were razed by fire , thereby giving rise to the obligation of the insurer to indemnify the Yupangco Cotton Mills. Partial payments were made by Worldwide Surety and Insurance and some of the reinsurance companies. Worldwide Surety and Insurance, in a deed of Assignment, acknowledge a remaining balance of P19,444,447.75 still due Yupangco Cotton Mills, and assigned to the latter all reinsurance proceeds still collectible from all the foreign reinsurance companies. Thus, in its interest as assignee and original insured, Yupangco Cotton Mills instituted this collection suit against the petitioners. Service of summons upon the petitioners was made by notification to the Insurance Commissioner, pursuant to Section 14, Rule 14 of the Rules of Court Yupangco Cotton Mills filed a complaint against several foreign reinsurance companies (among which are petitioners) to collect their alleged percentage liability under contract treaties between the foreign insurance companies and the international insurance broker C.J. Boatright, acting as agent for respondent Worldwide Surety and Insurance Company. Inasmuch as petitioners are not engaged in business in the Philippines with no offices, places of business or agents in the Philippines, the reinsurance treaties having been rendered abroad, service of summons upon motion of respondent Yupangco, was made upon petitioners through the office of the Insurance Commissioner. Petitioners, by counsel on special appearance, seasonably filed motions to dismiss disputing the jurisdiction of RTC and the extra-territorial service of summons. Respondent Yupangco filed its opposition to the motion to dismiss, petitioners filed their reply, and

respondent Yupangco filed its rejoinder. RTC denied the motions to dismiss and directed petitioners to file their answer. Issue: Whether or not the petitioners were determined to be doing business in the Philippines Held: In relation with Article 44 of the Omnibus Investments Code of 1987, doing business ordinarily implies a continuity of commercial dealings and arrangements, and contemplates, to that extent, the performance of acts or works or the exercise of the functions normally incident to and in progressive prosecution of the purpose and object of its organization. A single act or transaction made in the Philippines, however, could not qualify a foreign corporation to be doing business in the Philippines, if such singular act is not merely incidental or casual, but indicates the foreign corporations intention to do business in the Philippines. There is no sufficient basis in the records which would merit the institution of this collection suit in the Philippines. More specifically, there is nothing to substantiate the private respondents submission that the petitioners had engaged in business activities in this country. This is not an instance where the erroneous service of summons upon the defendant can be cured by the issuance and service of alias summons, as in the absence of showing that petitioners had been doing business in the country, they cannot be summoned to answer for the charges leveled against them. The purpose of the law in requiring that foreign corporations doing business in the country be licensed to do so, is to subject the foreign corporations doing business in the Philippines to the jurisdiction of the courts, otherwise, a foreign corporation illegally doing business here because of its refusal or neglect to obtain the required license and authority to do business may successfully though unfairly plead such neglect or illegal act so as to avoid service and thereby impugn the jurisdiction of the local courts. There is no showing that petitioners had performed any act in the country that would place it within the sphere of the courts jurisdiction. The assertion that a resident of the Philippines will be inconvenienced by an outof-town suit against a foreign entity, is irrelevant and unavailing to sustain the continuance of a local action, for jurisdiction is not dependent upon the

convenience or inconvenience of a party. If the appearance of a party in a suit is precisely to question the jurisdiction of the said tribunal over the person of the defendant, then this appearance is not equivalent to service of summons, nor does is constitute an acquiescence to the courts jurisdiction. Thus it cannot be argued that the petitioners had abandoned their objections to the jurisdiction of the court, as their motions to dismiss in the trial court, and all their subsequent posturings, were all in protest of the private respondent's insistence on holding them so answer a charge in a forum where they believe they are not subject to. Clearly, to continue the proceedings in a case such as those before Us would just be useless and a waste of time. COLUMBIA PICTURES, INC., ORION PICTURES CORPORATION, et.al. vs. COURT OF APPEALS G.R. No. 110318 August 28, 1996 REGALADO, J.: Facts: Complainants lodged a formal complaint with the NBI for violation of PD No. 49 and sought its assistance in their anti-film piracy drive. Agents of the NBI and private researchers made discreet surveillance on various video establishments in Metro Manila including Sunshine Home Video Inc. (Sunshine for brevity), owned and operated by Danilo A. Pelindario. NBI Senior Agent Lauro C. Reyes applied for a search warrant with the trial court against Sunshine seeking the seizure, among others, for piracy. In the course of the search of the premises indicated in the search warrant, the NBI Agents found and seized various video tapes of duly copyrighted motion pictures/films owned or exclusively distributed by private complainants, and machines, equipment, television sets, paraphernalia, materials, accessories all of which were included in the receipt for properties accomplished by the raiding team. Copy of the receipt was furnished and/or tendered to Mr. Danilo A. Pelindario, registered owner-proprietor of Sunshine Home Video.

A Motion To Lift the Order of Search Warrant was filed but was later denied for lack of merit (p. 280, Records). The motion however wa sltaer granted by the trial court. Private respondents aver that being foreign corporations, petitioners should have such license to be able to maintain an action in Philippine courts under Section 1(f) (1) and (2), Rule 1 of the Rules of the Board of Investments as well as Section 133 of the Corporation Code. Issue: Whether or not private respondents are doing business in the Philippines Held: The obtainment of a license prescribed by Section 125 of the Corporation Code is not a condition precedent to the maintenance of any kind of action in Philippine courts by a foreign corporation. However, under the aforequoted provision, no foreign corporation shall be permitted to transact business in the Philippines, as this phrase is understood under the Corporation Code, unless it shall have the license required by law, and until it complies with the law in transacting business here, it shall not be permitted to maintain any suit in local courts. As thus interpreted, any foreign corporation not doing business in the Philippines may maintain an action in our courts upon any cause of action, provided that the subject matter and the defendant are within the jurisdiction of the court. It is not the absence of the prescribed license but doing business in the Philippines without such license which debars the foreign corporation from access to our courts. In other words, although a foreign corporation is without license to transact business in the Philippines, it does not follow that it has no capacity to bring an action. Such license is not necessary if it is not engaged in business in the Philippines. The true tests, however, seem to be whether the foreign corporation is continuing the body or substance of the business or enterprise for which it was organized or whether it has substantially retired from it and turned it over to another. As a general proposition upon which many authorities agree in principle, subject to such modifications as may be

necessary in view of the particular issue or of the terms of the statute involved, it is recognized that a foreign corporation is doing, transacting, engaging in, or carrying on business in the State when, and ordinarily only when, it has entered the State by its agents and is there engaged in carrying on and transacting through them some substantial part of its ordinary or customary business, usually continuous in the sense that it may be distinguished from merely casual, sporadic, or occasional transactions and isolated acts. Certainly, a corporation whose legal rights have been violated is undeniably such, if not the only, real party-in-interest to bring suit thereon although, for failure to comply with the licensing requirement, it is not capacitated to maintain any suit before our courts. The doctrine of lack of capacity to sue based on failure to first acquire a local license is based on considerations of public policy. It was never intended to favor nor insulate from suit unscrupulous establishments or nationals in case of breach of valid obligations or violations of legal rights of unsuspecting foreign firms or entities simply because they are not licensed to do business in the country. CARGILL, INC.,vs. INTRA STRATA ASSURANCE CORPORATION G.R. No. 168266 March 15, 2010

permitted to draw up to $500,000 representing the minimum price of the contract upon presentation of some documents. The contract was amended three times. In compliance with the terms of the third amendment of the contract, respondent Intra Strata Assurance Corporation issued a performance bondto guarantee NMCs delivery of the 10,500 tons of molasses, and a surety bond[to guarantee the repayment of downpayment as provided in the contract. NMC was only able to deliver 219.551 metric tons of molasses out of the agreed 10,500 metric tons. Thus, Cargil sent demand letters to respondent claiming payment under the performance and surety bonds. When respondent refused to pay, petitioner filed a complaint[for sum of money against NMC and respondent. Cargil, NMC, and respondent entered into a compromise agreement,[which the trial court approved. The compromise agreement provides that NMC would pay petitioner P3,000,000 upon signing of the compromise agreement and would deliver to petitioner 6,991 metric tons of molasses. However, NMC still failed to comply with its obligation under the compromise agreement. Hence, trial proceeded against respondent and the trial court ruled later for Cargil. On appeal, the Court of Appeals reversed the trial courts decision and dismissed the complaint. Issues:

CARPIO, J.: Facts: Cargill, Inc. is a corporation organized and existing under the laws of the State of Delaware, United States of America. Cargil and Northern Mindanao Corporation (NMC) executed a contract NMC agreed to sell to petitioner 20,000 to 24,000 metric tons of molasses, to be delivered from 1 January to 30 June 1990 at the price of $44 per metric ton. The contract provides that petitioner would open a Letter of Credit with the Bank of Philippine Islands. Under the red clause of the Letter of Credit, NMC was

Whether petitioner is doing or transacting business in the Philippines in contemplation of the law and established jurisprudence; 1 Held: Under Article 133 of the Corporation Code, a foreign corporation must first obtain a license and a certificate from the appropriate government agency before it can transact business in the Philippines. Where a foreign corporation does business in the Philippines without the proper license, it cannot maintain any action or proceeding before Philippine courts .

Republic Act No. 7042 (RA 7042), otherwise known as the Foreign Investments Act of 1991, which repealed Articles 44-56 of Book II of the Omnibus Investments Code of 1987, enumerated not only the acts or activities which constitute doing business but also those activities which are not deemed doing business. Section 3(d) of RA 7042 states: The phrase doing business shall include soliciting orders, service contracts, opening offices, whether called liaison offices or branches; appointing representatives or distributors domiciled in the Philippines or who in any calendar year stay in the country for a period or periods totallingone hundred eighty (180) days or more; participating in the management, supervision or control of any domestic business, firm, entity or corporation in the Philippines; and any other act or acts that imply a continuity of commercial dealings or arrangements, and contemplate to that extent the performance of acts or works, or the exercise of some of the functions normally incident to, and in progressive prosecution of, commercial gain or of the purpose and object of the business organization: Provided, however, That the phrase doing business shall not be deemed to include mere investment as a shareholder by a foreign entity in domestic corporations duly registered to do business, and/or the exercise of rights as such investor; nor having a nominee director or officer to represent its interests in such corporation; nor

appointing a representative or distributor domiciled in the Philippines which transacts business in its own name and for its own account. Since respondent is relying on Section 133 of the Corporation Code to bar petitioner from maintaining an action in Philippine courts, respondent bears the burden of proving that petitioners business activities in the Philippines were not just casual or occasional, but so systematic and regular as to manifest continuity and permanence of activity to constitute doing business in the Philippines. In this case, we find that respondent failed to prove that petitioners activities in the Philippines constitute doing business as would prevent it from bringing an action.The determination of whether a foreign corporation is doing business in the Philippines must be based on the facts of each case. The Implementing Rules and Regulations of RA 7042 provide under Section 1(f), Rule I, that doing business does not include the following acts: 1. Mere investment as a shareholder by a foreign entity in domestic corporations duly registered to do business, and/or the exercise of rights as such investor; 2. Having a nominee director or officer to represent its interests in such corporation; 3. Appointing a representative or distributor domiciled in the Philippines which transacts business in the representative's or distributor's own name and account; 4. The publication of a general advertisement through any print or broadcast media; 5. Maintaining a stock of goods in the Philippines solely for the purpose of

having the same processed by another entity in the Philippines; 6. Consignment by a foreign entity of equipment with a local company to be used in the processing of products for export; 7. Collecting information in the Philippines; and 8. Performing services auxiliary to an existing isolated contract of sale which are not on a continuing basis, such as installing in the Philippines machinery it has manufactured or exported to the Philippines, servicing the same, training domestic workers to operate it, and similar incidental services.

Philippine Veterans Bank Employees Union-NUBE vs. Vega GR 105364 28 June 2001 Facts: The Central Bank of the Philippines filed with the RTC of Manila a Petition for Assistance in the Liquidation of the Philippine Veterans Bank. Thereafter, the Philippine Veterans Bank Employees Union-N.U.B.E., represented by Perfecto V. Fernandez, filed claims for accrued and unpaid employee wages and benefits with said court. After lengthy proceedings, partial payment of the sums due to the employees were made. However, due to the piecemeal hearings on the benefits, many remain unpaid. Fernandez moved to disqualify the presiding judge thereat (RTC Br. 39), Judge Benjamin Vega, from hearing the case on grounds of bias and hostility towards petitioners. Congress enacted RA 7169 entitled "An Act To Rehabilitate The Philippine Veterans Bank Created Under Republic Act 3518, Providing The Mechanisms Therefor, And For Other Purposes. Thereafter, Fernandez filed with the labor tribunals their residual claims for benefits and for reinstatement upon reopening of the bank. RA 7169 provides in part for the reopening of the Philippine Veterans Bank together with all its branches within the period of 3 years from the date of the reopening of the head office. The law likewise provides for the creation of a rehabilitation committee in order to facilitate the implementation of the provisions of the same. Pursuant to thereto, the Rehabilitation Committee submitted the proposed Rehabilitation Plan of the PVB to the Monetary Board for its approval. Meanwhile, PVB filed a Motion to Terminate Liquidation of Philippine Veterans Bank with Judge Vega praying that the liquidation proceedings be immediately terminated in view of the passage of RA 7169. The Monetary Board issued Monetary Board Resolution 348 which approved the Rehabilitation Plan submitted by the Rehabilitation Committee. Thereafter, the Monetary Board issued a Certificate of Authority allowing PVB to reopen. The Central Bank issued a certificate of authority allowing the PVB to reopen. Despite the legislative mandate for

Other factors which support the finding that petitioner is not doing business in the Philippines are: (1) petitioner does not have an office in the Philippines; (2) petitioner imports products from the Philippines through its non-exclusive local broker, whose authority to act on behalf of petitioner is limited to soliciting purchases of products from suppliers engaged in the sugar trade in the Philippines; and (3) the local broker is an independent contractor and not an agent of petitioner. In the present case, petitioner is a foreign company merely importing molasses from a Philipine exporter. A foreign company that merely imports goods from a Philippine exporter, without opening an office or appointing an agent in the Philippines, is not doing business in the Philippines.

rehabilitation and reopening of PVB, Judge Vega continued with the liquidation proceedings of the bank. The Central Bank was set to order the payment and release of employee benefits upon motion of another lawyer, while employees union claims have been frozen to their prejudice. The liquidator filed A Motion for the Termination of the Liquidation Proceedings of the Philippine Veterans Bank with Judge Vega. Fernandez, on the other hand, filed the petition for Prohibition with Petition for Preliminary Injunction and application for Ex Parte Temporary Restraining Order. The Supreme Court resolved to issue a Temporary Restraining Order enjoining the trial court from further proceeding with the case. MOP Security & Detective Agency (VOPSDA) and its 162 security guards filed a Motion for Intervention with prayer that they be excluded from the operation of the Temporary Restraining Order issued by the Court. The Philippine Veterans Bank opened its doors to the public and started regular banking operations. Issue: Whether a liquidation court can continue with liquidation proceedings of the Philippine Veterans Bank when Congress had mandated its rehabilitation and reopening. Held: The enactment of Republic Act 7169, as well as the subsequent developments has rendered the liquidation court functus officio. Consequently, Judge Vega has been stripped of the authority to issue orders involving acts of liquidation. Liquidation, in corporation law, connotes a winding up or settling with creditors and debtors. It is the winding up of a corporation so that assets are distributed to those entitled to receive them. It is the process of reducing assets to cash, discharging liabilities and dividing surplus or loss. On the opposite end of the spectrum is rehabilitation which connotes a reopening or reorganization. Rehabilitation contemplates a continuance of corporate life and activities in an effort to restore and reinstate the corporation to its former position of successful operation and solvency. It is crystal clear that the concept of liquidation is diametrically

opposed or contrary to the concept of rehabilitation, such that both cannot be undertaken at the same time. To allow the liquidation proceedings to continue would seriously hinder the rehabilitation of the subject bank.

Pepsi-Cola Products vs. Court of Appeals G.R. No. 145855. November 24, 2004 Facts: Pepsi-Cola Products Philippines, Inc. Employees and Workers Union (PCEWU) is a duly- registered labor union of the employees of the Pepsi-Cola Distributors of the Philippines (PCDP). PCEWU, through its local union president Bombeo, filed a Complaint against PCDP with the DOLE for payment of overtime pay of 53 of its members working as salesman, warehousemen, truck helpers, route salesmen, route sales workers, distributors, conductors and forklift operators, on 8 days dulydesignated as Muslim holidays for 1985. PCDP maintained that there were only 5 legal Muslim holidays under the Muslim Code. It asserted that under the law, the cities of Cagayan de Oro and Dipolog were not included in the areas that officially observed the Muslim holidays, and that the said holidays were only applicable to Muslims. It also argued that even assuming that the employees were entitled to such overtime pay, only the rank-and-file employees and not the managerial employees should be given such benefit. The Executive Labor Arbiter (ELA) rendered a Decision in favor of PCEWU while the NLRC, upon appeal, affirmed the decision of the ELA with modification that the Labor Arbiter is directed to conduct further proceedings and that the monetary award is vacated. The PCDP and the employees filed their respective motions for partial reconsideration of the NLRC decision. Pending resolution thereof, ownership of various Pepsi-Cola bottling plants was transferred to petitioner Pepsi-Cola Products Philippines, Inc. (PCPPI). The PCDP alleged that it had ceased to exist as a corporation and that it has winded up its corporate

affairs in accordance with law. It also averred that it was now owned by PCPPI. Thus, the NLRC dismissed the complaint of PCEWU ruling that it was not competent for it to proceed against the PCDP because it had ceased to exist as a juridical entity. The petitioner filed a petition for the nullification of the Resolution of the NLRC which the Supreme Court referred to the Court of Appeals. For its part, the respondent averred that notwithstanding the dissolution of the PCDP while the complaint was pending resolution by the NLRC, the latter continued existing as a corporation for a period of three years from the time when it would have been dissolved, conformably to Section 122 of the Corporation Code. In its comment on the petition, the Office of the Solicitor General recommended that the petition be granted and that the NLRC be ordered to resolve the motions for reconsideration of the petitioner and respondent therein. The CA rendered judgment annulling the resolution of the NLRC. The CA declared that the PCDP was still in existence when the complaint was filed, and that the supervening dissolution of the corporation did not warrant the dismissal of the complaint against it. Every corporation is given 3 years to wind up its affairs. Hence, in case any litigation is filed by or against the corporation within the period which could not be terminated within the expiration of the same, such period must necessarily be prolonged until the final determination of the case, for if the rule were otherwise, corporations in liquidation would lose what should justly belong to them or would be exempt from the payment of just obligations through mere technicality, something that courts should not countenance. The CA set aside the decision of the NLRC and reinstated the decision of the ELA. Issue: WON the PCDP was still in existence and thus does not warrant dismissal of the labor complaint against it. Held: Yes. Under Section 122 of the Corporation Code, a corporation whose corporate existence is terminated in any manner continues to be a

body corporate for 3 years after its dissolution for purposes of prosecuting and defending suits by and against it and to enable it to settle and close its affairs, culminating in the disposition and distribution of its remaining assets. It may, during the 3-year term, appoint a trustee or a receiver who may act beyond that period. At any time during the said 3 years, the corporation is authorized and empowered to convey all of its properties to trustees for the benefit of stockholders, members, creditors, and other persons in interest. From and after any such conveyance by the corporation of its properties in trust for the benefit of its stockholders, members, creditors and others in interest, all interest which the corporation had in the properties terminates the legal interest vests in the trustees, and the beneficial interest in the stockholders, members, creditors or other persons in interest. Upon the winding up of the corporate affairs, any asset distributable to any creditor or stockholder or member, who is unknown or cannot be found, shall be escheated to the city or municipality where such assets are located. Except by decrease of capital stock and as otherwise allowed by this Code, no corporation shall distribute any of its assets or property except upon lawful dissolution and after payment of all its debts and liabilities. The termination of the life of a corporate entity does not by itself cause the extinction or diminution of the rights and liabilities of such entity. If the three-year extended life has expired without a trustee or receiver having been expressly designated by the corporation, within that period, the board of directors (or trustees) itself, may be permitted to so continue as "trustees" by legal implication to complete the corporate liquidation. SEC vs. INTERPORT RESOURCES CORPORATION G. R. No . 1 3 5 8 0 8 O cto b er 6 , 2 0 0 8 Facts: The Board of Directors of IRC approved a Memorandum of Agreement with GHB (Ganda Holdings Berhad). Under said memorandum

of agreement, IRC acquired 100% of the entire capital stock of GEHI (Ganda Energy Holdings Inc.) which would own and operate a 102 megawatt gas turbine power generating barge. In exchange, IRC will issue to GHB 55% of the expanded capital stock of IRC. On the side, IRC would acquire 67% of the entire capital of PRCI (Philippine Racing Club). It is alleged herein that a press release announcing the approval of the agreement was sent to the Philippine Stock Exchange through facsimile and the SEC, but the facsimile machine of the SEC could not receive it. However, the SEC received reports that the IRC failed to make timely public disclosures of its negotiations with GHB and that some of its directors, heavily traded IRC shares utilizing this material insider information. For this reason, the SEC required the directors to appear before the SEC to explain the alleged failure to disclose material information as required by the Rules on Disclosure of Material Facts. Unsatisfied with the explanation, the SEC issued an order finding that the IRC violated the Rules in connection with the then Old Securities Act when it failed to make timely disclosures of its negotiations with GHB. In addition, the SEC found that the directors of IRC entered into transactions involving IRC shares in violation of the Revised Securities Act. Respondents, however, questioned the authority of the SEC to investigate on said matter since according to PD 902-A, jurisdiction upon the matter was conferred upon the PED (Prosecution and Enforcement Department) of the SEC however, this issue is already moot since pending the disposition of the case, the Securities Regulation Code was passed thereby effectively repealing PD 902-A and abolishing the PED. They also contended that their right to due process was violated when the SEC required them to appear before the SEC to show cause why sanctions should not be imposed upon them since such requirement shifted the burden of proof to respondents. The case reached the CA and said court ruled in favor of the respondents and effectively enjoined the SEC from filing any criminal, civil or administrative cases against respondents. In its resolution, the CA stated

that since there are no rules and regulations implementing the rules regarding disclosure, insider trading, or any of the provisions of the Revised Securities Act, the SEC has no statutory authority to file any suit against respondents. The CA, therefore, prohibited the SEC from taking cognizance or initiating any action against the respondents for the alleged violations of the Revised Securities Act. While this case was pending in this Court, Republic Act No. 8799, otherwise known as the Securities Regulation Code, took effect on 8 August 2000. Section 8 of Presidential Decree No. 902-A, as amended, which created the PED, was already repealed as provided for in Section 76 of the Securities Regulation Code Issues: (1) Whether or not the SEC has authority to file suit against respondents for violations of the RSA (2) Whether or not their right to due process was violated when the SEC denied the parties of their right to cross-examination Held: (1) Yes. Sections 8, 30 and 36 of the Revised Securities Act do not require the enactment of implementing rules to make them binding and effective. The provisions of the RSA are sufficiently clear and complete by themselves. The requirements are specifically set out and the acts which are enjoined are determinable. To rule that absence of implementing rules can render ineffective an act of Congress would empower administrative bodies to defeat the legislative will by delaying the implementing rules. Where the statute contains sufficient standards and an unmistakable intent there should be no impediment as to its implementation. The provision explains in simple terms that the insider's misuse of non-public and undisclosed information is the gravamen of illegal conduct and that the intent of the law is the protection of investors against fraud committed when an insider, using secret information, takes advantage of an uninformed investor. Insiders are obligated to disclose material information to the other party or abstain from trading the shares of his corporation. This

duty to disclose or abstain is based on 2 factors: (1) the existence of a relationship giving access, directly or indirectly to information intended to be available only for a corporate purpose and not for the personal benefit of anyone; and (2) the inherent unfairness involved when a party takes advantage of such information knowing it is unavailable to those with whom he is dealing. This obligation to disclose is imposed upon "insiders" which are particularly officers, directors or controlling stockholders but that definition has already been expanded and does not include those persons whose relationship of former relationship to the issuer or the security that is not generally available and the one who learns such a fact from an insider knowing that the person from whom he learns such fact is an insider. In some case, however, there may be valid corporate reasons for the nondisclosure of material information but it should not be used for noncorporate purposes. Respondent contends that the terms "material fact", "reasonable person", "nature and reliability" and "generally available" are vaguely used in the RSA because under the provision of the said law what is required to be disclosed is a fact of special significance. But the court dismissed said contention and stated that material fact is already defined and explained as one which induces or tends to induce or otherwise affect the sale or purchase of securities. On the other hand, "reasonable person" has already been used many times in jurisprudence and in law since it is a standard on which most of legal doctrines stand (even the doctrine on negligence uses such standard) and it has been held to mean "a man who relies on the calculus of common sense of which all reasonable men have in abundance" As to "nature and reliability" the proper adjudicative body would be able to determine if facts of a certain nature and reliability can influence a reasonable person's decision to retain, buy or sell securities and thereafter explain and justify its factual findings in its decision since the same must be viewed in connection with the particular circumstances of a

case. As to "generally available", the court held also that such is a matter which may be adjudged given the particular circumstances of the case. The standards of which cannot remain at a standstill. (2) No. There is no violation of due process in this case since the proceedings before the PED are summary in nature. The hearing officer may require the parties to submit their respective verified position papers together will all supporting documents and affidavits of witnesses. A formal hearing is not mandatory and it is within the discretion of the hearing officer to determine whether or not there is a need for a formal hearing. Moreover, the law creating the PED empowers it to investigate violations of the rules and regulations and to file and prosecute such cases. It does not have adjudicatory powers. Thus, the PED need not comply with the provisions of the Administrative Code on adjudication. The SEC retained jurisdiction to investigate violations of the RSA, reenacted in the Securities Regulations Code despite the abolition of the PED. In this case, the SEC already commenced investigating the respondents for violations of the RSA but during the pendency of the case the Securities and Regulations Code was passed thereby repealing the RSA. However, the repeal cannot deprive the SEC of its jurisdiction to continue investigating the case. Investigations by the SEC is a requisite before a criminal case may be referred to the DOJ since the SEC is an administrative agency with the special competence to do so. According to the doctrine of primary jurisdiction, the courts will not determine a controversy involving a question within the jurisdiction of an administrative tribunal where the question demands the exercise of sound administrative discretion requiring the specialized knowledge and expertise of said administrative tribunal to determine technical and intricate matters of fact.

UNIVERSAL RUBBER PRODUCTS, INC., petitioner, vs. HON. COURT OF APPEALS, CONVERSE RUBBER CORPORARION, EDWARDSON MANUFACTURING CO., INC. AND HON. PEDRO C. NAVARRO, respondents. FACTS: Two respondent corporations herein sued the present petitioner before the Court of First Instance of Rizal for unfair competition with damages and attorney's fees. After they have presented about nine witnesses and various pieces of documentary evidence, herein private respondents made a request to the respondent Judge to issue a subpoena duces tecum against the treasurer of herein petitioner. Acting favorably on that request, said respondent Judge issued a subpoena duces tecum. Petitioner filed a motion in the court below praying that the subpoena duces tecum be quashed on the grounds that: (1) the said subpoena is both unreasonable and oppressive as the books and documents caned for are numerous and voluminous; (2) there is no good cause shown for the issuance thereof; and (3) the books and documents are not relevant to the case pending below. The private respondents herein opposed that motion of the petitioner. Acting on the said motion and on the opposition thereto, respondent Judge issued the first controverted order denying the motion to quash the subpoena duces tecum. Consequently, petitioner Universal Rubber Products, Inc. filed its present petition for certiorari with preliminary injunction, alleging that in so denying its motion to quash the subpoena duces tecumand its subsequent motion for reconsideration, respondent Judge acted with grave abuse of discretion amounting to an excess of jurisdiction. The respondent Court rendered its decision denying the petition for certiorari filed by petitioner for lack of merit. ISSUE: whether the issuance of the "subpoena duces tecum" is proper in a suit for unfair competition. HELD:

Well-settled is Our jurisprudence that, in order to entitle a party to the issuance of a "subpoena duces tecum ", it must appear, by clear and unequivocal proof, that the book or document sought to be produced contains evidence relevant and material to the issue before the court, and that the precise book, paper or document containing such evidence has been so designated or described that it may be identified. A "subpoena duces tecum once issued by the court may be quashed upon motion if the issuance thereof is unreasonable and oppressive or the relevancy of the books, documents or things does not appear, or if the persons in whose behalf the subpoena is issued fails to advance the reasonable cost of production thereof. Petitioner also assails that private respondent is a foreign corporation not licensed to do business in the Philippines and that respondent Edwardson is merely its licensee; that respondent Converse has no goodwill to speak of and that it has no registrable right over its own name. We have already answered this issue squarely in Our decision of the case of Converse Rubber Corporation vs. Jacinto Rubber & Plastic Co., Inc., where We explained: The disability of a foreign corporation from suing in the Philippines is limited to suits to enforce any legal of contract rights arising from, or growing out, of any business which it has transacted in the Philippine Islands ... On the other hand, where the purpose of the suit is "to protect its reputation, its corporate name, its goodwill, whenever that reputation, corporate name or goodwill have, through the natural development of its trade, established themselves", an unlicensed foreign corporation may sue in the Philippines. So interpreted by the Supreme Court, it is clear that Section 29 of the Corporation Law does not

disqualify plaintiff-appellee Converse Rubber, which does not have a branch office in any part of the Philippines and is not "doing business" in the Philippines, from filing and prosecuting this action for unfair competition. LORENZO SHIPPING CORP., petitioner, vs. CHUBB and SONS, Inc., GEARBULK, Ltd. and PHILIPPINE TRANSMARINE CARRIERS, INC.,respondents. FACTS: Mayer Steel Pipe Corporation of Binondo, Manila, loaded 581 bundles of ERW black steel pipes on board the vessel M/V Lorcon IV, owned by petitioner Lorenzo Shipping, for shipment to Davao City. Petitioner Lorenzo Shipping issued a clean bill of lading designated for the account of the consignee, Sumitomo Corporation of San Francisco, California, USA, which in turn, insured the goods with respondent Chubb and Sons, Inc. The M/V Lorcon IV arrived at the Sasa Wharf in Davao City. Respondent Transmarine Carriers received the subject shipment which was discharged and discovered seawater in the hatch of M/V Lorcon IV, and found the steel pipes submerged in it. Del Pans Survey Report showed that the subject shipment was no longer in good condition, as in fact, the pipes were found with rust formation on top and/or at the sides. Due to its heavily rusted condition, the consignee Sumitomo rejected the damaged steel pipes and declared them unfit for the purpose they were intended It then filed a marine insurance claim with respondent Chubb and Sons, Inc. Respondent Chubb and Sons, Inc. filed a complaint for collection of a sum of money, against respondents Lorenzo Shipping, Gearbulk, and Transmarine. Respondent Chubb and Sons, Inc. alleged that it is not doing business in the Philippines, and that it is suing under an isolated transaction. Respondents Gearbulk and Transmarine filed their answer with counterclaim and cross-claim against petitioner Lorenzo Shipping denying liability on the following grounds: (a) respondent Chubb and Sons, Inc. has no capacity to sue before Philippine courts; (b) the action should be dismissed on the ground of forum non conveniens; (c) damage to the steel pipes was due to the inherent nature of the goods or to the

insufficiency of packing thereof; (d) damage to the steel pipes was not due to their fault or negligence; and, (e) the law of the country of destination, U.S.A., governs the contract of carriage. he Regional Trial Court ruled in favor of the respondent Chubb and Sons, Inc., finding that: (1) respondent Chubb and Sons, Inc. has the right to institute this action; and, (2) petitioner Lorenzo Shipping was negligent in the performance of its obligations as a carrier. Petitioner Lorenzo Shipping appealed to the Court of Appeals insisting that: (a) respondent Chubb and Sons does not have capacity to sue before Philippine courts; and, (b) petitioner Lorenzo Shipping was not negligent in the performance of its obligations as carrier of the goods. The appellate court denied the petition and affirmed the decision of the trial court. Petitioner argues that respondent Chubb and Sons is a foreign corporation not licensed to do business in the Philippines, and is not suing on an isolated transaction. It contends that because the respondent Chubb and Sons is an insurance company, it was merely subrogated to the rights of its insured, the consignee Sumitomo, after paying the latters policy claim. Sumitomo, however, is a foreign corporation doing business in the Philippines without a license and does not have capacity to sue before Philippine courts. Since Sumitomo does not have capacity to sue, petitioner then concludes that, neither the subrogee-respondent Chubb and Sons could sue before Philippine courts ISSUE: Whether the respondent has the right to sue in the Philippines HELD: Yes. in the first place, petitioner failed to raise the defense that Sumitomo is a foreign corporation doing business in the Philippines without a license. It is therefore estopped from litigating the issue on appeal especially because it involves a question of fact which this Court cannot resolve. Secondly, assuming arguendo that Sumitomo cannot sue in the Philippines, it does not follow that respondent, as subrogee, has also no capacity to sue in our jurisdiction. Subrogation is the substitution of one person in the place of another with reference to a lawful claim or right, so that he who is substituted succeeds to the rights of the other in relation to a debt or claim, including its remedies or securities. The principle covers the situation under which an insurer that has paid a loss under an insurance policy is entitled to all the rights and remedies belonging to the insured

against a third party with respect to any loss covered by the policy. It contemplates full substitution such that it places the party subrogated in the shoes of the creditor, and he may use all means which the creditor could employ to enforce payment When the insurer succeeds to the rights of the insured, he does so only in relation to the debt. The person substituted (the insurer) will succeed to all the rights of the creditor (the insured), having reference to the debt due the latter. In the instant case, the rights inherited by the insurer, respondent Chubb and Sons, pertain only to the payment it made to the insured Sumitomo as stipulated in the insurance contract between them, and which amount it now seeks to recover from petitioner Lorenzo Shipping which caused the loss sustained by the insured Sumitomo. The capacity to sue of respondent Chubb and Sons could not perchance belong to the group of rights, remedies or securities pertaining to the payment respondent insurer made for the loss which was sustained by the insured Sumitomo and covered by the contract of insurance. Capacity to sue is a right personal to its holder. It is conferred by law and not by the parties. Lack of legal capacity to sue means that the plaintiff is not in the exercise of his civil rights, or does not have the necessary qualification to appear in the case, or does not have the character or representation he claims. It refers to a plaintiffs general disability to sue, such as on account of minority, insanity, incompetence, lack of juridical personality, or any other disqualifications of a party. Respondent Chubb and Sons who was plaintiff in the trial court does not possess any of these disabilities. On the contrary, respondent Chubb and Sons has satisfactorily proven its capacity to sue, after having shown that it is not doing business in the Philippines, but is suing only under an isolated transaction, i.e., under the one (1) marine insurance policy issued in favor of the consignee Sumitomo covering the damaged steel pipes. Art. 133 of the Corporation Code states: Doing business without a license. No foreign corporation transacting business in the Philippines without a license, or its successors or assigns, shall be permitted to maintain or intervene in any action, suit or proceeding in any court or administrative agency of the Philippines; but such corporation may be sued or proceeded against before Philippine courts or

administrative tribunals on any valid cause of action recognized under Philippine laws. The law does not prohibit foreign corporations from performing single acts of business. A foreign corporation needs no license to sue before Philippine courts on an isolated transaction. What is determinative of "doing business" is not really the number or the quantity of the transactions, but more importantly, the intention of an entity to continue the body of its business in the country. The number and quantity are merely evidence of such intention. The phrase "isolated transaction" has a definite and fixed meaning, i.e. a transaction or series of transactions set apart from the common business of a foreign enterprise in the sense that there is no intention to engage in a progressive pursuit of the purpose and object of the business organization. Whether a foreign corporation is "doing business" does not necessarily depend upon the frequency of its transactions, but more upon the nature and character of the transactions.

CEMCO HOLDINGS, INC. VS. NATIONAL LIFE INSURANCE COMPANY OF THE PHILIPPINES, INC. FACTS: Union Cement Corporation (UCC), a publicly-listed company, has two principal stockholders UCHC, a non-listed company, with shares amounting to 60.51%, and petitioner Cemco with 17.03%. Majority of UCHCs stocks were owned by BCI with 21.31% and ACC with 29.69%. Cemco, on the other hand, owned 9% of UCHC stocks. In a disclosure letter dated 5 July 2004, BCI informed the Philippine Stock Exchange (PSE) that it and its subsidiary ACC had passed resolutions to sell to Cemco BCIs stocks in UCHC equivalent to 21.31% and ACCs stocks in UCHC equivalent to 29.69%. In the PSE Circular for Brokers No. 3146-2004 dated 8 July 2004, it was stated that as a result of petitioner Cemcos acquisition of BCI and ACCs shares in UCHC, petitioners total beneficial ownership, direct and indirect, in UCC has

increased by 36% and amounted to at least 53% of the shares of UCC. As a consequence of this disclosure, the PSE, in a letter to the SEC dated 15 July 2004, inquired as to whether the Tender Offer Rule under Rule 19 of the Implementing Rules of the Securities Regulation Code is not applicable to the purchase by petitioner of the majority of shares of UCC. In a letter dated 16 July 2004, Director Justina Callangan of the SECs Corporate Finance Department responded to the query of the PSE that while it was the stance of the department that the tender offer rule was not applicable, the matter must still have to be confirmed by the SEC en banc. Director Callangan confirmed that the SEC en banc had resolved that the Cemco transaction was not covered by the tender offer rule. Respondent, a minority stockholder of UCC, sent a letter to Cemco demanding the latter to comply with the rule on mandatory tender offer. Cemco, however, refused. A Share Purchase Agreement was executed by ACC and BCI, as sellers, and Cemco, as buyer. Respondent filed a complaint with the SEC asking it to reverse its 27 July 2004 Resolution and to declare the purchase agreement of Cemco void and praying that the mandatory tender offer rule be applied to its UCC shares. Impleaded in the complaint were Cemco, UCC, UCHC, BCI and ACC, which were then required by the SEC to file their respective comment on the complaint. In their comments, they were uniform in arguing that the tender offer rule applied only to a direct acquisition of the shares of the listed company and did not extend to an indirect acquisition arising from the purchase of the shares of a holding company of the listed firm. The SEC ruled in favor of the respondent by reversing and setting aside its 27 July 2004 Resolution and directed petitioner Cemco to make a tender offer for UCC shares to respondent and other holders of UCC shares similar to the class held by UCHC in accordance with Section 9(E), Rule 19 of the Securities Regulation Code. Petitioner filed a petition with the Court of Appeals challenging the SECs jurisdiction to take cognizance of respondents complaint and its authority to require Cemco to make a tender offer for UCC shares, and arguing that the tender offer rule does not apply, or that the SECs reinterpretation of the rule could not be made to retroactively apply to Cemcos purchase of UCHC shares. The Court of Appeals rendered a decision affirming the ruling of the SEC. It ruled that the SEC has jurisdiction to render the questioned decision and, in any event, Cemco was barred by estoppel from questioning the SECs jurisdiction.

ISSUE:
1.

Whether or not the SEC has jurisdiction over respondents complaint and to require Cemco to make a tender offer for respondents UCC shares. Whether or not the rule on mandatory tender offer applies to the indirect acquisition of shares in a listed company, in this case, the indirect acquisition by Cemco of 36% of UCC, a publicly-listed company, through its purchase of the shares in UCHC, a non-listed company.

2.

HELD: 1. YES. In taking cognizance of respondents complaint against petitioner and eventually rendering a judgment which ordered the latter to make a tender offer, the SEC was acting pursuant to Rule 19(13) of the Amended Implementing Rules and Regulations of the Securities Regulation Code, to wit: 13. Violation If there shall be violation of this Rule by pursuing a purchase of equity shares of a public company at threshold amounts without the required tender offer, the Commission, upon complaint, may nullify the said acquisition and direct the holding of a tender offer. This shall be without prejudice to the imposition of other sanctions under the Code.

The foregoing rule emanates from the SECs power and authority to regulate, investigate or supervise the activities of persons to

ensure compliance with the Securities Regulation Code, more specifically the provision on mandatory tender offer under Section 19 thereof. Moreover, petitioner is barred from questioning the jurisdiction of the SEC. It must be pointed out that petitioner had participated in all the proceedings before the SEC and had prayed for affirmative relief. 2. Tender offer is a publicly announced intention by a person acting alone or in concert with other persons to acquire equity securities of a public company. A public company is defined as a corporation which is listed on an exchange, or a corporation with assets exceeding P50,000,000.00 and with 200 or more stockholders, at least 200 of them holding not less than 100 shares of such company[ Stated differently, a tender offer is an offer by the acquiring person to stockholders of a public company for them to tender their shares therein on the terms specified in the offer. Tender offer is in place to protect minority shareholders against any scheme that dilutes the share value of their investments. It gives the minority shareholders the chance to exit the company under reasonable terms, giving them the opportunity to sell their shares at the same price as those of the majority shareholders. Under existing SEC Rules, the 15% and 30% threshold acquisition of shares under the foregoing provision was increased to thirty-five percent (35%). It is further provided therein that mandatory tender offer is still applicable even if the acquisition is less than 35% when the purchase would result in ownership of over 51% of the total outstanding equity securities of the public company.

PHILIPPINE SAVINGS BANK VS. SENATE IMPEACHMENT COURT G.R NO. 200238 FACTS: Philippine Savings Bank (PS Bank) and its President, Pascual M. Garcia III, filed before the Supreme Court an original civil action for certiorari and prohibition with application for temporary restraining order and/or writ of preliminary injunction. The TRO was sought to stop the Senate, sitting as impeachment court, from further implementing the Subpoena Ad Testificandum et Duces Tecum, dated February 6, 2012, that it issued against the Branch Manager of PS Bank, Katipunan Branch. The subpoena assailed by petitioners covers the foreign currency denominated accounts allegedly owned by the impeached Chief Justice Renato Corona of the Philippine Supreme Court.

ISSUE: Should a TRO be issued against the impeachment court to enjoin it from further implementing the subpoena with respect to the alleged foreign currency denominated accounts of CJ Corona?

HELD: YES, a TRO should be issued against the impeachment court to enjoin it from further implementing the subpoena with respect to the alleged foreign currency denominated accounts of CJ Corona. There are two requisite conditions for the issuance of a preliminary injunction: (1) the right to be protected exists prima facie, and (2) the acts sought to be enjoined are violative of that right. It must be proven that the violation sought to be prevented would cause an irreparable injustice.

A clear right to maintain the confidentiality of the foreign currency deposits of the Chief Justice is provided under Section 8 of Republic Act No. 6426, otherwise known as the Foreign Currency Deposit Act of the Philippines (RA 6426). This law establishes the absolute confidentiality of foreign currency deposits: Under R.A. No. 6426 there is only a single exception to the secrecy of foreign currency deposits, that is, disclosure is allowed only upon the written permission of the depositor. In Intengan v. Court of Appeals, the Court ruled that where the accounts in question are U.S. dollar deposits, the applicable law is not Republic Act No. 1405 but RA 6426. Similarly, in the recent case of Government Service Insurance System v. 15th Division of the Court of Appeals, the Court also held that RA 6426 is the applicable law for foreign currency deposits and not Republic Act No. 1405. The written consent under RA 6426 constitutes a waiver of the depositors right to privacy in relation to such deposit. In the present case, neither the prosecution nor the Impeachment Court has presented any such written waiver by the alleged depositor, Chief Justice Renato C. Corona. Also, while impeachment may be an exception to the secrecy of bank deposits under RA 1405, it is not an exemption to the absolute confidentiality of foreign currency deposits under RA 6426.

Vous aimerez peut-être aussi